bstrandable NCLEX Miscellaneous 9

Réussis tes devoirs et examens dès maintenant avec Quizwiz!

Which of these factors is most likely to affect the nutritional status of an elderly person?

B) Living alone on a fixed income

84. During auscultation of the lungs of an adult patient, the nurse notices the presence of bronchophony. The nurse should assess for signs of which condition? A) Airway obstruction B) Emphysema C) Pulmonary consolidation D) Asthma

C) Pulmonary consolidation Page: 446. Pathologic conditions that increase lung density, such as pulmonary consolidation, will enhance transmission of voice sounds, such as bronchophony. See Table 18-7.

14. If an American Indian has come to the clinic to seek help with regulating her diabetes, the nurse can expect that she:

C) may also be seeking the assistance of a shaman or medicine man.

54. A 65-year-old man with emphysema and bronchitis has come to the clinic for a follow-up appointment. On assessment, the nurse might expect to see which assessment finding?

D) Clubbing of the nails

30. During a mental status examination, the nurse wants to assess a patient's affect.

A) "How do you feel today?"

46. During a nutritional assessment, why is it important for the nurse to ask a patient what medications he or she is taking?

A) Certain drugs can affect the metabolism of nutrients.

64. The nurse is performing an assessment on a 7-year-old child who has symptoms of chronic watery eyes, sneezing, and clear nasal drainage. The nurse notices the presence of a transverse line across the bridge of the nose, dark blue shadows below the eyes, and a double crease on the lower eyelids. These findings are characteristic of:

A) allergies.

17. The nurse recognizes that working with children with a different cultural perspective may be especially difficult because:

A) children have spiritual needs that are influenced by their stages of development.

37. When percussing over the liver of a patient, the nurse notices a dull sound. The nurse should:

A) consider this a normal finding.

8. The nurse is conducting an interview. Which of these statements is true regarding open-ended questions? Select all that apply.

B) They allow for self-expression. C) They build and enhance rapport. F) They are used when narrative information

96. During an assessment, the nurse uses the "profile sign" to detect: A) pitting edema. B) early clubbing. C) symmetry of the fingers. D) insufficient capillary refill.

B) early clubbing. Page: 506. The nurse should use the profile sign (viewing the finger from the side) to detect early clubbing.

56. A 40-year-old woman reports a change in mole size, accompanied by; color changes, itching, burning, and bleeding over the past month. She has a dark complexion and has no family history of skin cancer, but she has had many blistering sunburns in the past. The nurse would:

B) refer the patient because of the suspicion of melanoma on the basis of her symptoms.

13. An individual who takes the magicoreligious perspective of illness and disease is likely to believe that his or her illness was caused by:

B) supernatural forces.

25. The nurse is performing a functional assessment on an 82-year-old patient who recently had a stroke. Which of these questions would be most important to ask?

B. "Are you able to dress yourself?" Page: 67. Functional assessment measures how a person manages day-to-day activities. For the older person, the meaning of health becomes those activities that they can or cannot do.

26. The nurse is conducting a developmental history on a 5-year-old child. Which questions are appropriate to ask the parents for this part of the assessment? Select all that apply.

B. "How many teeth has he lost, and when did he lose them?" C. "Is he able to tie his shoelaces?" E. "Can he tell time?"

12. Among many Asians there is a belief in the yin/yang theory, rooted in the ancient Chinese philosophy of Tao. The nurse recognizes which statement that most accurately reflects "health" in an Asian with this belief?

C) All aspects of the person are in perfect balance.

78. A 65-year-old patient with a history of heart failure comes to the clinic with complaints of "being awakened from sleep with shortness of breath." Which action by the nurse is most appropriate? A) Obtain a detailed history of the patient's allergies and history of asthma. B) Tell the patient to sleep on his or her right side to facilitate ease of respirations. C) Assess for other signs and symptoms of paroxysmal nocturnal dyspnea. D) Assure the patient that this is normal and will probably resolve within the next week.

C) Assess for other signs and symptoms of paroxysmal nocturnal dyspnea. Pages: 419-420. The patient is experiencing paroxysmal nocturnal dyspnea: being awakened from sleep with shortness of breath and the need to be upright to achieve comfort.

60. During an examination, the nurse knows that Paget's disease would be indicated by which of these assessment findings?

C) Headache, vertigo, tinnitus, and deafness

32. The nurse is performing a mental status examination. Which statement is true regarding the assessment of mental status?

C) Mental status functioning is inferred through assessment of an individual's behaviors.

27. During an examination, the nurse can assess mental status by which activity?

C) Observing the patient and inferring health or dysfunction

58. A patient has been admitted for severe psoriasis. The nurse can expect to see what finding in the patient's fingernails?

C) Pitting

55. The nurse has discovered decreased skin turgor in a patient and knows that this is an expected finding in which of these conditions?

C) Severe dehydration

67. The nurse notices the presence of periorbital edema when performing an eye assessment on a 70-year-old patient. The nurse should: A) check for the presence of exophthalmos. B) suspect that the patient has hyperthyroidism. C) ask the patient if he or she has a history of heart failure. D) assess for blepharitis because this is often associated with periorbital edema.

C) ask the patient if he or she has a history of heart failure. Page: 312. Periorbital edema occurs with local infections, crying, and systemic conditions such as heart failure, renal failure, allergy, and hypothyroidism. Periorbital edema is not associated with blepharitis.

80. The nurse is reviewing the technique of palpating for tactile fremitus with a new graduate. Which statement by the graduate nurse reflects a correct understanding of tactile fremitus? "Tactile fremitus: A) is caused by moisture in the alveoli." B) indicates that there is air in the subcutaneous tissues." C) is caused by sounds generated from the larynx." D) reflects the blood flow through the pulmonary arteries."

C) is caused by sounds generated from the larynx." Pages: 422-423. Fremitus is a palpable vibration. Sounds generated from the larynx are transmitted through patent bronchi and the lung parenchyma to the chest wall where they are felt as vibrations. Crepitus is the term for air in the subcutaneous tissues.

28. The nurse is assessing a 75-year-old man. As the nurse begins the mental status portion of the assessment, the nurse expects that this patient:

C) may take a little longer to respond, but his general knowledge and abilities should not have declined.

77. During an assessment, the nurse knows that expected assessment findings in the normal adult lung include the presence of: A) adventitious sounds and limited chest expansion. B) increased tactile fremitus and dull percussion tones. C) muffled voice sounds and symmetrical tactile fremitus. D) absent voice sounds and hyperresonant percussion tones.

C) muffled voice sounds and symmetrical tactile fremitus. Pages: 429-430. Normal lung findings include symmetric chest expansion, resonant percussion tones, vesicular breath sounds over the peripheral lung fields, muffled voice sounds, and no adventitious sounds.

61. A woman comes to the clinic and states, "I've been sick for so long! My eyes have gotten so puffy, and my eyebrows and hair have become coarse and dry." The nurse will assess for other signs and symptoms of:

C) myxedema.

44. When assessing the quality of a patient's pain, the nurse should ask which question?

D) "What does your pain feel like?"

92. During an assessment of a healthy adult, where would the nurse expect to palpate the apical impulse? A) Third left intercostal space at the midclavicular line B) Fourth left intercostal space at the sternal border C) Fourth left intercostal space at the anterior axillary line D) Fifth left intercostal space at the midclavicular line

D) Fifth left intercostal space at the midclavicular line Pages: 473-474. The apical impulse should occupy only one intercostal space, the fourth or fifth, and it should be at or medial to the midclavicular line.

4. During an interview, a parent of a hospitalized child is sitting in an open position. As the interviewer begins to discuss his son's treatment, however, he suddenly crosses his arms against his chest and crosses his legs. This would suggest that the parent is:

D) uncomfortable talking about his son's treatment.

22. The mother of a 16-month-old toddler tells the nurse that her daughter has an earache. What would be an appropriate response?

D. "Please describe what she is doing to indicate she is having pain."

24. The nurse is preparing to do a functional assessment. Which statement best describes the purpose of a functional assessment?

D. It helps to determine how a person is managing day-to-day activities.

7. A female patient does not speak English well, and the nurse needs to choose an interpreter. Which of the following would be the most appropriate choice?

A) A trained interpreter

1. In an interview, the nurse may find it necessary to take notes to aid his or her memory later. Which statement is true regarding note-taking?

A) Note-taking may impede the nurse's observation of the patient's nonverbal behaviors. Page: 31 Some use of history forms and note-taking may be unavoidable.

59. The nurse suspects that a patient has hyperthyroidism and laboratory data indicate that the patient's thyroxine and tri-iodothyronine hormone levels are elevated. Which of these findings would the nurse most likely find on examination?

A) Tachycardia

85. The nurse is listening to the breath sounds of a patient with severe asthma. Air passing through narrowed bronchioles would produce which of these adventitious sounds? A) Wheezes B) Bronchial sounds C) Bronchophony D) Whispered pectoriloquy

A) Wheezes Page: 445. Wheezes are caused by air squeezed or compressed through passageways narrowed almost to closure by collapsing, swelling, secretions, or tumors, such as with acute asthma or chronic emphysema.

33. When performing a physical assessment, the technique the nurse will always use first is:

B) inspection.

43. When assessing the force, or strength, of a pulse, the nurse recalls that it:

C) is a reflection of the heart's stroke volume.

Normal Creatinine

0.6-1.5

APGAR: HR

0=absent; 1=slow, <100; 2=over 100

APGAR: Respiration

0=absent; 1=slow, weak cry; 2=good cry

APGAR: Color

0=blue; 1=pink body, blue extremities; 2=pink

incomplete abortion

expulsion of part of products of conception NC: D&C, oxytocin

Bipolar disorders refers to a group of psychotic disorders that are evident in...

extreme changes in mood or affect. Believed to be caused by alterations in serotonin, dopamine, and norepinephrine.

Prior to an amniocentesis at 20 weeks gestation, the client should...

have a full bladder. After 20 weeks the client should be asked to void before the procedure.

A CPP above 70 is needed to...

have adequate brain viability.

Kernicterus

high bilirubin levels result in deposition of yellow pigmentation basal ganglia of brain=>irreversible retardation

Phenylketonuria: Assessment

high blood phenylalanine that leads to mental retardation

During the prenatal period, the client should eat foods...

high in vitamins and minerals.

If auscultating bowel sounds in pt w/obstruction what would you expect to hear?

high pitch above obstruction and absent below obstruction

Gout: Assessment

high uric acid level that leads to progressive joint deterioration

Excessively rapid uterine enlargement can indicate...

hydatidiform mole

Alcohol withdrawal includes;

hyper-alert, startles easily, anorexia, increased pulse, anxiety, tremors, insomnia, hallucinations

What would you teach a pt that has diverticulitis?

increase fiber, decrease fat and red meat, increase activity to increase peristalsis, avoid tight restrictive clothing

Gestational diabetes (GDM)

increased demand for insulin during pregnancy after 20 wks when insulin is needed. NO oral agent(terategenic)

Management of hirschusprung disease

increased fluid and fiber intake; administration of occassional enemas using isotonic or normal saline solutions

Hiatal Hernia:

opening in diaphragm through which the esophagus passes becomes enlarged, part of the upper stomach comes up into the lower portion of the thorax

Diverticulosis:

out pouching in intestinal wall

Reaction formation

outward expression of feelings that are opposite to those experienced

<1.010

overhydration

Symptoms of stage 3 (12-24 hours after last use)

severe anxiety, increased BP, profuse sweating, severe hallucinations, and grand mal seizures

A GCS less than or equal to 8 indicates...

severe coma.

second stage of labor

short period of peace and increase in bloody show, baby is born

Cerebral perfusion pressure (CPP) is calculated by...

subtracting the ICP reading from the mean arterial pressure (MAP).

Treatment of MG includes...

supportive care and medications (Pyridostigmine Bromide (Mestinon), neostigmine bromide (Prostigmin) and steroids))

73. The nurse is performing an ear examination of an 80-year-old patient. Which of these would be considered a normal finding? A) A high-tone frequency loss B) Increased elasticity of the pinna C) A thin, translucent membrane D) A shiny, pink tympanic membrane

A) A high-tone frequency loss Pages: 337-338. A high-tone frequency hearing loss is apparent for those affected with presbycusis, the hearing loss that occurs with aging. The pinna loses elasticity, causing earlobes to be pendulous. The eardrum may be whiter in color and more opaque and duller than in the young adult.

10. During an assessment, the nurse notices that a patient is handling a small charm that is tied to a leather strip around his neck. Which action by the nurse is appropriate?

A) Ask the patient about the item and its significance.

76. When assessing a patient's lungs, the nurse recalls that the left lung: A) consists of two lobes. B) is divided by the horizontal fissure. C) consists primarily of an upper lobe on the posterior chest. D) is shorter than the right lung because of the underlying stomach.

A) consists of two lobes. Pages: 413-414. The left lung has two lobes, and the right lung has three lobes. The right lung is shorter than the left lung because of the underlying liver. The left lung is narrower than the right lung because the heart bulges to the left. The posterior chest is almost all lower lobe.

74. During an examination, the patient states he is hearing a buzzing sound and says that it is "driving me crazy!" The nurse recognizes that this symptom indicates: A) vertigo. B) pruritus. C) tinnitus. D) cholesteatoma.

C) tinnitus. Pages: 328-329. Tinnitus is a sound that comes from within a person; it can be a ringing, crackling, or buzzing sound. It accompanies some hearing or ear disorders.

38. The nurse is unable to palpate the right radial pulse on a patient. The best action would be to:

C) use a Doppler device to check for pulsations over the area.

3. A nurse is taking complete health histories on all of the patients attending a wellness workshop. On the history form, one of the written questions asks, "You don't smoke, drink, or take drugs, do you?" This question is an example of:

C) using biased or leading questions.

35. The nurse is preparing to assess a patient's abdomen by palpation. How should the nurse proceed?

D) Start with light palpation to detect surface characteristics and to accustom the patient to being touched.

What is glomerulonephritis?

-loss of kidney function -acute lasts 5-21 days -chronic after acute phase or slowly over time

Non-stress Test (NST)

-ultra sound record movement, doppler measures fetal HR, assess fetal well being -2 or more acceleration with 15bpm lasting 15 seconds for 20 min with return to baseline -done after 28 wks

Cervical cap

-worn for 8 hrs but not longer than 48 hrs -cannot be worn during menstrual flow or up to 6 wks

18. A nurse notices frequent artifact on the electrocardiographic monitor for a client whose leads are connected by cable to a console at the bedside. The nurse examines the client to determine the cause. Which of the following items is unlikely to be responsible for the artifact? 1. Frequent movement of the client 2. Tightly secured cable connections 3. Leads applied over hairy areas 4. Leads applied to the limbs

18. 2 Rationale: Motion artifact, or "noise," can be caused by frequent client movement, electrode placement on limbs, and insufficient adhesion to the skin, such as placing electrodes over hairy areas of the skin. Electrode placement over bony prominences also should be avoided. Signal interference also can occur with electrode removal and cable disconnection. Test-Taking Strategy Use the process of elimination, focusing on the subject, artifact and note the strategic word unlikely. Recalling the causes of artifact will direct you to option 2. Review these causes if you had difficulty with this question.

2. A client develops atrial fibrillation with a ventricular rate of 140 beats/min and signs of decreased cardiac output. Which of the following medications should the nurse first anticipate administering? 1. Atropine sulfate 2. Warfarin (Coumadin) 3. Lidocaine (Xylocaine) 4. Metoprolol (Lopressor)

2. 4 Rationale: b-Blockers such as metoprolol slow conduction of impulses through the AV node and decrease the heart rate. In rapid atrial fibrillation, the goal first is to slow the ventricular rate and improve the cardiac output and then attempt to restore normal sinus rhythm. Test-Taking Strategy: Use the process of elimination. Eliminate option 1 because atropine sulfate will further increase the heart rate and will further decrease the cardiac output. Eliminate option 3 because lidocaine is only useful in suppressing ventricular dysrhythmias. Although warfarin (Coumadin) is administered to clients with atrial fibrillation to prevent clots from forming in the atria it will have no effect in decreasing the ventricular rate or restoring normal sinus rhythm. Review these medications if you had difficulty with this question.

692. A client is admitted to an emergency department with chest pain that is being ruled out for myocardial infarction. Vital signs are as follows: at 11 AM, pulse (P), 92 beats/min, respiratory rate (RR), 24 breaths/min, blood pressure (BP), 140/88 mm Hg; 11:15 AM, P, 96 beats/min, RR, 26 breaths/min, BP, 128/82 mm Hg; 11:30 AM, P, 104 beats/min, RR, 28 breaths/min, BP, 104/68 mm Hg; 11:45 AM, P, 118 beats/min, RR, 32 breaths/min, BP, 88/58 mm Hg. The nurse should alert the physician because these changes are most consistent with which of the following complications? 1. Cardiogenic shock 2. Cardiac tamponade 3. Pulmonary embolism 4. Dissecting thoracic aortic aneurysm

692. 1 Rationale: Cardiogenic shock occurs with severe damage (more than 40%) to the left ventricle. Classic signs include hypotension, a rapid pulse that becomes weaker, decreased urine output, and cool, clammy skin. Respiratory rate increases as the body develops metabolic acidosis from shock. Cardiac tamponade is accompanied by distant, muffled heart sounds and prominent neck vessels. Pulmonary embolism presents suddenly with severe dyspnea accompanying the chest pain. Dissecting aortic aneurysms usually are accompanied by back pain. Test-Taking Strategy: Use the process of elimination. Recalling that the early serious complications of myocardial infarction include dysrhythmias, cardiogenic shock, and sudden death will direct you to option 1. No information in the question is associated with options 2, 3, or 4. Review the complications of myocardial infarction if you had difficulty with this question.

724. A nurse is planning to administer hydrochlorothiazide (HydroDIURIL) to a client. The nurse understands that which of the following are concerns related to the administration of this medication? 1. Hypouricemia, hyperkalemia 2. Increased risk of osteoporosis 3. Hypokalemia, hyperglycemia, sulfa allergy 4. Hyperkalemia, hypoglycemia, penicillin allergy

724. 3 Rationale: Thiazide diuretics such as hydrochlorothiazide are sulfa-based medications, and a client with a sulfa allergy is at risk for an allergic reaction. Also, clients are at risk for hypokalemia, hyperglycemia, hypercalcemia, hyperlipidemia, and hyperuricemia. Test-Taking Strategy: Use the process of elimination. Recalling that thiazide diuretics carry a sulfa ring will direct you to option 3. Review the nursing considerations related to administering this medication if you had difficulty with this question.

725. A home health care nurse is visiting a client with elevated triglyceride levels and a serum cholesterol level of 398 mg/dL. The client is taking cholestyramine (Questran). Which of the following statements, if made by the client, indicates the need for further education? 1. "Constipation and bloating might be a problem." 2. "I'll continue to watch my diet and reduce my fats." 3. "Walking a mile each day will help the whole process." 4. "I'll continue my nicotinic acid from the health food store."

725. 4 Rationale: Nicotinic acid, even an over-the-counter form, should be avoided because it may lead to liver abnormalities. All lipid-lowering medications also can cause liver abnormalities, so a combination of nicotinic acid and cholestyramine resin is to be avoided. Constipation and bloating are the two most common side effects. Walking and the reduction of fats in the diet are therapeutic measures to reduce cholesterol and triglyceride levels. Test-Taking Strategy: Use the process of elimination and note the strategic words need for further education. These words indicate a negative event query and ask you to select an option that is an incorrect statement. Remembering that over-the-counter medications should be avoided when a client is taking a prescription medication will direct you to option 4. Review client teaching points related to this medication if you had difficulty with this question.

727. A 66-year-old client complaining of not feeling well is seen in a clinic. The client is taking several medications for the control of heart disease and hypertension. These medications include atenolol (Tenormin), digoxin (Lanoxin), and chlorothiazide (Diuril). A tentative diagnosis of digoxin toxicity is made. Which of the following assessment data would support this diagnosis? 1. Dyspnea, edema, and palpitations 2. Chest pain, hypotension, and paresthesia 3. Double vision, loss of appetite, and nausea 4. Constipation, dry mouth, and sleep disorder

727. 3 Rationale: Double vision, loss of appetite, and nausea are early signs of digoxin toxicity. Additional signs of digoxin toxicity include bradycardia, difficulty reading, other visual alterations such as green and yellow vision or seeing spots or halos, confusion, vomiting, diarrhea, decreased libido, and impotence. Test-Taking Strategy: Use the process of elimination. Recalling that gastrointestinal and visual disturbances occur with digoxin toxicity will direct you to option 3. If you had difficulty with this question, review the signs of digoxin toxicity.

8. A home health care nurse is visiting an older client at home. Furosemide (Lasix) is prescribed for the client and the nurse teaches the client about the medication. Which of the following statements, if made by the client, indicates the need for further teaching? 1. "I will sit up slowly before standing each morning." 2. "I will take my medication every morning with breakfast." 3. "I need to drink lots of coffee and tea to keep myself healthy." 4. "I will call my doctor if my ankles swell or my rings get tight."

8. 3 Rationale: Tea and coffee are stimulants and mild diuretics. These are a poor choice for hydration. Taking the medication at the same time each day improves compliance. Because furosemide is a diuretic, the morning is the best time to take the medication so as not to interrupt sleep. Notification of the health care provider is appropriate if edema is noticed in the hands, feet, or face or if the client is short of breath. Sitting up slowly prevents postural hypotension. Test-Taking Strategy: Use the process of elimination, noting the strategic words need for further teaching. These words indicate a negative event query and ask you to select an option that is an incorrect statement. Recalling that tea and coffee are stimulants and that diuretics potentially can worsen dehydration will direct you to option 3. In addition, coffee and tea are not healthy items to consume. Review client teaching points related to this medication if you had difficulty with this question.

68. A patient comes to the emergency department after a boxing match, and his left eye is swollen almost shut. He has bruises on his face and neck. He says he is worried because he "can't see well" from his left eye. The physician suspects retinal damage. The nurse recognizes that signs of retinal detachment include: A) loss of central vision. B) shadow or diminished vision in one quadrant or one half of the visual field. C) loss of peripheral vision. D) sudden loss of pupillary constriction and accommodation.

B) shadow or diminished vision in one quadrant or one half of the visual field. Page: 316. With retinal detachment, the person has shadows or diminished vision in one quadrant or one half of the visual field. The other responses are not signs of retinal detachment.

The operating room is aware that which of the following patients are at a greater risk related to a surgical procedure?

B)Infants and older adults are at greatest risk from surgery than are children and young or middle-aged adults. Physiologic changes associated with aging increase the surgical risk for older patients.

A patient returning to the floor after orthopedic surgery is complaining of nausea. The nurse is aware that an appropriate intervention is to:

B)Nursing care for a patient with nausea includes avoiding strong smelling foods. Providing oral hygiene, administering prescribed medications (especially if medications ordered are anti-nausea/antiemetics), and avoid the use of a straw.

As the nurse is preparing a patient for surgery, the patient refuses to remove a wedding ring. Which of the following is the most appropriate action by the nurse? A. Note the presence of the ring in the nurse's notes of the chart. B. Insist the patient remove the ring. C. Explain that the hospital will not be responsible for the ring. D. Tape the ring securely to the finger.

C. Explain that the hospital will not be responsible for the ring. It is customary policy to tape a patient's wedding band to the finger and make a notation on the preoperative checklist that the ring is taped in place.

Which of the following should be included in the plan of care for a patient who had spinal anesthesia?

C. Instructing the patient to remain flat in bed for 6 hours. In addition to interventions designed to replace fluids and indirectly replace lost spinal fluid after administration of spinal anesthesia, the patient is instructed to lie flat for 6 to 8 hours. Elevating the head of the bed after spinal anesthesia can precipitate "spinal headache" or nausea related to losses of cerebrospinal fluid or changes in ICP. Elevating the patient's feet or administering oxygen are not necessary interventions unless the patient becomes hypotensive or hypoxic.

A client is admitted with upper GI bleeding. The nurse promotes hemodynamic stability by: 1) Encouraging oral fluid intake 2) Monitoring central venous pressure (CVP) 3) Monitoring laboratory test results and vital signs 4) Giving blood, electrolyte, and fluid replacement.

RATIONALE: 4) to stabilize a client with acute bleeding, normal saline solution or lactated Ringer's solution is given until blood pressure rises and urine output returns to 30ml/hr. A CVP line is inserted to monitor circulatory volume. When shock is severe, plasma expanders are given until typed and crossmatched blood is available. Oral fluid intake is contraindicated with upper GI bleeding. Monitoring vital signs and laboratory values enables the nurse to evaluate the results of treatment, but these measures don't facilitate hemodynamic stabilization.

A nurse is advising a client with a colostomy who reports problems with flatus. Which food should the nurse recommend? 1) Peas 2) Cabbage 3) Broccoli 4) Yogurt

RATIONALE: 4)High fiber food stimulate peristalsis and thus, flatulence. Tell the client to include yogurt in his diet to reduce gas formation. Other helpful foods include crackers and toast. Peas, cabbage, and broccoli are all gas forming foods.

A nurse is caring for a client with liver cirrhosis who has developed ascites and requires paracentesis. Relief of which symptom indicates that the paracentesis was effective?

RATIONALE: ) 2) Dyspnea 2) Ascites (fluid buildup in the abdomen) puts pressure on the diaphragm. Paracentesis (surgical puncture of the abdominal cavity to aspirate fluid) is done to remove fluid from the abdominal cavity and thus reduce pressure on the diaphragm. The goal is to improve the client's breathing. Pruritus, jaundice, and peripheral neuropathy are signs of cirrhosis that aren't relieved or treated by paracentesis.

A client admitted with peritonitis is under a NPO order. The client is complaining of thrist. Which action is the most appropriate for the nurse to take?

RATIONALE: ) Provide frequent mouth care 3) frequent mouth care helps relieve dry mouth. Increasing the I.V. infusion rate does not alleviate the feeling of thirst. Diversion activities aren't specific. Ice chips are a form of liquid and shouldn't be given as long as the client is under an NPO order.

A client with renal insufficiency is admitted with a diagnosis of pneumonia. He&#039;s being treated with IV antibiotics, which can be nephrotoxic. Which laboratory value(s) should be monitored closely?

RATIONALE: 1) BUN and creatinine levels should be monitored closely to detect elevations due to nephrotoxicity. ABG determinations are inappropriate for this situation. Platelets and potassium levels should be monitored according to routine.

A nurse is caring for a client who requires a NG tube for feeding. What should the nurse do immediately after inserting an NG tube for enteral feedings.

RATIONALE: 1) Before starting a feeding, it's essential to ensure that the tube is in the proper location. Aspirating for stomach contents confirms correct placement. Giving the feeding without proper placement puts the client at risk for aspiration. If an X-ray is ordered, it should be done immediately, not in the next 24 hrs. Clamp tube provided no informal about the tube placement.

A client with chronic renal failure is undergoing peritoneal dialysis. A nurse knows that the proper infusion time for the dialysate is:

RATIONALE: 1) Dialysate should be infused quickly. When performing dialysis, the dialysate should be infused over 15 minutes or less. The fluid then dwells in the peritoneum, whre the exchange of fluid and waste products takes place over a period ranging from 30min to several hours.

A client is experiencing an acute episode of ulcerative colitis. What should be the nurse's highest priority?

RATIONALE: 1) Diarrhea caused by an acute episode of ulcerative colitis leads to fluid and electrolyte losses; therefore; fluid and sodium replacement is necessary. There is no need to restrict foods high in potassium; potassium may need to be replaced. If the client is taking steroid medications, the nurse should monitor his glucose levels, but this isn't the highest priority. Noting changes in stool consistency is important, but fluid replacement takes priority.

A nurse is caring for a client with hepatic encephalopathy. The nurse expects which of the following lab values to be abnormal?

RATIONALE: 1) Hepatic encephalopathy is a degenerative disease of the brain caused by advanced liver disease. It develops because of increasing blood ammonia levels. Ammonia levels increase because of proper shunting of blood, causing ammonia to enter the systemic circulation, with carries it to the brain. Excess protein intake, sepsis, excessive accumulation of nitrogenous body wastes (from constipation or GI hemorrhage), and bacterial action on protein and urea also lead to increases in ammonia levels. Amylase levels increase with panceatitis, and inflammation of the pancreas. Hepatic encephalopathy doesn't result from increasing levels of potassium or calcium.

A nurse is preparing to administer an I.M. injection in a client with a neuro/paralytic injury. Which muscle is best to use in this case? 1) Deltoid 2) Dorsal gluteal 3) Vastus lateralis 4) Ventral gluteal

RATIONALE: 1) IM injections should be give in the deltoid muscle in clients with neuropathic/ paralytic and spinal cord injuries. These clients exhibit reduced use of - and consequently reduced blood flow to - muscles in the buttocks (dorsal gluteal and ventral gluteal) and legs (vastus lateralis). Decreased blood flow results in decreased drug absorption.

A nurse is preparing to administer an I.M. injection in a client with a spinal cord injury. Which muscle is best to use in this case?

RATIONALE: 1) IM injections should be give in the deltoid muscle in clients with spinal cord injuries. These clients exhibit reduced use of - and consequently reduced blood flow to - muscles in the buttocks (dorsal gluteal and ventral gluteal) and legs (vastus lateralis). Decreased blood flow results in decreased drug absorption.

A nurse is teaching the family of a client with liver failure. The nurse instructs them to limit which foods in the client diet?

RATIONALE: 1) Meats and beans are high in protein foods. In liver failure, the liver can't metabolize protein adequately, causing protein by-products to build up in the body rather than be excreted. This causes such problems as hepatic encephalophathy (a neurologic syndrome that develops as a result of rising blood ammonia levels). One intervention in liver failure is to limit the client's intake of protein. Although other nutrients, such as fat and carbohydrates, may be regulated, it's mostly important to limit protein.

A nurse is caring for a client with renal calculi. Which drug does the nurse expect the physician to order?

RATIONALE: 1) Opioid analgesics are usually needed to relieve the severe pain of renal calculi. NSAIDs and Salicylates are used for their anti-inflammatory and antipyretic properties and to treat less severe pain. Muscle relaxants are typically used to treat skeletal muscle spasms.

The nurse is assessing a client who reports painful urination during and after voiding. The nurse suspects the client may have a problem with which area of the client&#039;s urinary system?

RATIONALE: 1) Pain during or after voiding indicates a bladder problems, usually infection. Kidney and ureter pain would be in the flank area, and problems or the urethra would cause pain at the external orifice that's commonly felt at the start of voiding.

The nurse is assessing a client who reports painful urination during and after voiding. The nurse suspects the client may have a problem with which area of the client&#039;s urinary system? -

RATIONALE: 1) Pain during or after voiding indicates a bladder problems, usually infection. Kidney and ureter pain would be in the flank area, and problems or the urethra would cause pain at the external orifice that's commonly felt at the start of voiding.

A nurse is preparing to teach a client who has been newly diagnosed with stomach cancer. Which statement should the nurse include in her teaching?

RATIONALE: 1) Stomach pain is typically a late symptom of stomach cancer. 1) Stomach pain is typically a late sign of stomach cancer; outcomes are particularly poor when the cancer reaches that point. Surgery, chemotherapy, and radiation have minimal positive effects on stomach cancer. TPN may increase the growth of cancer cells.

A client is hospitalized and diagnosed with acute hydronephrosis. Which complaint does the nurse expect from this client?

RATIONALE: 1) Sudden, acute colicky pain is a clinical sign of acute hydronephrosis. Hydronephrosis occurs when urine collects in the renal pelvis and calyces due to obstruction or atrophy of the urinary tract. Flank pain most commonly indicates a kidney infection, although it may occur hydronephrosis. Distention and pressure are commonly felt in the pelvis and bladder with lower urinary tract obstructions.

A client is receiving peritoneal dialysis. What should the nurse do when the return fluid is slow to drain?

RATIONALE: 1) Tubing problems are common cause of outflow difficulties. When the return fluid is slow to drain, check the tubing for kinks and ensure all clamps are open. Other measures that may improve drainage include having the client change positions (moving side to side or sitting up in bed), applying gentle pressure over the abdomen, or having a bowel movement. Placing the drainage bag lower (not higher) than the abdomen may also improve drainage.

A client with hiatal hernia reports to the nurse that he has trouble sleeping because of abdominal pain. The nurse should instruct the client to sleep:

RATIONALE: 1) Upper body elevation can reduce the gastric reflux associated with hiatal hernia. Sleeping in a prone or side lying position, or with his lower body slightly elevated, won't help the client.

A client is complaining of severe flank and abdominal pain. A flat plate of the abdomen shows urolithiasis. Which intervention is important?

RATIONALE: 1) Urine should be strained for calculi and sent to the laboratory for analysis. Fluid intake of 3 to 4 qt. 3 to 4 L/day is encouraged to flush the urinary tract and prevent further calculi formation. Ambulation is encouraged to help pass the calculi through gravity. A low-calcium formation of calcium calculi.

A client is admitted to the med-surg. Floor with a diagnosis of acute pancreatitis. His BP 136/76, P 96 bpm, R 22 breaths/min, and T 101F/38.3C. His PMHx reveals hyperlipidemia and alcohol abuse. The physician prescribes an NG tube for the client. The nurse knows the NG tube will:

RATIONALE: 1) an NG tube is inserted into the client's stomach to drain fluids and gas. An NG tube doesn't prevent spasms at the sphincter of Oddi or prevent air from forming in the small and large intestine. A T tube collects bile drainage from the common bile duct.

A client is admitted to the med-surg. Floor with a diagnosis of acute pancreatitis. His BP 136/76, P 96 bpm, R 22 breaths/min, and T 101F/38.3C. His PMHx reveals hyperlipidemia and alcohol abuse. The physician prescribes an NG tube for the client. The nurse knows the NG tube will: 1) Empty the stomach of fluids and gas 2) Prevent spasms at the spincter of Oddi 3) Prevent air from forming in the small and large intestines 4) Remove bile from the gallbladder

RATIONALE: 1) an NG tube is inserted into the client's stomach to drain fluids and gas. An NG tube doesn't prevent spasms at the sphincter of Oddi or prevent air from forming in the small and large intestine. A T tube collects bile drainage from the common bile duct.

A nurse is teaching a male client how to collect a clean-catch midstream urine specimen. What cleaning technique should the nurse include in her teaching?

RATIONALE: 1) before collecting a clean-catch urine specimen, a male client should clean around the urethral meatus in a circular motion and move several inches down the shaft of the penis. When the penis is cleaned from down the shaft to up toward the urethral meatus, organisms from the skin of the penis are dragged toward the meatus.

A nurse is caring for a client with chronic renal failure. The laboratory results indicate hypocalcemia and hyperphosphatemia. When assessing the client, the nurse should be alert for which of the following? SELECT ALL THAT APPLY.

RATIONALE: 1, 2, 6. Hypocalcemia is a calcium deficit that causes irritability and repetitive muscle spasms. S/S of hypocalcemia include Trousseau's sign, cardiac arrhythmias, diarrhea, increased clotting times, anxiety, and irritability. The calcium-phosphorus imbalance leads to brittle bones and pathologic fractures.

A nurse is caring for a client after a renal biopsy. The nurse observes the client for:

RATIONALE: 2) A renal biopsy is obtained through needle insertion into the lower lobe of the kidney, which can need to hemorrhage, so the nurse needs to watch for signs and symptoms of bleeding. After the procedure, the client should remain still for 4 to 12 hours. Changes in mental status (unless the client is bleeding heavily) or blood pressure aren't related to renal biopsy.

A nurse is assessing a client diagnosed with acute pyelonephritis. Which of the following symptoms does the nurse expect to see?

RATIONALE: 2) Costovertebral angle tenderness and chills are symptoms of acute pyelonephritis (inflammation of the kidney and renal pelvis). Jaundice indicates gallbladder or liver obstruction. A burning sensation on urination is a sign of lower urinary tract infection (UTI). Nocturia is associated with a lower UTI or benign prostatic hyperplasia. Polyuria is seen with diabetes mellitus, diabetes insipidus, or the use of diuretics.

A client is diagnosed with cystitis. Client teaching aimed at preventing a recurrence should include which instruction?

RATIONALE: 2) Cotton underwear prevents infection because it allows for air to flow to the perineum. Women should shower instead of taking a tub bath to prevent infection. Feminine hygiene spray can act as an irritant. Cranberry juice helps prevent cystitis because it increases urine acidity; alkaline urine supports bacterial growth, so cranberry juice intake should be increased, not limited.

A nurse is assessing a client who might have a UTI. What statement by the client suggests that a UTI is likely?

RATIONALE: 2) Dysuria (painful urination) is a common symptom of a UTI. Voiding large amounts of urine isn't associated with UTI's; clients with UTI's commonly report frequent voiding of small amounts of urine. A client with a UTI is unlikely to be able to go for hours without urinating because UTI's increase feelings of urgency to void. Urine with a sweet acetone odor is associated with diabetic ketoacidosis. Foul-smelling urine may be a sign of infection.

A nurse is caring for a client in the immediate postoperative period after a prostatectomy. What complication requires priority assessment?

RATIONALE: 2) Immediately after a prostatectomy, , hemorrhage is a potential complication. Pneumonia may occur if the client doesn't turn, cough, and breathe deeply after surgery. Urine retention isn't a problem immediately after surgery because a catheter is in place. Thrombosis may occur later if the client doesn't ambulate.

Discharge instructions for a client treated for acute pyelonephritis should include which statement?

RATIONALE: 2) The client needs to return for follow-up urine cultures because bacteriuria may be present but asymptomatic. Intake of dairy products won't contribute to pyelonephritis. Antibiotics need to be taken for the full course of therapy regardless of the symptoms. Pyelonephritis typically recurs as a relapse or new infection and frequently recurs within 2 weeks of completing therapy.

A nurse is teaching a female client how to prevent the recurrence of urinary tract infection. The nurse should teach her to do which action?

RATIONALE: 2) The nurse should instruct the client to void every 2 to 3 hours to flush bacteria from the urethra and prevent urinary stasis in the bladder. Wiping from front to back (Not back to front) after a bowel movement or urination moves bacteria away from the urethral meatus. Drink 2 to 3 quarts (2 to 3L) of fluid per day helps flush bacteria out of the urinary tract. The nurse should tell the client to avoid bubble baths because they can irritate the urethra, increasing the risk of inflammation and infection.

While undergoing hemodialysis, a client complains of muscle cramps. What intervention is effective in relieving muscle cramps?

RATIONALE: 3) Because muscle cramps can occur when sodium and water are removed too quickly during dialysis, treatment includes administering normal saline or hypertonic normal saline solution. ROM exercises and an infusion of 5% dextrose solution wouldn't reduce muscle cramps. Reducing, not increasing, the rate of dialysis may also alleviate muscle cramps.

A client is admitted with dark urine, fever, and flank pain and is diagnosed with acute glomerulonephritis. What finding is the nurse most likely to find in the client's history?

RATIONALE: 3) Recent sore throat. Typically, acute glomerulonephritis occurs 2 to 3 weeks after a strep throat infection. The Most Common form of acute glomerulonephritis is caused by group A beta-hemolytic streptococcal infection elsewhere in the body. Renal calculi and renal trauma aren't known to cause acute glomerulonephritis. A family history isn't associated with the development of acute glomerulonephritis.

A nurse is teaching a client how to collect a clean catch midstream urine specimen for culture and sensitivity testing. What instructions should a nurse include?

RATIONALE: 3) To collect a clean catch midstream urine specimen; tell the client to void 30ml, stop, and then begin collecting the urine in a sterile urine container. After the sterile container is removed, the client should then finish voiding rest of the urine in the bladder. Discarding the first 30ml of urine flushes away microorganisms that may be around the urinary meatus and distal portions of the urethra. Collecting the first 30ml of urine voided on rising in the morning results in a contaminated specimen. Urine isn't collected for 24hrs for a clean-catch specimen. The first and last voided urine are discarded in a clean catch specimen.

Which client is at greatest risk for developing a UTI?

RATIONALE: 3) Women are more prone to UTI's after menopause. Urinary stasis may develop due to a loss of pelvic muscle tone and prolapse of the bladder or uterus. Reduced estrogen levels lead to reduced levels of vaginal Lactobacilli bacteria, which protect against infection. While chronic diseases, including diabetes mellitus and impaired immunity, increase the risk of UTI, angina, asthma, and fractures don't increase the risk of UTI.

A client is scheduled to undergo a transurethral prostatectomy (TURP) under spinal anesthesia. During the preoperative teaching, the nurse explains to the client that as a result of spinal anesthesia he'll:

RATIONALE: 3) a client who had anesthesia can't move extremities below the level of the anesthesia. This client wouldn't be able to move his legs but could move his arms. Back pain isn't necessarily caused by spinal anesthesia. He wouldn't have difficulty breathing.

A nurse is caring for a client who has undergone surgery to create an ileal conduit. Which expected outcome statement is appropriate for this client?

RATIONALE: 4) A healthy stoma is pink and moist. Sterile gloves aren't necessary when changing the appliance. The stoma isn't to be irrigated. There's no physiologic reason why the client can't engage in sexual relations.

A client has undergone a colostomy for a ruptured diverticulum. The nurse is assessing the client's colostomy stoma 2 days after surgery. Which assessment finding should the nurse report to the physician?

RATIONALE: 4) Brownish black stoma 4) A brownish black stoma color indicates a lack of blood flow to the stoma, and necrosis is likely. A blanched or pale stoma indicates possible decreased blood flow and should be assessed regularly. 2 days postoperatively, the stoma should be edematous and reddish pink.

Which method should be used to collect a specimen for urine culture?

RATIONALE: 4) Catching urine midstream reduces the amount of contamination by microorganisms at the meatus. Voiding in a clean container is done for a random specimen, not a clean-catch specimen for urine culture. When cleaning an uncircumcised male, the foreskin should be retracted and the glands penis should be cleaned to prevent specimen contamination. Voiding in a specimen because the urinal isn't sterile.

Which method should be used to collect a specimen for urine culture? -

RATIONALE: 4) Catching urine midstream reduces the amount of contamination by microorganisms at the meatus. Voiding in a clean container is done for a random specimen, not a clean-catch specimen for urine culture. When cleaning an uncircumcised male, the foreskin should be retracted and the glands penis should be cleaned to prevent specimen contamination. Voiding in a specimen because the urinal isn't sterile.

A client is admitted with upper GI bleeding. The nurse promotes hemodynamic stability by:

RATIONALE: 4) Giving blood, electrolyte, and fluid replacement. 4) to stabilize a client with acute bleeding, normal saline solution or lactated Ringer's solution is given until blood pressure rises and urine output returns to 30ml/hr.

A nurse is conducting discharge teaching for a client with Hepatitis B. Which statement by the client indicates that he understands the teaching?

RATIONALE: 4) Hep.B is characterized by reappearing S/S, including fatigue, nausea, vomiting, bleeding and bruising. Hep.B can recur. Clients who have had Hepatitis are permanently barred from donating blood. Alcohol is metabolized by the liver and should be avoided by the client with Hep.B

A client is receiving pancrelipase (Viokase) for the treatment of chronic pancreatitis. Which observation by the nurse best indicates the treatment.

RATIONALE: 4) Pancrelipase provides a exocrine and pancreatic enzyme necessary for proper protein, fat and carbohydrate digestion. With increased fat digestion and absorption, stools become less frequent and are normal in appearance. Lack of skin breakdown, an improved appetite, and weight loss aren't effects of pancrelipase.

A nurse is writing the teaching plan for a client with cystitis who's receiving phenazopyridine (Pyridium). What instruction should the nurse include?

RATIONALE: 4) Phenazopyridine is taken to relieve dysuria because it provides an analgesic and anesthetic effect on the urinary tract mucosa. The client can stop taking it after the dysuria is relieved. Warn the client that the dye in the drug (azo dye) may temporarily turn the urine red or orange but that isn't cause for calling the physician. Phenazopyridine is usually taken three times per day for 2 days. It isn't taken just before voiding. Antibiotics must be taken for the full course of therapy, even if the burning on urination is relieved.

A nurse is writing the teaching plan for a client with cystitis who's receiving phenazopyridine (Pyridium). What instruction should the nurse include? .

RATIONALE: 4) Phenazopyridine is taken to relieve dysuria because it provides an analgesic and anesthetic effect on the urinary tract mucosa. The client can stop taking it after the dysuria is relieved. Warn the client that the dye in the drug (azo dye) may temporarily turn the urine red or orange but that isn't cause for calling the physician. Phenazopyridine is usually taken three times per day for 2 days. It isn't taken just before voiding. Antibiotics must be taken for the full course of therapy, even if the burning on urination is relieved.

MS: MULTIPLE SCLEROSIS : S/S Complication Crisis

Relapsing/Remitting: Difficulty chewing, speaking, walking. Shakiness, muscle weakness, tinnitus, visual problems, incontinent, Ataxia, Nystagmus, Spasticity, tremors, dysphagia, speech impaired, fatigue Help pts identify triggers: illness, stress

Physical Assessment of Renal System- PALPATION:

Renal System- PALPATION: A landmark useful in locating the kidneys is the costovertebral angle formed by the rib cage and the vertebral column. The normal-size kidney is usually not palpable. If the kidney is palpable, its size, contour, and tenderness should be noted. Kidney enlargement is suggestive of neoplasm or other serious renal pathologic condition. The urinary bladder is normally not palpable unless it is distended with urine.

Physical Assessment of Renal System- PERCUSSION:

Renal System- PERCUSSION: Tenderness in the flank area may be detected by fist percussion (kidney punch). Normally a firm blow in the flank area should not elicit pain. Normally a bladder is not percussible until it contains 150 ml of urine. If the bladder is full, dullness is heard above the symphysis pubis. A distended bladder may be percussed as high as the umbilicus.

The nurse instructs a client who is scheduled for a 24-hour creatinine clearance test. Which of the following statements, if made by the client to the nurse, indicates further teaching is required? 1. "I will eat a high-protein meal before the test begins." 2. "I will use the specimen collection time to catch up on my reading." 3. "I will drink as much fluid as I want before and during the test." 4. "I will save all of my urine during the 24 hours and keep it in the refrigerator."

Strategy: "Further teaching is necessary" indicates incorrect information. 1) CORRECT— high-protein diet before the test may increase creatinine clearance and affect the accuracy of the test 2) appropriate action; avoid strenuous physical activity, will increase creatinine excretion and compromise the accuracy of the test 3) appropriate action 4) appropriate action; bottle should contain a preservative

The nurse receives a call from the emergency management team that 50 victims will be transported to the hospital in 15 minutes by ambulance. Which of the following actions should the nurse take FIRST? 1. Contact the nursing supervisor. 2. Tell the emergency management team they will have to re-route 25 victims. 3. Activate the hospital's disaster plan. 4. Inform the emergency department nurses they must work overtime.

Strategy: "FIRST" indicates priority. 1) CORRECT— nurse must follow chain of command 2) not the nurse's responsibility 3) must notify immediate supervisor about the call; disaster plans are hospital policies that detail how nurses are to perform duties 4) not the responsibility or role of the nurse

The nurse cares for a client in labor. The client's examination reveals that the cervix is 5 cm dilated and 100% effaced and the fetal head is at -1. The membranes rupture and the nurse notes clear fluid. Which of the following actions should the nurse take FIRST? 1. Ambulate the client for 15 minutes and evaluate the fetal heart rate every 30 minutes. 2. Prepare for delivery and notify the care provider. 3. Apply an electronic fetal monitor and start an IV. 4. Encourage the client to void every 1-2 hours and take her temperature every hour.

Strategy: "FIRST" indicates priority. 1) do not ambulate the client; head is too high, may cause cord to prolapse 2) too early to set up for delivery, has approximately 2-3 remaining hours of labor; sterile equipment should be opened for no more than 1 hour 3) no indication that the client is in trouble 4) CORRECT— facilitates descent of the fetal head; temperature evaluation is necessary because of ruptured membranes

The nurse in the psychiatric emergency room assesses 4 clients. Which of the following clients should the nurse see FIRST? 1. A patient was raped 30 minutes ago and expresses feelings of self-blame, anxiety, and worthlessness. 2. A patient indicates an intent to kill himself and says he has access to a gun. 3. A patient had a miscarriage last evening and is experiencing anger and resentment. 4. A patient witnessed a child stabbed to death 2 weeks ago and is experiencing anxiety.

Strategy: "FIRST" indicates priority. 1) need to assess physical needs and examine patient; second patient to see 2) CORRECT— patient is at risk for self-harm; client has intent and a way to carry out threat 3) allow client to verbalize feelings 4) allow client to verbalize feelings

The nurse cares for a 27-year-old female diagnosed with type 1 diabetes. Two days after admission, the client begins complaining of severe nausea. Which of the following actions should the nurse take FIRST? 1. Determine the client's most recent fasting serum glucose level. 2. Perform a comprehensive client assessment. 3. Ask the client if she is pregnant. 4. Administer an antiemetic.

Strategy: "FIRST" indicates priority. 1) no relationship between diabetes and nausea; last glucose reading does not give the nurse information about client's current condition 2) CORRECT— nausea not usually associated with diabetes; assess before implementing 3) nurse is making assumptions based on client's age; should perform a comprehensive assessment 4) assess before implementing

The nurse has just received change-of-shift report. Which of the following patients should the nurse see FIRST? 1. A patient diagnosed with COPD with an PaO 2 of 70%. 2. A patient diagnosed with type 1 diabetes who was just informed her husband is seriously injured. 3. A patient scheduled to leave for the operating room in 30 minutes for a heart valve replacement. 4. A patient 10 hours postop after a right mastectomy complaining of wet sheets under her back.

Strategy: "FIRST" indicates priority. 1) oxygenation considered "normal to good" for patient with COPD; stable patient 2) physical needs take priority 3) requires preop injection; all other preparation should be completed; stable patient 4) CORRECT— may indicate hemorrhage from operative site; unstable patient

After being admitted for management of a cervical spine injury, a client in a rehabilitation center reports a severe headache. Which of the following actions should the nurse take FIRST? 1. Administer an analgesic medication 2. Ask the client to rank the pain from 1 to 10. 3. Ask the client if he is worried about something. 4. Place the client in a sitting position.

Strategy: "FIRST" indicates priority. 1) priority is to decrease blood pressure 2) cervical spine injury and severe headache should clue nurse that client is possibly in imminent danger 3) assess for physical causes before psychosocial causes 4) CORRECT— pounding headache and profuse sweating are indications of autonomic hyperreflexia; place in a sitting position immediately to decrease blood pressure and reduce risk of cerebral hemorrhage

The nurse is completing a neurological assessment on a client who sustained a closed head injury 2 hours ago. Which of the following actions by the nurse would be correct? a. using a tongue blade to check the client's glossopharyngeal nerve b. using the tip of a sterile needle to check the client's facial nerve c. checking the client's patellar deep tendon reflexes after extending the client's leg d. checking the client's biceps reflex after turning the client's palm downward

The correct answer is A. The glossopharyngeal and vagus nerves are checked using a tongue blade. The facial nerve is tested by asking the client to make various facial movements and to identify several different tastes. The client's knee should be flexed before checking the patellar deep tendon reflex. The client's palm should be upward before checking the biceps reflex.

The nurse is caring for a client with severe rheumatoid arthritis. Which of the following actions would be appropriate for the nurse to take while the client is resting in bed? a. Place a rolled washcloth in the client's hands. b. Support the client's forearms on a small pillow. c. Position a folded towel under the client's knees. d. Use a rolled sheet along the client's thighs.

The correct answer is B. A client with rheumatoid arthritis is at risk for increased joint deformities due to incorrect positioning and lack of exercise. The nurse should support the client's forearms on a small pillow while the client is resting in bed to help minimize strain on the shoulder region and to promote blood flow. A rolled washcloth in the client's hands will increase stiffness of the hands.

The nurse is conducting a community-based health fair. The nurse should recognize that which of the following is an appropriate age-based screening? a. breast cancer screening for a 16-year-old female who has anorexia nervosa b. skin cancer screening for a 27-year-old female who is a bridge construction worker c. prostate cancer screening for a 30-year-old male who is Asian d. colorectal cancer screening for a 35-year-old male who is Caucasian

The correct answer is B. Cancer screening is an important role for the nurse in the community. Skin cancer screening is appropriate for a 27-year-old female construction worker since the client works outdoors. Prostate cancer screening begins between 45 and 50 years of age, colorectal cancer screening begins at 50 years of age and breast cancer screening begins at 18 years of age.

The home health nurse is reinforcing teaching with the parents of a child who has a new permanent tracheostomy. Which of the following statements should the nurse make?

The correct answer is B. Changing a tracheostomy tube may cause episodes of coughing. Scheduling the change prior to a meal or at least an hour after a meal may help to minimize vomiting.

The nurse is assessing a client who takes prescribed furosemide (Lasix). Which of the following statements by the client may indicate that the client may be experiencing an adverse effect and would require follow-up?

The correct answer is B. Clients who take Lasix are at risk for anemia and hypokalemia. The report of being tired may indicate an adverse effect of Lasix and would require follow up. A stable weight indicates the medication is effective.

The nurse is teaching a client about prescribed insulin lispro (Humalog). Which of the following statements would be correct for the nurse to make? a. "If you are nauseated, do not take this insulin until you are able to eat a meal." b. "This type of insulin can cause hypoglycemia within an hour." c. "This insulin normally looks cloudy." d. "If you need to change to an insulin pump, a different type of insulin will be required."

The correct answer is B. Humalog insulin peaks within an hour therefore this is the time interval that the client is at increased risk for hypoglycemia. Humalog insulin should be clear and should not be used if it is cloudy.

The nurse is assessing a client who is taking hydrochlorothiazide (HCTZ). Which of the following questions would be essential for the nurse to ask the client? a. "Do bright lights hurt your eyes?" b. "Have you noticed any change in your appetite?" c. "How much water do you drink during the day?" d. "Do you have a dry cough?"

The correct answer is B. Hydrochlorothiazide may cause hypokalemia. Hypokalemia may be indicated by muscle cramps, abdominal distention, anorexia, and constipation. Asking the client about a change in appetite would be essential to help identify potential fluid and electrolyte imbalances. The client's intake of water has lower priority than identifying indications of fluid and electrolyte imbalances.

The home health nurse is assessing a 2-week-old infant who was born at 35 weeks gestation. The parent states, "It seems like I just got home. I have not been able to do anything that I need to do. If I am not taking care of the baby then I am trying to clean the house. I feel overwhelmed." Which of the following responses would be appropriate for the nurse to make initially? a. "Do you have anyone to help?" b. "Let's talk." c. "The hardest days are in the past." d. "Are you sleeping?"

The correct answer is B. The transition to parenthood requires an opportunity to talk about common concerns and joys. Based on the question, the nurse should initially encourage the parent to continue talking by indicating that the nurse is available to talk. Asking questions that can be answered "yes" or "no" block communication if stated initially.

The nurse is reinforcing teaching with a client who has a prescription to begin metoprolol (Toprol XL). The nurse should advise the client that Toprol XL may cause a. urinary retention b. bronchospasms c. photophobia d. paresthesias

The correct answer is B. Toprol XL can cause bronchospasms, urinary frequency and joint pain, and orthostatic hypotension.

The nurse is reinforcing teaching with a client who has a prescription to begin metoprolol (Toprol XL). The nurse should advise the client that Toprol XL may cause a. urinary retention b. bronchospasms c. photophobia d. paresthesias

The correct answer is B. Toprol XL can cause bronchospasms, urinary frequency and joint pain, and orthostatic hypotension.

The nurse is reinforcing teaching with a client who is receiving chemotherapy. The nurse should reinforce that which of the following vitamins may increase the risk for bleeding?

The correct answer is B. Vitamin C can increase the risk for bleeding.

The nurse is conducting a staff education conference regarding end-of-life care of clients who practice a variety of religions. The nurse should state that which of the following practices is commonly associated with the Mormon religion? a. Cremation is required within 24 hours of death. b. The Burial Society washes the client's body after death. c. Last rites may be given when the client develops Cheyne-Stokes respirations. d. A white garment is placed on the client when the skin begins to look mottled.

The correct answer is C. A client from the Church of Jesus Christ of Latter-Day Saints (Mormonism) often receives last rites or communion as part of the death ritual. Cheyne-Stokes respirations are associated with impending death.

The nurse is talking with a client who has small oat cell lung cancer. The client states, "I have decided to stop my radiation therapy and will use an herbal treatment to help cure my cancer." Which of the following responses would be most appropriate for the nurse to make first?

The correct answer is C. A client has a right to make decisions regarding care based on accurate information about treatment alternatives. When a client decides to use alternative therapies rather than conventional therapies, it would be most appropriate for the nurse initially to explore how the client made the decision regarding the alternative treatment. The nurse should not initially ask if the primary health care provider has been advised about the decision since this may block communication.

The nurse is assessing a client who is at 39 weeks gestation and reports the onset of contractions 12 hours ago. The nurse observes a linear bruise on the client's legs and the client reports this occurred when she fell down 2 steps on the front porch. Which of the following questions would be essential for the nurse to ask the client? a. "When did you fall?" b. "Do you have any other injuries?" c. "Are you being mistreated by someone you love?" d. "How many prenatal vitamins are you taking each day?"

The correct answer is C. A client who is pregnant is at risk for domestic violence. Linear bruises on the legs is an injury that is not consistent with the client's report of the injury. It would be essential for the nurse to ask the client about being mistreated by another person.

The nurse is confirming an appointment with a client who is scheduled for her first Papanicolaou smear. Which of the following statements would be appropriate for the nurse to make? a. "This test will help to determine if you have any sexually transmitted diseases." b. "A vaginal irrigation can be done the morning of the test." c. "Do not use any vaginal medications for at least twenty-four hours before the test." d. "An over-the-counter analgesic will help minimize cramping during the test."

The correct answer is C. A client who is scheduled for a Papanicolaou smear is instructed to not douche or use any vaginal medications prior to the test to help ensure reliability of the results.

The nurse is reviewing the record of a client who was just admitted with a traumatic brain injury. Which of the following prescriptions should the nurse clarify? a. Keep the head of the bed elevated 30 degrees. b. Deep breathing exercises without coughing every hour while awake. c. Start IV with D5W 1,000 ml every 8 hours. d. Bisacodyl (Dulcolax) 5 mg, po, q.d.

The correct answer is C. A client with a traumatic brain injury is at risk for increased intracranial pressure. The nurse should verify the prescription for D5W since this is contraindicated if a client is at risk for increased intracranial pressure. A stool softener may be given to help prevent straining with bowel movements which can increase the intracranial pressure.

The nurse has reinforced teaching with a client with bacterial conjunctivitis. Which of the following statements by the client would indicate a correct understanding of the teaching? a. "I need to sleep in a different bed than my spouse until this infection resolves." b. "I can wear my contact lenses after twenty-four hours of treatment." c. "I should clean my eyes using a different section of my washcloth for each eye." d. "I will rest in bed until the redness is gone."

The correct answer is C. A client with bacterial conjunctivitis needs to know methods to control the infection. This includes wiping each eye with a separate section of a washcloth to help prevent cross-contamination. Contact lenses should not be worn until the infection is resolved (typically after 7 days).

Several clients have asked the nurse for a between-meal snack. The nurse should offer the client with

The correct answer is C. A client with hyperthyroidism is expending extra energy with high levels of activity. A liquid nutritional supplement would be appropriate to provide nutrients to help prevent weight loss.

The nurse is planning care for a client who was recently diagnosed with hypothyroidism. Which of the following should the nurse include in the client's plan of care? a. showing the client how to monitor urinary output b. encouraging the client to rest until symptoms resolve c. teaching the client about high fiber foods d. demonstrating foot care to the client

The correct answer is C. A client with hypothyroidism often experiences constipation. The nurse should teach the client about high fiber foods to help regulate bowel movements. Hypothyroidism does not affect urinary output.

The home health nurse is assessing a client who is at 32 weeks gestation and who is being treated for preterm labor. Which of the following is an expected finding? a. low back pain b. yellow-colored vaginal discharge c. yellow discharge from the nipples d. suprapubic pressure

The correct answer is C. A normal finding for a client who is at 32 weeks gestation is yellow nipple discharge, since colostrum may be excreted from the breasts during the last trimester of pregnancy. The other findings may indicate uncontrolled preterm labor, a urinary tract infection and/or a vaginal infection.

The nurse has reinforced teaching with the spouse of a client who will start prescribed donepezil (Aricept). Which of the following statements by the spouse would indicate a correct understanding of the teaching?

The correct answer is C. Aricept is used to stabilize and slow memory loss associated with dementia. This medication should not be discontinued abruptly.

The nurse is assisting while an adult client has a thoracentesis. Which of the following actions would be correct for the nurse to take? a. Ask the client to hold the breath as the needle is inserted. b. Position the client on the side with a pillow support under the waist. c. Check the client's blood pressure while the pleural fluid is extracted. d. Attach a pulse oximetry probe to the client's finger.

The correct answer is D. During a thoracentesis, the client's respiratory status is monitored. Attaching a pulse oximetry probe to the client's finger would be appropriate. The client is not required to hold the breath as the needle is inserted.

The nurse is reinforcing teaching with a client who is at 38 weeks gestation with her first child. The client asks what changes should be reported prior to the next regular obstetrical appointment. The nurse should advise the client to report a. head congestion b. urinary frequency c. yellow-colored nipple discharge d. right upper quadrant abdominal pain

The correct answer is D. During the last few weeks of pregnancy the client continues to monitor for any indication of pregnancy-induced hypertension. The development of right upper quadrant abdominal pain may indicate liver congestion associated with PIH and would require reporting prior to the next scheduled appointment.

The nurse is assessing a 35-year-old client who has scheduled an annual physical examination. The client reports a history of mild pregnancy-induced hypertension (PIH) during two pregnancies and that both infants weighed over 4.0 kg (9 lb). Before the primary health care provider examines the client, the nurse should give priority to a. reviewing the client's record for a recent echocardiography report b. reviewing the client's record for a recent electrocardiography (EKG) report c. obtaining a urine specimen from the client d. checking the client's capillary glucose level

The correct answer is D. Risk factors for type 2 diabetes mellitus include giving birth to an infant that is large-for-gestational age. Based on the client's history and age, the nurse should give priority to checking the client's capillary glucose level. The client's history does not indicate a risk for urinary problems therefore obtaining a urine specimen is not a priority.

The charge nurse has completed the shift assignments for nursing assistants. Which of the following statements by the charge nurse would provide the best directions to help ensure that the task is completed as assigned? a. "The client with right-sided hemiparesis must be assisted to walk in the hallway." b. "Make sure that the client with Alzheimer's disease is assisted to the bathroom several times this shift." c. "Snacks are in the refrigerator for clients who are receiving insulin." d. "The client with multiple sclerosis needs to get out of bed for each meal."

The correct answer is D. The charge nurse must provide specific, detailed information to nursing assistants to help ensure that the client care assignment is completed appropriately and within the required time frame. Informing the nursing assistant that the client needs to get out of bed for each meal is specific. The charge nurse needs to be more specific regarding when snacks should be given to clients who are receiving insulin.

The nurse is talking with the parent of a child who just died. The parent states, "Just get out of here. No one can bring my child back to life." Which of the following should the nurse do first? a. Tell the parent anger is a normal response. b. Ask if the parent would like to talk. c. Slowly exit the room. d. Remain silent.

The correct answer is D. The death of a loved one can cause a variety of emotions. Anger is a common emotion and the nurse should initially remain silent to see if the parent will continue talking. Exiting the room is not a therapeutic initial response.

The nurse is reviewing the laboratory test results of a client who had a transurethral resection of the prostate (TURP) 2 days ago. Which of the following laboratory test results is within the normal range? a. white blood cell (WBC) count, 3,000/cu mm b. red blood cell (RBC) count, 4.0 million/cu mm c. hemoglobin (Hgb), 20.0 grams/dl d. hematocrit (HCT), 46%

The correct answer is D. The normal hematocrit for a male is 42 to 52 %; WBC 5,000 to10,000/cu mm; RBC, 4.7 to 6.1 million/cu mm; and Hgb 14 to 18 grams/dl.

The nurse is completing oral care on a client who is comatose. Which of the following actions would be appropriate for the nurse to take? a. Brush the client's tongue with a medium bristle toothbrush. b. Use one moist gauze pad to gently wipe both cheeks inside of the client's mouth. c. Irrigate the client's mouth with normal saline after flossing the client's teeth. d. Turn the client onto the side with the head slightly lower than the shoulders.

The correct answer is D. The nurse completes oral care on a client who is comatose by turning the client onto one side with the head slightly lower than the shoulders. This helps to prevent aspiration. The client's tongue, roof of the mouth and cheeks are each cleaned with a separate gauze pad to minimize cross-infection. Oral irrigation is not performed due to the risk of aspiration.

The nurse has been advised that a client is being admitted with severe diarrhea. There are no private rooms available on the unit. It would be most appropriate for the nurse to assign the client to share the room with a client

The correct answer is D. The nurse is required to assist in determining the correct room assignments for clients based on an understanding of infection control procedures. A client with severe diarrhea requires contact precautions as does the client with acute pancreatitis. It would be most appropriate for these two clients to share a room. Clients with open wounds and who are receiving TPN or PEG feedings would not be appropriate roommates for the client with severe diarrhea based on the choices given.

The nurse is reinforcing teaching with a client who will start warfarin sodium (Coumadin) due to the new diagnosis of atrial fibrillation. Which of the following statements would be appropriate for the nurse to make? a. "Multiple vitamin supplements are recommended while you take Coumadin." b. "Orange juice should be limited since this contains a high amount of vitamin K." c. "Avoid using antibacterial soap on your skin." d. "You should get a shower chair to use in your bathtub."

The correct answer is D. The nurse must be able to identify a client's risk for injury in the home. A client with atrial fibrillation may develop dizziness during activities. In addition, the client is at risk for potentially life-threatening bleeding due to the therapeutic effects of Coumadin. It would be appropriate for the nurse to recommend that the client get a shower chair to use in the bathtub.

The nurse is assessing a 78-year-old client with prostate cancer. The client lives with an adult child in a ground-level apartment. Which of the following statements by the client should increase the nurse's suspicion regarding elder abuse and would require follow-up?

The correct answer is D. The nurse must be observant for indications of elder abuse. A 78-year-old client is at risk for harm by others and the report of friends not being allowed to visit requires additional information to determine if the client is being isolated. Capillary fragility causes easy bruising on the hands of the elderly.

The nurse is working in an immunization clinic. The nurse should inform parents that the varicella vaccine is recommended at a. 2 months of age b. 4 months of age c. 9 months of age d. 12 months of age

The correct answer is D. The varicella vaccine is recommended at 12 months of age.

The nurse is admitting an adult client with a wound infection. The client has a prescription for intravenous vancomycin. The nurse should ask the nursing assistant to closely monitor the client's a. voice tone and quality b. gait c. daily weight d. intake and output

The correct answer is D. Vancomycin can cause elevated blood urea nitrogen (BUN) and creatinine levels as well as diarrhea. The nurse should ask the nursing assistant to closely monitor the client's intake and output.

4. Provides a minimal standard of knowledge for a registered nurse in practice

The examination for registered nurse licensure is exactly the same in every state in the United States. The examination: 1. Guarantees safe nursing care for all patients 2. Ensures standard nursing care for all patients 3. Ensures that honest and ethical care is provided 4. Provides a minimal standard of knowledge for a registered nurse in practice

3. Ask the family to stay with the patient if possible. 4. Inform the family of the risks associated with side rail use. 6. Discuss alternatives with the family that are appropriate for this patient.

The family of a patient who is confused and ambulatory insists that all four side rails be up when the patient is alone. What is the best action to take in this situation? (Select all that apply.) 1. Contact the nursing supervisor. 2. Restrict the family's visiting privileges. 3. Ask the family to stay with the patient if possible. 4. Inform the family of the risks associated with side rail use. 5. Thank the family for being conscientious and put the four rails up. 6. Discuss alternatives with the family that are appropriate for this patient.

Expected side effects of magnesium sulfate include...

hypersomnolence (sleeping, have hot flashes, and lethargy)

Blood and body fluids of a client with HIV/AIDS should be cleaned up with...

hypochlorite solution (1 part bleach and 10 parts water).

Newborns of diabetic mothers should be assessed immediately for...

hypoglycemia by heel stick, lateral aspect; should also be tested for hypokalemia and acidosis

Signs of toxicity of MgSO4

hyporeflexia, oliguria, decreased respirations

Signs of TSS include...

hypotension, fever, dizziness, and a rash; treatment is antibiotics

Heel- stick blood sample

hypothyroidsm deficiency of thyroid hormone

Assessment of CNIX

identify sweet, sour, and salty tastes on posterior area of tongue

Junctional Rhythm

if SA nodes fire < 40-60 bpm junctional pacer occur. No P-wave and sinus brady

when do you give rhogam

if other is rh negative and father is rh positive rhogam given in the 28th wk

ataxia

impaired ability to coordinate movement

Clinical manifestations commonly observed in the child with autism include...

impaired social interaction, impaired communication, problems adapting to new situations, impaired attention span, speech difficulties or delays in speech, absence of babbling by one year of age and absence of speech by two years of age, inability to respond to social or emotional cues in a normal manner, presence of rigid, obsessive behaviors, head banging, twirling in circles, self-biting, abnormal and exaggerated responses to sensory stimuli

MS

impaires nerve impulse conduction, which is related to loss of myelin sheath

apraxia

impairment of the ability to perform purposeful acts or to use objects properly

MG

impairs transmission of impulses due to lack of acetylcholine

Fertilization:

impregnation of ovum and sperm

pyloric stenosis

in adults, narrowing or obstruction of the pyloric sphincter caused by scarring from healing ulcers (peptic ulcers). in infants, obstruction caused by hypertrophy and hyperplasia of pylorus

An early deceleration mirrors...

in depth and length the contraction.

Normal fetal response is increase...

in heart rate 15 beats per minute

Diverticular disease: diagnosis

infection, inflammation or obstruction of diverticula (sacs or pouches in the intestinal wall) cause the pt to be symptomatic

Diverticulitis:

inflamed matter trapped in the diverticula (out pouches)

Gastritis:

inflammation of stomach that may be actue or chronic.

Cholecystitis

inflammation of the gallbladder

Crohn's disease: diagnosis

inflammatory condition of any area of large or small intestine, usually ileum and ascending colon

ulcerative colitis: diagnosis

inflammatory condition of the colon characterized by eroded areas of the mucous membrane and tissues beneath it. Ulcerative colitis is a type of inflammatory bowel disease (IBD) that affects the large intestine (colon) and rectum

Complication of autonomic hyperreflexia, or dysreflexia with SCI clients usually occurs after...

the spinal shock has resolved.

weber test

the stem of the tuning fork is held to the middle of the forehead and the clients hearing is assessed on both ears.

gravida

the total number of pregnancies regardless of duration

what symptom is different for older pts suffering from UTI?

they are more likely to present with confusion and not abd pain.

Be careful with coughing exercises because...

they increase ICP.

Effacement

thinning of cervix

Absence (Petite-Mal) Seizure

usually occurs during childhood and decreases with age. Sudden LOC w/ little or no tonic clonic movement, occurs without warning, and appears a few hours after arising or when pt is quiet. Signs: vacant facial expression w/ eyes focused straight ahead.

Anterolateral

v4-v6

right ventricular

v4r, v5r, v6r

threatened abortion

vaginal bleeding cramping, soft uterus, cervix closed

Menstruation:

vaginal discharge of blood and fragments of the endometrium; occurs in response to drop in progesterone and estrogen

Persons with paranoid personality disorder spend a great deal of time and energy...

validating their suspicions. They do not have fixed delusions or hallucinations as those with paranoid schizophrenia do.

QRS complex & interval:

ventricular depolorization

T wave:

ventricular repol continues

ST segment:

ventricular repolorization

Cesarean Delivery: classifications

vertical incision: more blood loss, rapid delivery low-segment transverse incision: less blood loss, VBAC possible

Preeclampsia symptoms...

vertigo, headache, or edema of the hands and face

Treatment of chlamydia trachomatis during pregnancy

vibramycin 100 mg twice daily. Azithromycin one gram and treat the client and partner

homonymous hemianopsia

visual field cut both eyes

hemianopsia

visual field cut; defective vision or blindness 1/2 visual field

ulcers: Postoperative

vitamin b12 required for life.

Prior to pregnancy, the client should be encouraged to increase the intake of foods high in...

vitamins such as B9 (folic acid).

Prior to a vaginal ultrasound, the client should...

void.

Meds tx for A-fib

warfarin( 4x/wk), aspirin 325mg, digoxin, verapamil,beta blockers,diltiazem, amiodarone, procainamide, atropine if ventricular rate is low

P wave

wave of depolorization SA & atria

The client with catatonic schizophrenia exhibits...

waxy flexibility or stupor.

flaccid

weak, soft and flabby muscles lacking normal tone

-Paresis:

weakness or incomplete loss of muscle function

Meningitis: plan/implementation

-IV Antibiotic (penicillin, cephalosporin, vancomycin, -Droplet precaution Hib vaccine for infants

Gastritis: implementation

-NPO slowly progressing to bland diet -Antacids often relieve pain

HHNKS: Plan

-NS or .45% NaCl -regular insulin -K labs

Strabismus: Implementation

-Nonsurgical intervention begins no later than age 6 -Orthoptic exercise to strengthen eye muscles

Phenylketonuria: Nursing consideration

-Specially prepared milk formula [Lofenalac] -low protein diet ( no meat, dairy product, eggs, NUtrasweet)

Cataracts: Diagnosis

-partial or total opacity of the normally transparent crystalline lens -caused: drugs-steroid therapy

Retinopathy of prematurity (ROP): plan/implementation

-premature infant <36 weeks or <2000g have eye exam -Maintain Sa02 (95-100) Administer Vitamin E ( thought to affect tissue response to o2)

Chorionic villus sampling (CVS)

-sample of fetal placental tissue -test for fetal karyotype, sickle-cell anemia, PKU, down syndrome -done at 8-12 wks -full bladder required

Detached Retina: Diagnose

-separation of the retina from the choroid

QRS complex time

.06-.10 sec

CK-MB

0-3

Oxytocin (pitocin) administration

0.5-1.0 mU/min; increase 1-2mU/min at intervals 30-60 min. max 20 mU/min

Therapeutic range lithium

0.5-1.5 mEq/Liter

QT interval

0.55 sec

APGAR: Reflexes

0=absent, flaccid; 1=some flexion of extremities; 2=good reflexes, active movement

APGAR: Cry reflex, irritability

0=absent; 1=grimace; 2=cry

Cystic fibrosisL Assessment and nursing considerations

1) absence of pancreatic enzyme leads to malabsorption of fat weight loss, infection and lung dz leads to increase need for calories and protein. 2) cotazym pancreas(pancreatic enzyme replacement) before meals

Signs of IICP: (basic ideas)

1) changes in LOC 2) changes in Vital Signs 3) changes in Eyes 4) decreased motor function 5) HA

5 stages of grieving.

1) denial, 2anger, 3bargaining, 4depression, 5acceptance

3 types of phlebitis:

1) mechanical (cannula causes issue) 2) chemical (solution is irritating) 3) bacterial (microorganism introduced to vein)

What are the four phases of the Nurse/client relationship

1) preorientation 2) orientation 3) working 4) termination

1) Photophobia 2) Phonophobia

1) related to Light 2) related to Sound

Teaching for prilosec and prevacid should include:

1) take 30-60 mins before meals 2) highly protein bound. Stronger than Pepsid (H2 antagonist)

Hepatobiliary dz: Assessment and Nursing consideration

1)Decreased bile leads to fat malabsorption 2) low fat high protein vitamins

5 S/S of ICP:

1. Visual changes and headaches. 2. Change in LOC and blown pupil. 3. widened pulse pressure, increased BP, bradycardia, and hyperflexia. 4. Vomiting 5. papilledema (choked eye disc)

10. A client with pulmonary edema has been on diuretic therapy. The client has an order for additional furosemide (Lasix) in the amount of 40 mg intravenous push. Knowing that the client will also be started on digoxin (Lanoxin), the nurse should review which laboratory result? 1. Sodium level 2. Digoxin level 3. Creatinine level 4. Potassium level

10. 4 Rationale: The serum potassium level is measured in the client receiving digoxin and furosemide. Heightened digoxin effect leading to digoxin toxicity can occur in the client with hypokalemia. Hypokalemia also predisposes the client to ventricular dysrhythmias. Test-Taking Strategy Use the process of elimination. Eliminate option 2 because the client will just be beginning digoxin therapy. No data indicate the presence of renal insufficiency; therefore, eliminate option 3. Furosemide therapy can cause hyponatremia and hypokalemia, but remember that the risk of hypokalemia has more severe consequences in this situation. Review the nursing considerations related to administering furosemide if you had difficulty with this question.

11. A home health nurse instructs a client about the use of a nitrate patch. The nurse tells the client that which of the following will prevent client tolerance to nitrates? 1. "Do not remove the patches." 2. "Have a 12-hour 'no-nitrate' time." 3. "Have a 24-hour 'no-nitrate' time." 4. "Keep nitrates on 24 hours, then off 24 hours."

11. 2 Rationale: To help prevent tolerance, clients need a 12-hour "no-nitrate" time, sometimes referred to as a pharmacological vacation away from the medication. Options 1, 3, and 4 are incorrect. Test-Taking Strategy: Use the process of elimination, focusing on the subject, preventing tolerance to nitrates. This subject and knowledge regarding administering this medication will direct you to option 2. Review the administration of nitrate patches if you had difficulty with this question.

Bilirubin level that may require phototherapy

12 mg/dl

Fetal heart tones can be heard with a Doppler ultrasound at approximately...

12 weeks

12. A nurse assesses the sternotomy incision of a client on the third day after cardiac surgery. The incision shows some slight "puffiness" along the edges and is nonreddened, with no apparent drainage. Temperature is 99° F orally. The white blood cell count is 7500 cells/mm3. How should the nurse interpret these findings? 1. Incision is slightly edematous but shows no active signs of infection. 2. Incision shows early signs of infection, although the temperature is nearly normal. 3. Incision shows early signs of infection, supported by an elevated white blood cell count. 4. Incision shows no sign of infection, although the white blood cell count is elevated.

12. 1 Rationale: Sternotomy incision sites are assessed for signs and symptoms of infection, such as redness, swelling, induration, and drainage. Elevated temperature and white blood cell count after 3 to 4 days postoperatively usually indicate infection. Test-Taking Strategy Use the process of elimination. Eliminate options 3 and 4 because the white blood cell count is within normal range. From the remaining options, focus on the data in the question. A nonreddened incision with no apparent drainage indicates no signs of infection. Review the signs of infection if you had difficulty with this question.

12. A client is admitted to a medical unit with nausea and bradycardia. The family hands a nurse a small white envelope labeled "heart pill." The envelope is sent to the pharmacy and it is found to be digoxin (Lanoxin). A family member states, "That doctor doesn't know how to take care of my family." Which of the following statements would convey a therapeutic response by the nurse? 1. "Don't worry about this. I'll take care of everything." 2. "You are concerned your loved one receives the best care." 3. "You're right! I've never seen a doctor put pills in an envelope." 4. "I think you're wrong. That physician has been in practice over 30 years."

12. 2 Rationale: This is a therapeutic, nonjudgmental response. The statement reflects the family's concern but remains nonjudgmental. Option 1 dismisses the family's concerns and disempowers the family. Option 3 creates doubt about the physician's practice without actually knowing the circumstances. Option 4 is argumentative and nontherapeutic. Test-Taking Strategy: Use therapeutic communication techniques. Reflection of the client's or family's concerns is the most therapeutic. Review these techniques if you had difficulty with this question.

accelerations

15 bpm rise above baseline followed by a return; usually in response to fetal movements or contractions; indicate fetal well-being

Amniocentesis can be performed as early as...

16 weeks gestation.

Growth height generally ceases by age...

17 in girls and age 18-20 in boys.

Head circumference at birth (13-14 inches) increases to an average of...

17 inches by six months and 18 inches at one year of age.

If Macrobid causes pulmonary side effects such as SOB, cough, etc when will they subside?

2-3 days after stopping

Average height increases about...

2-3 inches per year; 3 year old 37 inches; 4 year old 40 inches; 5 year old 43 inches

20. A client has frequent bursts of ventricular tachycardia on the cardiac monitor. Why should the nurse be most concerned about with this dysrhythmia? 1. It can develop into ventricular fibrillation at any time. 2. It is almost impossible to convert to a normal rhythm. 3. It is uncomfortable for the client, giving a sense of impending doom. 4. It produces a high cardiac output that quickly leads to cerebral and myocardial ischemia.

20. 1 Rationale: Ventricular tachycardia is a life-threatening dysrhythmia that results from an irritable ectopic focus that takes over as the pacemaker for the heart. The low cardiac output that results can lead quickly to cerebral and myocardial ischemia. Clients frequently experience a feeling of impending doom. Ventricular tachycardia is treated with antidysrhythmic medications, cardioversion (client awake), or defibrillation (loss of consciousness). Ventricular tachycardia can deteriorate into ventricular fibrillation at any time. Test-Taking Strategy Use the process of elimination and note the strategic words most concerned. Option 2 is incorrect and is eliminated first. From the remaining options, focusing on the strategic words will direct you to option 1 because this option identifies the life-threatening condition. Review the concerns associated with ventricular tachycardia if you had difficulty with this question.

23. A client is on enalapril (Vasotec) for the treatment of hypertension. The nurse teaches the client that he should seek emergent care if he experiences which adverse effect? 1. Nausea 2. Insomnia 3. Dry cough 4. Swelling of the tongue

23. 4 Rationale: Enalapril (Vasotec) is an angiotensin-converting enzyme inhibitor. Angioedema is an adverse effect. Swelling of the tongue and lips can result in airway occlusion. Nausea, insomnia, and a cough can occur as side (not adverse) effects of the medication. Test-Taking Strategy: Note the strategic word adverse. Use the ABCs—airway, breathing, and circulation—to direct you to option 4. Review the adverse effects of this medication if you had difficulty with this question.

3. A client with myocardial infarction has been transferred from a coronary care unit to a general medical unit with cardiac monitoring via telemetry. A nurse plans to allow for which of the following client activities? 1. Strict bed rest for 24 hours after transfer 2. Bathroom privileges and self-care activities 3. Ad lib activities because the client is monitored 4. Unsupervised hallway ambulation with distances under 200 feet

3. 2 Rationale: On transfer from the coronary care unit, the client is allowed self-care activities and bathroom privileges. Supervised ambulation in the hall for brief distances is encouraged, with distances gradually increased (50, 100, 200 feet). Test-Taking Strategy Use the process of elimination. Eliminate options 3 and 4 first because they are excessive, given that the client has just been transferred from the coronary care unit. Option 1 is not appropriate because the client would be doing less activity than in the coronary care unit before transfer. Review activity prescriptions for the client with a myocardial infarction if you had difficulty with this question.

35. A client diagnosed with thrombophlebitis 1 day ago suddenly complains of chest pain and shortness of breath and is visibly anxious. The nurse should immediately assess the client for signs and symptoms of which of the following? 1. Pneumonia 2. Pulmonary edema 3. Pulmonary embolism 4. Myocardial infarction

35. 3 Rationale: Pulmonary embolism is a life-threatening complication of deep vein thrombosis and thrombophlebitis. Chest pain is the most common symptom, which is sudden in onset, and may be aggravated by breathing. Other signs and symptoms include dyspnea, cough, diaphoresis, and apprehension. Test-Taking Strategy Focus on the client's diagnosis to answer the question. Recalling the complications related to thrombophlebitis will direct you to option 3. Review these complications and the associated signs and symptoms if you had difficulty with this question.

36. A client seeks treatment in a physician's office for unsightly varicose veins, and sclerotherapy is recommended. Before leaving the examining room, the client says to the nurse, "Can you tell me again how this sclerotherapy is done?" Which of the following statements would reflect accurate teaching by the nurse? 1. "The varicosity is surgically removed." 2. "The vein is tied off at the upper end to prevent stasis from occurring." 3. "The vein is tied off at the lower end to prevent stasis from occurring." 4. "An agent is injected into the vein to damage the vein wall and close the vein off."

36. 4 Rationale: Sclerotherapy is the injection of a sclerosing agent into a varicosity. The agent damages the vessel and causes aseptic thrombosis, which results in vein closure. With no blood flow through the vessel, there is no distention. The surgical procedure for varicose veins is vein ligation and stripping. This procedure involves tying off the varicose vein and large tributaries and then removing the vein with hook and wires via multiple small incisions in the leg. Test-Taking Strategy Use the process of elimination and note the name of the procedure, sclerotherapy. A vessel that is sclerosed is blocked. This will direct you to option 4. Review this procedure if you had difficulty with this question.

40. A client is scheduled for a cardiac catheterization using a radiopaque dye. Which of the following assessments is most critical before the procedure? 1. Intake and output 2. Height and weight 3. Allergy to iodine or shellfish 4. Baseline peripheral pulse rates

40. 3 Rationale: A cardiac catheterization requires an informed consent because it involves injection of a radiopaque dye into the blood vessel. The risk of allergic reaction and possible anaphylaxis is a concern and the presence of allergies must be assessed before the procedure. Although options 1, 2, and 4 are accurate, they are not the most critical preprocedure assessments. Test-Taking Strategy Use the process of elimination and note the strategic words most critical. Recalling the concern related to allergy to the dye and the risk of anaphylaxis makes option 3 correct. Review preprocedure interventions for a cardiac catheterization if you had difficulty with this question.

42. A nurse is evaluating the condition of a client after pericardiocentesis performed to treat cardiac tamponade. Which of the following observations would indicate that the procedure was unsuccessful? 1. Rising blood pressure 2. Clearly audible heart sounds 3. Client expressions of relief 4. Rising central venous pressure

42. 4 Rationale: Following pericardiocentesis, a rise in blood pressure and a fall in central venous pressure are expected. The client usually expresses immediate relief. Heart sounds are no longer muffled or distant. Test-Taking Strategy Use the process of elimination and note the strategic word unsuccessful. Successful therapy is measured by the disappearance of the original signs and symptoms of cardiac tamponade. Therefore, look for the option that identifies a sign consistent with continued tamponade. Review signs of cardiac tamponade and the expected effects of pericardiocentesis if you had difficulty with this question.

sitting up with support is at what age

6 months

Normal BUN

6-20

Diaphragm should be removed after...

8 hours.

9. A nurse is caring for a client receiving a heparin intravenous (IV) infusion. The nurse anticipates that which laboratory study will be prescribed to monitor the therapeutic effect of heparin? 1. Hematocrit 2. Hemoglobin 3. Prothrombin time 4. Activated partial thromboplastin time

9. 4 Rationale: The prothrombin time will assess for the therapeutic effect of warfarin sodium (Coumadin), and the activated partial thromboplastin time (aPTT) will assess the therapeutic effect of heparin. Hematocrit and hemoglobin values assess red blood cell concentrations. Baseline assessment, including an aPTT value, should be completed, as well as ongoing daily aPTT values while the client is taking heparin. Heparin doses are determined based on the result of the aPTT. Test-Taking Strategy: Use the process of elimination. Eliminate options 1 and 2 because they are comparative or alike and are unrelated to heparin therapy. From the remaining options, recall the relationship between the prothrombin time and warfarin and the aPTT and heparin. Review care of a client on heparin infusion if you had difficulty with this question.

LDH

<115

Sinus Brady

<60 bpm

1 small box

=0.4 sec

5 large boxes (or 25 small boxes)

=1 sec total

hydramnios

> 2000 ml of amniotic fluid

normal serum bilirubin level

>/=13-15 mg/dl

1. "Are you thinking of suicide?"

A 34-year old man who is anxious, tearful, and tired from caring for his three young children tells you that he feels depressed and doesn't see how he can go on much longer. Your best response would be which of the following? 1. "Are you thinking of suicide?" 2. "You've been doing a good job raising your children. You can do it!" 3. "Is there someone who can help you?" 4. "You have so much to live for."

1. Cognitive

A nurse is teaching a 27-year old gentleman how to adjust his insulin dosages based on his blood sugar results. What type of learning is this? 1. Cognitive 2. Affective 3. Adaptation 4. Psychomotor

5. The nurse is interviewing a patient who has a hearing impairment. What techniques would be most beneficial in communicating with this patient?

A) Determine the communication method he prefers.

82. The nurse is percussing over the lungs of a patient with pneumonia. The nurse knows that percussion over an area of atelectasis in the lungs would reveal: A) dullness. B) tympany. C) resonance. D) hyperresonance.

A) dullness. Pages: 424-425. A dull percussion note signals an abnormal density in the lungs, as with pneumonia, pleural effusion, atelectasis, or tumor.

50. The nurse is performing a nutritional assessment on an 80-year-old patient. The nurse knows that physiological changes that directly affect the nutritional status of the elderly include:

A) slowed gastrointestinal motility.

The nurse is admitting a patient to the same day surgery unit. The patient tells the nurse that he was so nervous he had to take kava last evening to help him sleep. Which of the following nursing actions would be most appropriate?

A. Inform the anesthesiologist of the patient's ingestion of kava. Kava may prolong the effects of certain anesthetics. Thus the anesthesiologist needs to be informed of recent ingestion of this herbal supplement.

Select all that apply. A nurse is caring for a surgical patient in the preoperative area. The nurse obtains the patient's informed consent for the surgical procedure. Which statements are true regarding informed consent?

A. Informed consent must be signed while the patient is free from mind-altering medications. B. Informed consent must be witnessed. An informed consent must be signed while the patient is free from mind-altering medications and must be witnessed after it has been determined that the patient has received all of the necessary information needed to make an informed decision. An informed consent may be withdrawn at any time before the procedure and must be signed by patients age 18 and older. A parent or guardian's signature is required for minors. The informed consent may be obtained by the physician or the nurse and is not required in the event of a life-threatening emergency.

A client with coronary artery disease complains of substernal chest pain. After assessing the client's heart rate and blood pressure, a nurse administers nitroglycerin, 0.4 mg, sublingually. After 5 minutes, the client states, "My chest still hurts." Select the appropriate actions that the nurse should take. Select all that apply. 1. Call a Code Blue. 2. Contact the physician. 3. Contact the client's family. 4. Assess the client's pain level. 5. Check the client's blood pressure. 6. Administer a second nitroglycerin, 0.4 mg, sublingually.

Answer: 4, 5, 6 Rationale: The usual guidelines for administering nitroglycerin tablets for chest pain include administering one tablet every 5 minutes PRN for chest pain, for a total dose of three tablets. If the client does not obtain relief after taking a third dose of nitroglycerin, the physician is notified. Because the client is still complaining of chest pain, the nurse would administer a second nitroglycerin tablet. The nurse would assess the client's pain level and check the client's blood pressure before administering each nitroglycerin dose. There are no data in the question that indicate the need to call a Code Blue. Additionally, it is not necessary to contact the client's family unless the client has requested this. Test-Taking Strategy: Focus on the data in the question. Use the steps of the nursing process to determine that assessing the client's pain level and checking the client's blood pressure are appropriate actions. Next, recalling the usual guidelines for administering nitroglycerin tablets will assist in determining that an appropriate action is to administer a second nitroglycerin tablet, 0.4 mg, sublingually. Review care of the client with chest pain and the guidelines for the administration of nitroglycerin if you had difficulty with this question.

MS: MULTIPLE SCLEROSIS : disease discription

Autoimmune dz, progressive degeneration of CNS; the body attacks its own tissues, destroying nerve fibers of the brain and spinal cord (disseminated demyelination) No Cure

31. During a mental status assessment, which question by the nurse would best assess a person's judgment?

B) "Tell me about what you plan to do once you are discharged from the hospital."

40. The nurse is examining an infant and prepares to elicit the Moro reflex at which time during the examination?

B) At the end of the examination

63. During an assessment of an infant, the nurse notes that the fontanels are depressed and sunken. The nurse suspects which condition?

B) Dehydration

Which of the following may be left in place when a patient is sent to the operating room?

B. Hearing aid If a patient is wearing a hearing aid, the perioperative nurse should be notified. Leaving the hearing aid in place enhances communication in the operating room. The nurse should make certain to record that the appliance is in place. Wigs, engagement rings, and dentures are not necessary items to facilitate quality patient care in the operating room.

The nurse would be alerted to the occurrence of malignant hyperthermia when the patient demonstrates:

B. Muscle rigidity Malignant hyperthermia is a metabolic disease characterized by hyperthermia with rigidity of skeletal muscles occurring secondary to exposure to certain anesthetic agents in susceptible patients. Hypoxemia, hypercarbia, and dysrhythmias may also be seen with this disorder.

If a 77-year-old patient who is NPO after surgery has dry oral mucous membranes, which of the following is the most appropriate nursing intervention?

B. Perform oral hygiene frequently. Frequent oral hygiene will help alleviate discomfort for a patient who is NPO. IV fluid rate is prescribed by the physician. Petroleum is always inappropriate intraorally. Oral fluid intake is contraindicated in a patient who is NPO.

Which of the following preoperative assessment findings should be reported to a surgeon for preoperative treatment?

B. Serum potassium level of 3 mEq/L. Electrolyte imbalances increase operative risk. Preoperative laboratory results should be checked to see whether they are within the normal range. The normal potassium level is 3.5 to 5.0 mEq/L. A low serum potassium level puts the patient at risk for cardiac dysrhythmias. A serum sodium level of 140 mEq/L is considered a normal value. An Hb concentration of 13.5 mg/dl is considered a normal value by most laboratory standards and does not interfere with operative decisions. A partial thromboplastin time of 25 seconds is a normal value and conducive to proceeding with a surgical procedure.

Preeclampsia mild (last 10 wk of gestation)

BP 140/90 or increase 30/15 mmhg 1+ 2+ proteinuria begins past 20 wks slight edema weight gain

Preeclampsia severe

BP 150-160/100-110mmhg 3+4+ proteinuria epigastric pain extreme irritability pulmonary edema Hyperreflexia HELLP (hemolysis, elevated liver enzyme low platelet)

Ejection Fraction

Blood that is ejected

While taking Cipro or Levaquin if you experience dizziness, light sensitivity or light-headedness what might this indicate:

CNS toxicity

Conjunctivitis (pink eye)

Caused: bacteria, virus, allergies Warm compress, topical antibiotics,Hydrocortisone ophthalmic ointment

2. During an interview, the nurse states, "You mentioned shortness of breath. Tell me more about that." Which verbal skill is used with this statement?

D) Open-ended question The nurse should use it to begin the interview, to introduce a new section of questions, and whenever the person introduces a new topic.

57. The nurse is assessing for clubbing of the fingernails and would expect to find:

D) an angle of the nail base of 180 degrees or greater with a nail base that feels spongy.

70. An ophthalmic examination reveals papilledema. The nurse is aware that this finding indicates: A) retinal detachment. B) diabetic retinopathy. C) acute-angle glaucoma. D) increased intracranial pressure.

D) increased intracranial pressure. Pages: 319-320. Papilledema, or choked disk, is a serious sign of increased intracranial pressure, which is caused by a space-occupying mass such as a brain tumor or hematoma. This pressure causes venous stasis in the globe, showing redness, congestion, and elevation of the optic disc, blurred margins, hemorrhages, and absent venous pulsations. Papilledema is not associated with the conditions in the other responses.

Cataracts: Implementation

Extracapsular extraction- cut through the anterior capsule to expose the opaque lens Intracapsular extration- removal of entire lens and capsule; places pt at risk for renal detachment -sleep on unaffected side to decrease pressure

Chemical: Acids, Cleansers, insecticides

Eye irrigation with water 15-20 min

Clients with tongs or pin require care with...

H2O2, normal saline, and an antibiotic cream

s/s of pyleonephritis:

HA, increased BP, facial/periorbital edema, lethargic, low grade temp, wt gain (edema), and protein-, hema-, olgi-, dys- uria

Irritable Bowel Syndrome and Diverticulosis: dietary intervention

High fiber: dietary consideration to prevent constipation/ diarrhea, to normalize bowel water content, and add bulk.

A nurse is caring for a client who requires total parenteral nutrition (TPN). The client asks the nurse why he's getting TPN. The nurse best response is:

RATIONALE: 2) TPN is given I.V. to provide all the nutrients the client needs; it provides more than just fluids and electrolytes. TPN solutions typically provide glucose, amino acids, trace elements and vitamins, and fats. TPN is neither a tube feeding nor a liquid dietary supplement.

A nurse is advising a client with a colostomy who reports problems with flatus. Which food should the nurse recommend?

RATIONALE: 4)High fiber food stimulate peristalsis and thus, flatulence. Tell the client to include yogurt in his diet to reduce gas formation. Other helpful foods include crackers and toast. Peas, cabbage, and broccoli are all gas forming foods.

A nurse is caring for a client after a renal biopsy. The nurse observes the client for:

RATIONALE: 2) A renal biopsy is obtained through needle insertion into the lower lobe of the kidney, which can need to hemorrhage, so the nurse needs to watch for signs and symptoms of bleeding. After the procedure, the client should remain still for 4 to 12 hours.

A nurse is instructing a client with renal calculi about recommended daily fluid consumption. The nurse would be most helpful by telling the client to drink approximately:

RATIONALE: 3) A client with renal calculi should drink 3L (12 cups) of fluid per day.

During a health history, which statement by a client indicates a risk of renal calculi?

RATIONALE: 4) Renal calculi are commonly composed of calcium. Diets high in calcium may predispose a person to renal calculi. Milk and milk products are high in calcium. Cola soft drinks don't contain ingredients that would increase the risk of renal calculi. Jogging and increased stress aren't considered risk factors for renal calculi formation.

A client admitted with renal failure is in the oliguric phase. A nurse expects the client's 24-hr urine output to be less than what amount?

RATIONALE: Oliguria is defined as a diminished urine output of less than 400ml/24hrs

Cardiac Output Calculation

SV x HR

The nurse supervises care for a patient admitted to the psychiatric unit with a diagnosis of bipolar disorder: manic phase. A student nurse plans activities for the patient. The nurse should intervene if the student nurse chooses which of the following activities? 1. Volleyball. 2. Painting. 3. Walking. 4. Dancing.

Strategy: "Nurse should intervene" indicates an incorrect action. 1) CORRECT— avoid competitive games because they increase agitation; assign to a single room away from activity; keep noise level low and lighting soft 2) appropriate activity; will not provoke or over-stimulate client 3) appropriate activity; activity that uses large movements until acute mania subsides 4) appropriate activity; provides structure and safety in the milieu

The nurse is teaching a 50-year-old client about the scheduled screening colonoscopy. Which of the following statements would be correct for the nurse to make? a. "Before the test begins, an intravenous catheter will be placed into your arm." b. "You will be able to return home after the test is completed and you are able to urinate." c. "A full liquid diet is permitted the night before the test." d. "The test will be rescheduled if you have any rectal itching."

The correct answer is A. A client who is scheduled for a colonoscopy is advised that a intravenous catheter will be inserted before the test so that medications can be given as needed prior to and during the procedure. The client is discharged when fully awake and when the vital signs are stable.

The nurse is distributing between-meal snacks to several clients. Which of the following snacks would be best for the nurse to offer each client? a. fresh vegetable sticks to a client with a white blood cell (WBC) count of 11,300/cu mm b. diced watermelon to a client with a hemoglobin (Hgb) level of 8.0 mEq/L c. graham crackers for a client with stomatitis d. plain gelatin to a client with end-stage renal disease (ESRD)

The correct answer is A. A client with an elevated WBC can eat fresh vegetables as a between-meal snack. Fresh fruit and vegetables would not be given if the client was immunosuppressed. A client with ESRD should receive high-quality proteins and calories to help maintain nitrogen balance.

The nurse is planning care for a client with bacterial pneumonia. The client is receiving oxygen therapy as prescribed via nasal cannula and a prescribed parenteral anti-infective medication. Which of the following nursing diagnoses should be included in the client's plan of care? a. Activity intolerance. b. Altered peripheral tissue perfusion. c. Sensory-perceptual alterations. d. Decreased cardiac output.

The correct answer is A. A client with bacterial pneumonia has nursing diagnoses that include ineffective airway clearance, ineffective breathing pattern, activity intolerance and pain.

The nurse is preparing to administer a unit of packed red blood cells to a client as prescribed. The nurse should ask the client to immediately report the onset of a. a dry mouth b. back pain c. yawning d. urinary urgency

The correct answer is B. A client who is receiving blood products is at risk for an acute transfusion reaction, such as a hemolytic reaction, that may be life-threatening. The client should immediately report the onset of chills, headache, backache, dyspnea or chest pain.

The nurse is reviewing the record of a client who has prescriptions to begin the following medications. Which of the following medications should the nurse verify since the client reports a previous allergic reaction to sulfonamides? a. carbidopa/levodopa (Sinemet) b. carvedilol (Coreg) c. glimepiride (Amaryl) d. metronidazole (Flagyl)

The correct answer is C. A client with an allergic reaction to sulfonamides may have a cross-sensitivity reaction to Amaryl.

The nurse is admitting a client with a metastatic brain tumor who just had a craniotomy. Which of the following actions should the nurse take first? a. Assess the client's level of consciousness. b. Assess the client's vital signs. c. Determine if the client has an advance directive. d. Determine if the client has family members in the waiting room.

The correct answer is C. A client with cancer who had a craniotomy is at risk for multiple potentially life-threatening postoperative complications. The nurse should first determine if the client has an advance directive so that appropriate actions can be planned if an emergency occurs during the shift. An advance directive provides information about how staff members should respond if a life-threatening emergency develops. The client's level of consciousness has lower priority based on the information presented in the question.

The nurse is preparing to administer a prescribed medication to a client with severe Alzheimer's disease. The client is not wearing an identification bracelet. Which of the following actions would be most appropriate for the nurse to take?

The correct answer is C. A client with severe Alzheimer's disease is unable to provide reliable information. Based on the 5-rights of medication administration, if the client is not wearing an identification bracelet, the nurse should return the medication to the medication cart until a new bracelet is obtained.

The nurse is talking with a client who will begin chemotherapy for leukemia. Which of the following statements would be correct for the nurse to make? a. "Your urine will have high levels of the chemotherapy particles after the first treatment so make sure that you flush your toilet at least twice after urinating." b. "You should avoid contact with young children until the chemotherapy treatments are completed." c. "Try taking the prescribed antiemetic at regular time intervals rather than when nausea occurs." d. "Wear loose fitting clothing to help prevent skin irritation associated with chemotherapy."

The correct answer is C. Chemotherapy can cause severe nausea and vomiting. A client should try taking the prescribed p.r.n. antiemetic at regular time intervals to help prevent nausea and vomiting. Radiation therapy may cause skin reactions so the client should wear loose fitting clothing to help minimize irritation.

The home health nurse is checking a client with chronic obstructive pulmonary disease (COPD). The client is using oxygen around-the-clock. Which of the following information would indicate to the nurse that the client's condition is worsening?

The correct answer is C. The client with COPD is at risk for pneumonia and pneumothorax. Painful breathing is an indication that the client's condition has worsened and requires follow up. The answers are consistent with this stage of illness that requires continuous oxygen. Pursed-lip breathing is characteristic for a client with COPD.

The nurse has completed discharge teaching with a 76-year-old client with dysphagia who will be discharged with percutaneous endoscopic gastrostomy (PEG) tube feedings. Which of the following statements by the client would indicate that the client is at increased risk for injury after discharge and would require intervention by the nurse? a. "I will coil the PEG tube under my clothing when I am gardening outdoors." b. "I plan to clamp the PEG tube when I resume my swimming classes next month." c. "I will keep the PEG tube feeding bags on top of my refrigerator." d. "I can fill the PEG tube feeding bag with enough formula to last eight hours."

The correct answer is C. The nurse must be able to identify a client's risk for injury in the home. A client who is 76 years old is at risk for falls and should be discouraged from reaching overhead to get frequently needed objects. Storing the PEG tube feeding bags on top of the refrigerator is a risk that requires intervention by the nurse. The feeding bags can be filled with enough formula to last 8 hours without risk to the client.

The nurse is assessing a client with heart failure who is taking prescribed hydrochlorothiazide (HydroDIURIL). The client states, "I do not know where I am. I don't feel good." The nurse should immediately review the results of which of the following laboratory tests? a. erythrocyte sedimentation rate (ESR) b. serum protein c. uric acid d. blood urea nitrogen (BUN)

The correct answer is D. A client who is on HydroDIURIL is at risk for renal dysfunction. The onset of confusion and malaise may indicate that the client has renal dysfunction. The nurse should immediately check the results of the client's BUN.

The nurse is participating in a disaster drill. The nurse is advised that a chemical spill occurred on the third floor of a large business. The nurse should understand that during the disaster drill priority should be given to the client with a. a bone protruding from the foot b. blood oozing from a hand c. red sclerae d. a dry cough

The correct answer is D. Maslow's hierarchy is used to establish priorities during a disaster. A chemical spill releases caustic fumes into the air. A client with a dry cough may have chemical burns in the airway and should receive priority. Blood oozing from a hand has lower priority than profuse bleeding.

The nurse is teaching the parent of a 4-hour-old, full term infant about feeding the infant. The parent has decided to only formula-feed the infant. Which of the following statements would be correct for the nurse to make? a. "Burp your infant often during the feeding." b. "The baby should sleep on the abdomen after eating." c. "Unopened cans of formula should be stored in the refrigerator." d. "You can prefill bottles with about two ounces of formula."

The correct answer is D. Principles of newborn feeding include filling the bottles with enough formula for each feeding, typically 2 to 3 ounces the first week of life; burping the infant after approximately 1/2 ounce of formula, placing the infant on the back after eating and storing unopened cans of formula in a cabinet.

The nurse is caring for a client who is receiving a dose of prescribed vancocin (Vancomycin) via IV piggyback. The nurse observes that the client's vital signs remain within the client's baseline and the client has facial flushing. Which of the following actions would be appropriate for the nurse to take? a. Obtain the emergency cart. b. Stop the infusion. c. Slow the rate of flow. d. Inform the client that this is normal.

The correct answer is D. Red man's syndrome may occur with vancomycin and this causes a flushed appearance to the face or neck. The client should be reassured that this is a common, non life-threatening side effect.

1. Place a bed alarm device on the bed.

The nursing assessment of an 80-year old patient who demonstrates some confusion but no anxiety reveals that the patient is a fall risk because she continues to get out of bed without help despite frequent reminders. The initial nursing intervention to prevent falls for this patient is to: 1. Place a bed alarm device on the bed. 2. Place the patient in a belt restraint. 3. Provide one-on-one observation of the patient. 4. Apply wrist restraints.

4. Risk for falls

The nursing assessment on a 78-year old woman reveals shuffling gait, decreased balance, and instability. On the basis of the patients data, which of the following nurses diagnoses indicates an understanding of the assessment findings? 1. Activity intolerance 2. Impaired bed mobility 3. Acute pain 4. Risk for falls

3. Develop topics for discussion that require problem solving

The school nurse is about to teach a freshman-level high school health class about nutrition. What is the best instructional approach to ensure that the students meet the learning outcomes? 1. Provide information using a lecture 2. Use simple words to promote understanding 3. Develop topics for discussion that require problem solving 4. Complete an extensive literature search focusing on eating disorders

3. Consistent

The surgical unit has initiated the use of a pain-rating scale to assess patients' pain severity during their postoperative recovery. The registered nurse (RN) looks at the pain flow sheet to see the pain scores recorded for a patient over the last 24 hours. Use of the pain scale is an example of which intellectual standard? 1. Deep 2. Relevant 3. Consistent 4. Significant

4. Anticipated when to make choices without others' assistance.

Which of the following is unique to the commitment level of critical thinking? 1. Weighs benefits and risks when making a decision. 2. Analyzes and examine choices more independently. 3. Concrete thinking. 4. Anticipated when to make choices without others' assistance.

2. Nurse practitioner 3. Certified clinical nurse specialist

Which of the following nursing roles may have prescriptive authority in their practice? (Select all that apply.) 1. Critical care nurse 2. Nurse practitioner 3. Certified clinical nurse specialist 4. Charge nurse

4. Problem solving.

While assessing a patient, the nurse observes that the patient's intravenous (IV) line is not infusing at the ordered rate. The nurse assesses the patient for pain at the IV site, checks the flow regulator on the tubing, looks to see if the patient is lying on the tubing, checks the point of connection between the tubing and the IV catheter, and then checks the condition of the site where the intravenous catheter enters the patient's skin. After the nurse readjusts the flow rate, the infusion begins at the correct rate. This is an example of: 1. Inference. 2. Diagnostic reasoning. 3. Competency. 4. Problem solving.

3. Takes antihypertensive and diuretics 4. History of recent fall 5. Neglect, spatial and perceptual abilities, impulsive 6. Requires assistance with activity, unsteady gait 7. IV line, urinary catheter

You are admitting Mr. Jones, a 64-year old patient who had a right hemisphere stroke and a recent fall. The wife stated that he has a history of high blood pressure, which is controlled by an antihypertensive and a diuretic. Currently he exhibits left sided neglect and problems with spatial and perceptual abilities and is impulsive. He has moderate left-sided weakness that requires the assistance of two and the use of a gait belt to transfer to a chair. He currently has an intravenous (IV) line and a urinary catheter in place. What factors increase his fall risk at this time? (Select all that apply.) 1. Smokes a pack a day 2. Used a cane to walk at home 3. Takes antihypertensive and diuretics 4. History of recent fall 5. Neglect, spatial and perceptual abilities, impulsive 6. Requires assistance with activity, unsteady gait 7. IV line, urinary catheter

Gonorrhea presents as...

a yellow discharge.

Treatment of sedative-hypnotic overdose...

activated charcoal and gastric lavage, mechanical ventilation and dialysis as needed

Symptoms of a client with a spinal cord injury include...

acute respiratory failure; compromised respiratory function; loss of bowel and bladder tone; loss of sweating and vasomotor tone; marked reduction in BP due to loss of peripheral vascular resistance; sensory and motor paralysis below the level of injury

Myasthenic Crisis

an acute exacerbation of disease caused by inadequate amount of meds, infection fatigue or stress.

Reglan (metoclopramide) is a (an) _______ that is most effective when given prophylactically (at least 30 min prior to chemo/radiation)

an antiemetic that blocks CTZ(no response to vomitus stimulus) and stimulates gastric emptying (downward into GI)

Diagnostic criteria for preeclampsia are...

an elevated BP, facial edema, and proteinuria

Transition to the second stage of labor is marked by

an increased urge to push, an increase in bloody show, grunting, gaping of the anus, involuntary defecation, thrashing about, loss of control over breathing techniques, and nausea and vomiting

Dizziness, circumoral numbness, and slurred speech indicate

anesthesia overdose

Sumatriptan succinate (Imitrex) is contraindicated in clients with...

angina.

Signs of withdrawal related to opiate abuse...

anorexia, irritability, runny nose, nausea, bone pain, chills

What classification of meds decreases bowel motility?

anti-diarrheals

What category is simethicone (Gas-X, Mylicon) and how does it work?

anti-flatulent that reduces surface tension of air bubbles which helps alleviate the pain associated with gas

It is contraindicated for a client with a hemorrhagic stroke to receive...

anticoagulants and thrombolytics.

Treatment of panic disorder and OCD usually respond to treatment with...

antidepressant medication.

Zofran (ondansetron):

antiemetic; antiserotonergic (blocks serotonin receptor in GI tract and CTZ)

Meniere's Disease: drug therapy

antihistamine: benadryl, epinephrine Antiemetics: Compazine Antivertigo medication: Antivert, valium, reglan, tigan antivert Diuretics: dyazide

Treatment of genital herpes (HSV2) during pregnancy

antiviral medications such as acyclovir

Treatment of condylomata acuminate (caused by HPV) during pregnancy

antiviral medications, Podophyllin (not recommended for pregnant clients because it can cause birth defects), antineoplastics such as 5-FU, imiquimod cream (Aldara0), trichloroacetic acid (TCA), gardasil

if you are teaching a GI pt to push fluids and that his stool may be white for up to 72 hrs after test, what test has this pt just undergone?

barium swallow - detects structural abnormalities

early deceleration

before peak of contraction, most often uniform mirror image of contraction on tracing; associated with head compression in second stage with pushing.

Physiologic jaundice is a...

benign condition resulting from an immature liver; does not become evident until 48-72 hours; does not cause brain damage

Jaundice that exists longer that two weeks of age accompannied by elevations in direct (conjugated) bilirubin points to the possiblity of...

biliary atresia

Lochia serosa

blood and serous fluid, noted on third or fourth postpartum day

Stroke volume

blood ejected from left ventricles with each contraction

Cephalohematoma

blood that does not cross the suture line on the baby's scalp

Acrocyanosis

bluish discoloration of hands and feet of newborn

Chadwick's sign

bluish discoloration of the vagina and cervix

Symptoms of PTSD include...

blunted emotions, feelings of detachment, flashbacks, moral guilt, numbing of responsiveness, survivor guilt; increased arousal, anxiety, restlessness, irritability, sleep disturbances, problems with memory and concentration; have problems with depression and self-destructive behaviors

flat

bone muscles

tubal ligation

both fallopian tube are ligated and severed; prevent the passage of eggs

breast milk jaundice (late onset)

caused by factor in breast milk; treated by dc breast feeding

Treatment for PTSD and phobic disorder usually benefits from...

cognitive behavioral therapy and desensitization.

Characteristic of Hematuria:

cola-colored urine from blood

Presbyopia

commonly after age 35. The result is a slow decrease in the ability of the eye to focus on nearby objects.

Management of severe preeclampsia

complete bed rest, moderation in sodium, magnesium sulfate (lowers BP)

CNS impairment

confusion, delirium, Safety measures

Symptoms of stage 2 (8-12 hours after last use)

confusion, disorientation, hallucinations, hyperactivity, and gross tremors

Symptoms of serotonin syndrome include...

confusion, hypomania, agitation, hyperthermia, hyperreflexia, tremors, rigidity, and GI upset. The medication should be discontinued immediately.

Complications of maternal diabetes on the newborn include...

congenital heart defects such as patent ductus arteriosus, polyhydramnios, premature delivery, respiratory distress syndrome, hypoglycemia

Signs of opiate abuse...

consticted pupils, decreased RR, decreased BP, euphoria, impaired attention span, impaired judgment

Postpartum depression: assessment

decrease in estrogen and progesterone may precipitate "baby blues"

Symptoms of spinal shock include...

decreased HR, flaccid paralysis, no reflex activity or perspiration below level of lesion, low BP

What neurological symptoms of hypokalemia would be observed in a patient?

decreased reflexes

Splitting

defense mechanism used by those with borderline personality disorder

Signs of overdose related to opiate abuse...

dilated pupils, respiratory depression, seizures, coma, death

Signs of hallucinogen abuse (LSD)...

dilated pupils, tachycardia, diaphoresis, irregular eye movement, grandiosity, hallucinations

Sympathetic Effects:

dilates pupils, inhibit saliva, accelerates heartbeat, dilates bronchi, inhibits peristalsis, conversion of glycogen to glucose, inhibits bladder contractions

Treatment of hydatidiform mole

dilation and curettage

Symptoms of Guillain Barre include...

dimished or absent tendon reflexes; low-grade fever; muscle weakness that gradually moves up the arms, trunk, and face; autonomic dysfunction (hypertension, hypotension, tachycardia, bradycardia); numbness, pain, and tingling in extremities

Treatment of late decelerations includes...

discontinue pitocin, apply oxygen, and change mother's position (left-side lying)

Huntington's Disease:

disorder passed down through families in which certain nerve cells in the brain waste away, or degenerate.

Conductive hearing loss

disorder that affect auditor canal, eardrum, ossicles. not producing sound to get to organ of corti

Types of Schizophrenia:

disorganized, catatonic, paranoid, undifferentiated, residual

Borderline personality disorder utilizes...

dissociation as a means of coping with stress, which results in splitting (inability of seeing self and others as having both good and bad qualitites

s/s of speed shock

dizziness, facial flushing, HA, chest tightness, hypotension, irregular pulse, progression of shock

Diplopia

double vision

diplopia

double vision

The birth weight (average 7-9 pounds)...

doubles by approximately six months of age and triples by one year of age.

Cluster B personality disorders includes...

dramatic, erratic, emotional behavior

At 6-8 years, the child has boundless...

energy; swimming, skating, biking, dancing, sports; ability to read, tell time, use simple math; child want to be with peers; participation in group activities

Biophosphonates: side effects

esophagitis, arthralgia (pain in joints), N. D.

Signs of stimulant abuse (amphetamine, nonamphetamine, cocaine, caffeine, alcohol)...

euphoria, grandiosity, dilated pupils, tachycardia, elevated BP, nausea and vomiting, paranoia, hallucinations, violent outbursts

Dependent personality disorder is characterized by an...

extreme need to be taken care of by someone else; see themselves as inferior and incompetent, frequently become involved in abusive relationships

Lip repair is performed earlier than palate repair to...

facilitate feeding and promote parental-infant bonding

Esophageal atresia (EA)

failure of the esophagus to develop a continuous passage

During infancy, symptoms of hirschusprung disease (congenital aganglionic megacolon)

failure to gain weight, constipation, abdominal distention, episodes of vomiting

Symptoms of hirschusprung disease (congenital aganglionic megacolon)

failure to pass meconium with the first 24-48 hours, refusal to feed, abdominal distention, and intestinal obstruction

decelerations

fall below baseline lasting 15 seconds or more followed by return

hallucinations

false sensory perception in the absence of an external stimulus; may be auditory, visual, olfactory or tactile

Steatorrhea

fat in feces with strong-foul smell, frothy urine and stool

Signs of overdose related to cannabis abuse...

fatigue, paranoia, psychosis

Shigella: source

fecal contamination: affects pediatric population

Decelerations are associated with...

fetal hypoxemia.

resonance

found with air-filled tissue

A pt that presents w/UTI s/s may be experiencing what:

frequency, urgency, suprapubic pain, dysuria, hematuria, fever, confusion in older adult

Characteristics of contractions: frequency, duration, intensity

frequency: beginning of one crontaction to the beginning of the next or the peak of one contraction to another, duration: beginning to end (> 90 bad) intensity:the strength of a contraction at acme

Stage 2 labor

from complete cervical dilation to delivery of the baby

Stage 3 labor

from delivery of the baby to delivery of the placenta

Stage 4 labor

from delivery of the placenta until completion of the recovery period

The best diagnostic test for diabetes in pregnancy is the...

glucose tolerance test.

Thayer-Martin culture (TMC) is done for...

gonorrhea.

The lateral diameter of chest circumference becomes...

greater than the anteroposterior diameter.

In the toddler the chest circumference is...

greater than the head circumference.

Suture line of the lip is cleaned using...

half-strength hydrogen peroxide.

Assessment of CNXI

have client shrug shoulders against resistance; neck strength checked by having client turn head against resistance

Assessment of CNXII

have client stick out tongue, observe for deviations or tremors, check strength of tongue movement as it presses against tongue blade

Following supratentorial crainiotomy, position the client with...

head of bed elevated 30 degrees.

Infant reflexes occur in a cephalocaudal--proximodistal development

head-to-toe and center-to-peripheral sequence

interventions for UTI may include

heating pad for discomfort and Pyridium for spasms

HELLP syndrome

hemolysis, elevated liver enzymes, and low platelets

Treatment of DIC

heparin to treat clotting, Amicar to stabilize bleeding, electrolyte replacement, blood transfusions, oxygen

A lesion that is painful is most likely a...

herpetic lesion.

Intestinal obstruction:assessment

high-pitched sounds above area of obstruction, decreased or absent below area of obstruction. Abdominal pain and distention; obstipation (no gas or stool)

Crohn's disease & ulcerative colitis: diet

high: protein & calories Low: fiber & fat TPN to rest bowel

Cluster B disorders include...

histrionic, narcissistic, antisocial, and borderline personality disorder.

Tx for menopause

hormonal replacement therapy, isoflavanoids, vitamin B and E for hot flashes, Calcium (osteoporosis) vaginal cream (dyspareunia)

Hypercortisolism causes:

hypergylcemia, osteoporosis, buffalo hump, moon face, obesity.

chloasma

increased pigmentation on the face; blotchy brown areas on the forehead and cheeks

Pancreatitis

inflammatory disease of the pancreas that may result in autodigestion of the pancreas by its own enzymes

Ipecac should not be used if____

ingested substance is sedating (may become too lethargic to prevent aspiration) or caustic (cause more oral and esophageal damage).

Biophosphonates: Action

inhibits one reabsorption

Nerve blocks: Subarachnoid (spinal) anesthesia

injected through third, fourth, or fifth lumbar interspace into subarachnoid space; relieve uterine pain; observe for hypotension and bradycardia; will probably be unable to assist with pushing during third stage labor

assessment of the abdomen

inspection, auscultation, percussion and palpation

order of assessments

inspection, palpation, purcussion, and auscultation

Gonorrhea

instill prophylactic medication into baby's eyes after delivery

How does insulin work?

insulin lowers blood glucose by facilitating uptake and the use of glucose by muscle and fat cells; decrease the release of glucose from the liver.

Phobic disorders are expressed as...

intense, irrational fears of some object, situation, or activity. The client recognizes the fear is irrational, but the phobia persists.

Intestinal obstruction: plan

intestinal decompression: insert of plastic or rubber tube into the stomach or intestine via nose or mouth. remove fluid or air mb removed.

Celiac Disease: Assessment

intestinal malabsorption that leads to malnutrition

variability

irregular fluctuations in the baseline of FHR of 2 cycles per minutes or greater. > 30 min indicator of fetal distress

Signs of withdrawal related to cannabis abuse...

irritability, restlessness, insomnia, tremors, sweating, GI upset

vaginal hysterectomy

is for prolapsed uterus

Macrobid may be used for tx of UTI, why?

it acts as a disinfectant in the urinary tract but is not effective outside of the UT

At 12-18 months, the toddler can...

kick a ball forward, walk up steps, build a tower of two or four cubes, use a spoon, drink from a cup, push and pull toys, remove clothes, and scribble with crayons. Toddler imitates housework, points to at least one named body part, and knows 10 words.

Alzheimer's

loss of brain cells from cerebral cortex and creation of neurofibrillary tangles, decreased acetylcholine

Tonic Clonic (Grand-Mal) Seizure:

loss of consciousness and falling to floor. Signs: aura, cries, loss of consciousness, fall, tonic clonic movements, incontinence, cyanosis, excessive salvation, tongue or cheek biting. Posticatal period: need 1-2hr for sleep after.

ataxia

loss of coordinated movements

Paralysis:

loss of movement

Preterm infants exhibit...

low birth weight (less than 1500 grams), lack of lanugo, absence of sucking pads, undescended testes

hematocrit level

male: 42-52 female/child: 35-47

Hemoglobin level:

males 13-18 females 12-16

Symptoms of neuroleptic malignant syndrome include...

malignant hyperthermia or extreme temperature elevation (sometimes as high as 107 degrees F)

Hemoglobin

man 13-18 g/dl women 12-16 g/dl child (3-12 yo) 11-12.5 gm/dl

Magnesium sulfate

medication used to treat preeclampsia; help to decrease uterine contractions; monitor for magnesium toxicity

Symptoms of Alzheimer's include...

memory loss, disorientation, emotional distress, agitation, and sundowning

Erythrocyte sedimentation rate (ESR)

men: 1-15 women: 1-20

lead poisoning should drink what type of liquid

milk binds to lead to decrease absorption

A GCS greater than or equal to 13 indicates...

minor coma.

Probable signs of pregnancy are...

more conclusive than presumptive signs, but are still not definitive.

Guillain barre syndrome: assessment

motor losses symmetrical, usually beginning in lower extremities, then extend upward to include trunk, upper extremities, cranial nerves, and vasomotor funciton; deep tendon reflexes disappear.

akathisia

motor restlessness

Dissociative identity disorder (DID) is previously referred to as...

multiple personality disorder

Symptoms of Parkinson's include...

muscle stiffness, non-intentional tremor, dysphagia, autonomic dysfunction, bradykinesia

Lithium levels greater than 1.5 mEq/Liter can produce signs of toxicity such as...

muscle weakness, confusion, ataxia, seizures, cardio-respiratory changes, and multiple organ failure

Addison's Disease: assessment

muscle weakness, lethargy, hypoglycemic, orthostatic hypotension, weight loss, Bronzing of skin, Low sodium, glucose. High potassium and hematocrit

In diagnosing bowel obstruction what must you do before barium enema?

must see xray - may not be administered if peritonitis is present

Ginger is a herbal remedy thought to be effective in treating _______

nausea and vomiting (n/v) particularly w/ chemo & radiation tx and hyperemesis in pregnancy

Signs of withdrawal related to sedative-hypnotic abuse...

nausea and vomiting, tachycardia, diaphoresis, tremors, and seizures

Nerve block: Spinal/epidural narcotics side effects

nausea, itching, urinary retention, respiratory distress

linea nigra

on the abdomen dark line from the umbilicus to the symphis pubis

Surgical repair of a cleft lip is generally performed between...

one and three months of age.

TEF oral feedings are begun about...

one week postoperatively and are started with sterile water follwed by small, frequent feedings of formula.

late deceleration:

onset after contraction is established wiht slow return to baseline; indicative of fetal hypoxia,

Gestational hypertension

onset of hypertension wihtout prteinuria or edema after 20 weeks of pregnancy; resolves after delivery

dilation

opening of cervix os during labor (0-10 cm)

Methadone is given for the treatment of...

opiate withdrawal

In what phase of the RN/Client relationship is the RN meeting the client and establishing his/her role?

orientation

Long term cortisone therapy can cause:

osteoporosis, weakness, muscle-wasting, depression, alopecia, buffalo hump, obesity, mood swings, slow wound healing

Plegia:

paralysis

Nonmechanical obstruction:

paralytic ileus, no mechanical blockage absence of peristalsis

Cluster A disorders include...

paranoid, schizoid, and schizotypal personality disorders.

presentation:

part of fetus that presents to enter maternal pelvic. (cephalic/vertex: head, Breech/buttocks: frank-flexion of hips and extension of knee, complete-flexion of hip and knee, footing/incomplete-extension of hips and knees)

An incomplete spinal cord injury results in...

partial function remaining below the injury.

Billroth I

partial removal of stomach

Five factors influence the labor process

passageway, passenger, powers, position, psychological response

Antisocial personality disorder is characterized by a...

pattern of disregard for the rights of others and a failure to learn from past mistakes; have history of law violations

Frequency of contractions are measured from the...

peak of one contraction to the peak of the next contraction (or beginning to beginning)

At 3 years, the preschooler can...

pedal a tricycle, jump in place, broad jump, balance on one foot, walk up and down steps using alternating feet, build a tower of 9 or 10 cubes, copy a circle, put facial features on a circle, and feed and dress; 900 words, complete sentences of 3-4 words, asks many questions, begins to sing songs

Treatment of syphilis during pregnancy

penicillin or other antibiotics

meningitis tx

penicillin, cephalosporin, vancomycin, droplet precaution

Treatment of gonorrhea during pregnancy

penicillin, tetracycline, Rocephin 125 mg IM in single dose with Vibramycin 100 mg twice daily for one week

Lecithin/sphingomyelin (L/S) ratio

performed on amniotic fluid to detect lung maturity

Estriol

performed on amniotic fluid which indicates fetal distress

Second stage of labor: complete cervical dilation to delivery of fetus

phase 1: 0 to +2 station phase 2: +2 to +4 station (increase in bloody show) phase 3: +4 to birth fetal head visible

Treatment of variable decelerations includes...

place mother in Trendelenburg position, oxygen administration, IV fluids, and notify MD.

for late deceleration:

place mother on left side, knee chest position or trendelendburg; administer o2, slow or stop oxytocin prepare for c-section

Third stage of labor: birth and delivery of placenta

placental delivery-slight gush of blood and lengthening of umbilical cord; check fragments

Treatment of physiologic jaundice includes...

placing baby under a bili-light; clothing removed, eyes and genitals covered; turn infant often

Systemic Lupus Erythematosus (SLE): assessment

polyarthralgia, arthritis, joint swelling, butterfly rach across bridge of nose ad cheeks,

In childhood, symptoms of hirschusprung disease (congenital aganglionic megacolon)

poor appetite, poor growth, abdominal distention, infrequent passage of foul-smelling, ribbonlike stools, palpable fecal masses

Cirrhosis complications

portal hypertension; edema and ascites, hepatic encephalopathy,

Nursing care that can decrease ICP

positioning to prevent neck and hip flexion, limiting suctioning, space nursing care, preventing isometric muscle contraction, elevate HOB as ordered, and carefully regulate administration of IV fluids to prevent fluid volume excess.

Murphy's Sign:

positive a sign of gallbladder disease consisting of pain on taking a deep breath when pressure is placed over the location of the gallbladder

Principles of skin protection:

pouch opening 1/8 in larger than stoma, use skin barrier under all tapes, use skin barrier to protect skin immediately surrounding the stoma, cleanse skin gently and pat dry (do not rub) pouch applied by pressing adhesive area to sin for 30 seconds, change appliance when seal breaks or when 1/3 or 1/4 full

Addisonian crisis

precipitated by physical or emotional stress, sudden withdrawal of hormone.

Alpha-fetal protein (AFP)

predict neural tube defects, threatened abortion, fetal distress, decreased AFP suggest Down syndrom -done 16-18 wks

Gestational trophoblastic disease (hydatidiform mole): plan

pregnancy is discouraged for 1 yr. curettage to completely remove all molar tissue that can become malignant

who are the most susceptible pts for UTI?

pregnant and/or sexually active women

cholelithiasis

presence of stones in the gall bladder

Medications used to treat H pylori:

prilosec and prevacid - acid reducers and proton pump inhibitors -

Symptoms of MG include...

profound muscle weakness, fatigue, respiratory failure, and lack of facial expression

Guillain barre syndrome: diagnosis

progressive inflammatory autoimmune response occuring in peripheral nervous system,

pyloric stenosis: assessment

projectile in infants, palpable olive-shaped tumor in epigastrium

Systemic Lupus Erythematosus (SLE): plan

protect skin from Ultraviolet rays and sunlight because of photosensitivity scaly and itchy rash.

Bell's Palsy Nursing Interventions:

protect the eyes. Eyes can be excessively dry or teary.

dystonias

protruding tongue, abnormal posturing

Abdominal hernias: Dx

protrusion of an organ through the wall of the cavity in which it is normally contained.

Treatment of hallucinogen overdose...

provide a quiet environment and sedation for anxiety

Treatment of sitmulant overdose...

provide respiratory and cardiac support, treat hyperpyrexia and seizures

Interventions for the client with acute mania include...

providing a quiet, nonstimulating environment and protecting them from physical exhaustion; provide high-calorie, high-protein finger foods and snacks that can be eaten while moving about; administer medications to stabilize mood (lithium, valporic acid, carbamezepine)

When hospitalized every pt is considered a _________

psych pt

Signs of overdose related to hallucinogen abuse...

psychosis, possible hypertensive crisis, hyperthermia, seizures

If pt is undergoing shockwave therapy as tx for stones what is an important teaching?

push fluids - stones will be broken up into sandlike particles

Clients with obsessive-compulsive personality disorder...

put off making decisions until all the facts are in; thus, they might do good work but not be very productive.

how often should IV site be checked by RN

q 1-2 hrs

WHen should you get a Mammography

q 1-2 yrs beginning age 40; q year beginning age 50

Usual contraction patterns

q 2-3 minutes lasting 90 seconds

On average, the birth weight of a toddler...

quadruples by age 2 1/2 years. Weight gain slow to average of 4-6 pounds per year.

U wave:

r/t to hypokalemia if seen ventricles repol late

Precipitate delivery

rapid labor and delivery; at risk for uterine rupture, vaginal lacerations, amniotic emboli, and postpartal hemorrhage

dumping syndrome:

rapid passage of food from stomach causing diaphoresis, diarrhea, hypotension

Hydatidiform mole

rapid poliferation of cells within the uterus due to trophoblastic disease

nystagmus

rapid shaking of the eyes

s/s fluid overload:

rapid/bounding pulse, distended neck veins, HTN, cough, SOB, crackles, HA, restlessness

Phenylketonuria:

rare condition in which a baby is born without the ability to properly break down an amino acid called phenylalanine.

Peritonitis s/s:

rebound tenderness, muscular rigidity, laying still w/fast shallow breaths, distended abd, ascites, fever

oophorectomy

removal of ovaries

gastrectomy

removal of stomach and attachment to upper portion of deodenum

total (simple mastectomy)

removal of the entire breast with lymph nodes intact

hysterectomy

removal of uterus

panabdominal hysterctomy

removal of uterus, fallopian tube, ovaries and is for endometriosis or carcinoma

Treatment for autonomic dysreflexia includes...

remove triggering stimuli; elevate head; check for full bladder or kinked Foley catheter tubing; administer antihypertensive medications; check for impaction after episode has resolved

Endoscopic retrade cholangipancreatogrpahy (ERCP)

removes stones from bile duct no incision, done under sedation no anesthesia.

A spinal cord injury located above C4 results in loss of...

respiratory muscle function.

dumping syndrome: teaching

restrict fluids with meals, drink 1 hr ac or 1 hr pc

Kernicterus

results when unconjugated bilirubin crosses blood-brain barrier; results in cerebral palsy

Clonus:

rhythmic contraction and alternate relaxation of a limb that is caused by suddenly stretched position

when should infants be introduced to solid foods?

rice cereal starting at 4-5 months

ROA

right occipt anterior; back of baby's head is anterior and tilted to the right side

Prostaglandin gel is used to...

ripen or soften the cervix.

Occurs at 4 and 6 months

rolls over from front to back and from back to side, infant plays with her own hands and toes, infant grasp object with both hands

stages of lochia

rubra (1-3 days): bloody flesh odor serosa (4-9 days): pink /brown alba (10+days): yellow /white

types of tube used for intestinal obstruction:

salem sump or levin tubes (NG tube) miller-abbot (intestinal) Cantor (intestinal)

When limitations are placed on a manic client's behavior, he typically reacts with...

sarcasm and beligerence.

Nerve blocks: Pudendal block

second stage of labor, episiotomy, and birth; blocks nerve impulses to perineum, cervix, and vagina

At 12-18 years, the adolescent develops...

secondary sex characteristics.

Schizophrenia: Assessment

sees world as a hostile, threatening place, hallucination and delusions,

Jitteriness in a newborn is a sign of...

seizure.

Aura

sensation of light or warmth that may precede the onset of a migraine or seizure.

contraindications to immunization

severe febrile illness, altered immune system, previous allergic response, recent acquired passive immunity

Prior to an abdominal ultrasound, the client...

should not void.

cirrhosis: plan

shunts to relieve portal hypertension, high protein, carbs, diet. sodium fat restrictions

The client with schizoid personality disorder is...

shy, aloof, and withdrawn. Hermit.

At 9 months, the infant can...

sit without support, transfer objects from hand to hand, bang cubes together, play patty-cake, creep on hands and knees, and pull herself to a standing position. Upper lateral incisors begin to appear. Reaches for familiar people, can say "mama" and "dada", and responds to simple verbal requests.

Surgical repair of a cleft palate is generally performed between...

six months and two years of age depending on whether the defect involves the hard palate or soft palate.

At 5 years, the preschooler can...

skip and hop on alternate feet, throw and catch ball, jump from height of 12 inches, balance on alternate feet with eyes closed, tie shoelaces, use scissors, begin to print a few letters and numbers, copy diamond and triangle, draw stick figure with 7 to 9 parts; 2,100 words, use sentence 6-8 words, many questions, name days of week and months of year, associative play without rules, enjoy wading pools, tricycles, wagons, dolls, books with pictures, musical toys, finger paints, toys that imitate objects used by adults

The preschooler's appearance is usually...

slender and agile and takes great pride in showing off for others.

Withdrawal from barbiturates should be done by...

slow taper to avoid fatal seizures.

PD: PARKINSON'S DISEASE : disease discription

slow, progressive disorder of the nervous system that affects movement. Characterized by tremor at rest, muscle rigidity and akinesia due to lack of dopamine.

Signs of sedative-hypnotic abuse (barbiturates, benzodiazepines)...

slurred speech, unsteady gait, drowsiness, decreased BP, irritability, inability to concentrate

pyloric stenosis: postoperative

small frequent feeding of glucose water or electrolyte solution 4-6 hrs Post Op, if clear fluids rained start formula 24 hrs postop. Fold diapers so it doesnt touch incision site.

Infant with Fetal alcohol syndrome are born with

small head, underdeveloped cheek bones, low birth weight

Intrauterine devices

small t shaped medicated decice inserted into uterine cavity

Trigeminal Neuralgia (Tic Douloureux) Assessment:

stabbing or burning facial pain. Twitching grimacing of facial muscles

Guillian-Barre S/S

starts with weakness of lower extremities and gradually progresses to upper extremities and facial muscles. Recovery is slow and can take years. "ground to brain"

Crohn's disease: assessment

steatorrhea, abdominal pain pc meals, ileum and right colon affected.

Guillain-Barre is often treated with...

steroids to decrease immune response, IV immunoglobulin (filtered) and plasmapheresis to remove circulating antibodies and speed healing response

dyskinesia

stiff neck, difficulty swallowing

contraction lasting longer than 90 seconds or occur more frequent than 2 min

stop infusion; hypertonic contractions

pink or reddish streaks on breast, abdomen buttocks, and or thigh

striae gravidarum

Presumptive signs of pregnancy

striae gravidarum, linea nigra, chloasma, increased BBT

Lightening

subjective sensation as fetus descends into pelvic inlet

Treatment of Parkinson's includes...

supportive care and medications (levodopa Carbidopa (Sinemet) Artane, and Cogentin)

Treatment of Alzheimer's includes...

supportive care and medications (ropinirole hydrochloride (Requip), tacrine hydrochloride (Cognex), folic acid, donepezil (Aricept), and rivastigmine (Exelon))

Postpartum infection:

temperature 100.4 F or higher on any two consecutive postpartum days exclusive of the first 24 hrs; chills; tachycardia

Spontaneous Abortions

termination of pregnancy spontaneously at any time before the fetus has attained viability

in what phase would the symptoms of the client be relieved/managed and the contract fulfilled?

termination phase

Pts taking Pyridium should be taught what?

that a reddish orange discoloration of urine may occur.

A positive reading for oxytocin challenge test (contraction stress test) indicates...

that labor might not be advisable.

modified radial mastectomy

the breast, axillary nodes, and superior nodes are removed but muscles are preserved

Glaucoma: open angle

the flow of aqueous humor through the trabecular meshwork is slowed and eventually builds up

Kerning's sign

the inability to extend legs

Agnosia

the inability to recognize familiar objects.

agnosia

the inability to recognize objects by sight

If neuroleptic malignant syndrome occurs due to antipsychotic medication...

the medication should be discontinued and an antiparkinsonian medication should be given.

Presentation

the part of the fetus that engages and presents first at delivery

Position

the relationship of the presenting part to the mother's pelvis

By one year of age, the circumferences of the head and chest are approximately...

the same.

abdominal hysterectomy

tx of choice for uterine fibroids with excessive vaginal bleeding.

myringotomy

tympanic membrane incision to relieve pressure and release purulent fluid.

Myringotomy

tympanic membrane incision to relieve pressure and release purulent fluid; no water can enter ear

trigeminal neuralgia (tic douloureux)

type of neuralgia involving 5th cranial nerve

ectopic pregnancy: Assessment

unilateral lower quadrant pain after 4-6 wks of normal pregnancy, referred shoulder pain, low hematocrit, frank bleeding

What is acid reflux?

when stomach acid splashes up into esophagus

IUD should not be used in clients...

with a history of PID, diabetes, or bleeding disorders.

In what phase is the RN maintaining the contract, gathering further data, promoting self-sufficiency and continuing the nursing process?

working phase

Nerve blocks: Local infiltration

xylocaine for episiotomy

Type 1 diabetes

-Insulin producing pancreatic beta cells destroyed by autoimmune process -requires insulin -KETOSIS prone

Mini Pill *progestin

-PO daily, thickens cervical mucus, decrease sperm penetration,

RN implications before GI meds can be given:

-Upper and lower GI assessment -Allergies -Assess fluid/electrolyte status

Diabetic Ketoacidosis: plan

-Volume replacement" 1L of 0.9% NS > .45% NS> D5W -IV insulin 5 units/h -Check K

Amniocentesis

-amniotic fluid aspirate by needle insertion -done at 16 wks to detect genetic disorder -takes 2-4 wks for results

Estradiol and levonorgestrel (emergency postcoital contraception)

-antifertility taken with 72 hrs of unprotected sex and repeat 12 hrs later

Detached Retina: postoperative care

-avoid sneezing, coughing straining at stool, bending down -discharge teaching: avoid stooping or straining at stool

Detached Retina: Implementation

-bedrest don't bend forward, avoid excessive movement -avoid strenuous activity for 3 months

Diabetic Ketoacidosis: assessment

-blood sugar 300-800 mg/dl -skin warm and dry, dry mucous membrane, elevated temp -kussmaul respirations -fruity odor to breath

Retinopathy of prematurity (ROP): Diagnosis

-cause of blindness in premature infants -High concentrations of oxygen causes premature infants retinal to constrict causing blindness -[02] greater than 40% for 48-72 hrs

Glaucoma: Assessment

-cloudy, blurry vision, or loss of vision -halos -HA, N/V=Increase in ICP

Strabismus: Assessment

-crossed eye, diplopia -child tilts head or squints to focus

Type 2 diabetes

-decreased sensitivity to insulin (insulin resistance) or decreased production of insulin -tx with diet and exercise

Strabismus: Diagnosis

-eyes don't function as a unit -imbalance of the extraocular muscles

Detached Retina: Assessment

-flashes of light -delineated areas of vision blank -loss of vision -confusion, apprehension

Contraction stress test (CST)

-nipple stimulation or oxytocin stimulation evaluates fetal response to stress -no deceleration with a min of 3 contractions lasting 40-60 seconds in 10 min -done at 28 wks

Cataracts: assessment

-object appear distorted and blurred; decreased color perception -glare, double vision

diaphragm

-shallow rubber device that fits over cervix; -insert up to 6 hrs before sex and leave it in place for 6 hrs no more than 24 hrs. -size must be check q 2 yrs

Ultra sound

-show fetal image -5 wks to confirm pregnancy -full bladder helps to clear image

Depo-Provera (synthetic progesterone)

-suppresses ovulation, changes uterine lining making it harder for sperm to sneter or survive the uterus -REQUIRES INJECTION Q 3 months

Contraceptive patch:

-suppresses ovulation, decrease sperm motility, thickens cervical mucus -change q/wk; patch applied to fatty areas

13. A nurse is caring for a client receiving dopamine. Which of the following potential nursing diagnoses is appropriate for this client? 1. Fluid volume, excess 2. Cardiac output, increased 3. Tissue perfusion, ineffective 4. Sensory perception, disturbed

13. 3 Rationale: The client receiving dopamine therapy should be assessed for ineffective tissue perfusion related to peripheral vasoconstriction. Options 1, 2, and 4 are not related directly to this medication therapy. Test-Taking Strategy: Use the process of elimination. Recalling that dopamine causes peripheral vasoconstriction will direct you to option 3. Review the action of this medication if you had difficulty with this question.

14. A nurse is preparing to ambulate a client on the third day after cardiac surgery. The nurse would plan to do which of the following to enable the client to best tolerate the ambulation? 1. Remove telemetry equipment. 2. Provide the client with a walker. 3. Premedicate the client with an analgesic. 4. Encourage the client to cough and deep breathe.

14. 3 Rationale: The nurse should encourage regular use of pain medication for the first 48 to 72 hours after cardiac surgery because analgesia will promote rest, decrease myocardial oxygen consumption resulting from pain, and allow better participation in activities such as coughing, deep breathing, and ambulation. Options 2 and 4 will not help in tolerating ambulation. Removal of telemetry equipment is contraindicated unless prescribed. Test Taking Strategy Use the process of elimination. Focus on the subject, how best to tolerate the ambulation. Coughing and deep breathing will not actively help endurance, so eliminate option 4. Removal of telemetry equipment is contraindicated unless ordered. From the remaining options, focusing on the subject will direct you to option 3. Review comfort measures for the client following cardiac surgery if you had difficulty with this question.

17. A client has developed paroxysmal nocturnal dyspnea. Which of the following medications does a nurse anticipate will be prescribed by the physician? 1. Propranolol (Inderal) 2. Bumetanide (Bumex) 3. Lidocaine (Xylocaine) 4. Streptokinase (Streptase)

17. 2 Rationale: Bumetanide (Bumex) is a diuretic. The paroxysmal nocturnal dyspnea may be due to increased venous return when the client is lying in bed, and the client needs diuresis. Propranolol is a b-blocker, lidocaine is an antiarrhythmic, and streptokinase is a thrombolytic. Test-Taking Strategy: Use the process of elimination. Knowledge of each medication type and that a diuretic will increase urine output will direct you to option 2. Review the actions of the medications identified in the options, if you had difficulty with this question.

Posterior fontanel closes by approximately...

18 months of age.

When should you get a pap smear:

18 yo or earlier if sexually active. then annually until 3 consecutive normal paps

Testicular self exam

18-20 monthly

25. A client is admitted to a hospital with acute myocardial infarction and is started on tissue plasminogen activator (tPA, Activase) by infusion. Of the following parameters, which one would a nurse determine requires the least frequent assessment to detect complications of therapy with tissue plasminogen activator? 1. Neurological signs 2. Presence of bowel sounds 3. Blood pressure and pulse 4. Complaints of abdominal and back pain

25. 2 Rationale: Thrombolytic agents dissolve existing clots, and bleeding can occur anywhere in the body. The nurse monitors for any obvious signs of bleeding and also for occult signs of bleeding, which would include hemoglobin and hematocrit values, blood pressure and pulse, neurological signs, assessment of abdominal and back pain, and the presence of blood in the urine or stool. Test-Taking Strategy: Note the strategic words least frequent assessment. Remember that bleeding is the primary complication of thrombolytic therapy. Therefore, look for the option that is not related to bleeding. A change in neurological signs could indicate cerebral bleeding, abdominal and back pain could indicate abdominal bleeding, and change in blood pressure and pulse could be general indicators of hemorrhage. The presence of bowel sounds is unrelated to this medication. Review nursing considerations for the client receiving tissue plasminogen activator if you had difficulty with this question.

27. A nurse is preparing to defibrillate a client in ventricular fibrillation. After placing the paddles on the client's chest and before discharging them, which of the following should be done? 1. Ensure that the client has been intubated. 2. Set the defibrillator to the "synchronize" mode. 3. Administer lidocaine hydrochloride (Xylocaine). 4. Confirm that the rhythm is actually ventricular fibrillation.

27. 4 Rationale: Until the defibrillator is attached and charged, the client is resuscitated by using cardiopulmonary resuscitation. Once the defibrillator has been attached, the electrocardiogram is checked to verify that the rhythm is ventricular fibrillation or pulseless ventricular tachycardia. Leads also are checked for any loose connections. A nitroglycerin patch, if present, is removed. The client does not have to be intubated to be defibrillated. Lidocaine may be given subsequently but is not required before defibrillation. The machine is not set to the synchronous mode because there is no underlying rhythm with which to synchronize. Test-Taking Strategy Use the process of elimination, focusing on the subject, ventricular fibrillation. Note that option 4 directly addresses this subject and also addresses assessment of the client. Review the procedure for defibrillation if you had difficulty with this question.

3. In reviewing the medication records of the following group of clients, the nurse determines that which client would be at greatest risk for developing hyperkalemia? 1. Client receiving furosemide (Lasix) 2. Client receiving bumetanide (Bumex) 3. Client receiving spironolactone (Aldactone) 4. Client receiving hydrochlorothiazide (HCTZ)

3. 3 Rationale: Spironolactone is a potassium-sparing diuretic and competes with aldosterone at receptor sites in the distal tubule, resulting in excretion of sodium, chloride, and water and retention of potassium and phosphate. Use of the medications noted in options 1, 2, and 4 could result in hypokalemia. Test-Taking Strategy: Use the process of elimination. Eliminate options 1 and 2 because they are both loop diuretics, which lead to the side effect of hypokalemia. Next eliminate option 4 because it is a thiazide diuretic, which acts on the distal tubule and inhibits sodium, chloride, and potassium reabsorption. Review the effects of these medications if you had difficulty with this question.

34. A nurse is caring for a client immediately after insertion of a permanent demand pacemaker via the right subclavian vein. Which of the following activities will assist with preventing dislodgement of the pacing catheter? 1. Limiting movement and abduction of the left arm 2. Limiting movement and abduction of the right arm 3. Assisting the client to get out of bed and ambulate with a walker 4. Having the physical therapist do active range-of-motion exercises to the right arm

34. 2 Rationale: In the first several hours after insertion of a permanent or a temporary pacemaker, the most common complication is pacing electrode dislodgment. The nurse helps prevent this complication by limiting the client's activities of the arm on the side of the insertion site. Test-Taking Strategy Use the process of elimination. Note that the pacemaker was inserted on the right side. Therefore, to prevent pacing electrode dislodgment, motion must be limited on that side. Options 3 and 4 involve movement of the right arm and are eliminated first. Limiting the movement of the left arm (option 1) is of no benefit to the client. Thus, option 2 is the correct option. Review care of the client following insertion of a pacemaker if you had difficulty with this question.

37. A client is having a follow-up physician office visit after vein ligation and stripping. The client describes a sensation of "pins and needles" in the affected leg. Which of the following would be an appropriate action by the nurse based on evaluation of the client's comment? 1. Instruct the client to apply warm packs. 2. Report the complaint to the physician. 3. Reassure the client that this is only temporary. 4. Advise the client to take acetaminophen (Tylenol) until it is gone.

37. 2 Rationale: Hypersensitivity or a sensation of "pins and needles" in the surgical limb may indicate temporary or permanent nerve injury following surgery. The saphenous vein and saphenous nerve run close together in the distal third of the leg. Because complications from this surgery are relatively rare, this symptom should be reported. Test-Taking Strategy Use the process of elimination. Pins and needles sensations usually indicate nerve irritation or damage. If you know this, you can eliminate options 1 and 4. Reassuring the client about something being "only temporary" is often not an appropriate action, unless this is known to be absolutely true. Review the complications associated with vein ligation and stripping if you had difficulty with this question.

5. A client who is receiving digoxin (Lanoxin) daily has a serum potassium level of 3.0 mEq/L and is complaining of anorexia. A physician orders a digoxin level to rule out digoxin toxicity. A nurse checks the results, knowing that which of the following is the therapeutic serum level (range) for digoxin? 1. 0.5 to 2 ng/mL 2. 1.2 to 2.8 ng/mL 3. 3 to 5 ng/mL 4. 3.5 to 5.5 ng/mL

5. 1 Rationale: Therapeutic levels for digoxin range from 0.5 to 2 ng/mL. Therefore, options 2, 3, and 4 are incorrect. Test-Taking Strategy: Knowledge of the therapeutic serum digoxin level will direct you to option 1. If you had difficulty with this question, learn the therapeutic level for digoxin.

BUN Normal range

6-20mg/100 ml

Symptoms of alcohol withdrawal usually begin aobut...

6-8 hours after the client's last drink, or when the amount consumed is less than usual.

During school age, the child gains on average...

6-8 pounds a year, 85-90 pounds by age 12 years.

6. A client is being treated with procainamide (Procanbid) for a cardiac dysrhythmia. Following intravenous administration of the medication, the client complains of dizziness. What intervention should the nurse take first? 1. Administer ordered nitroglycerin tablets. 2. Measure the heart rate on the rhythm strip. 3. Obtain a 12-lead electrocardiogram immediately. 4. Auscultate the client's apical pulse and obtain a blood pressure.

6. 4 Rationale: Signs of toxicity from procainamide include confusion, dizziness, drowsiness, decreased urination, nausea, vomiting, and tachydysrhythmias. If the client complains of dizziness, the nurse should assess the vital signs first. Although options 2 and 3 may be interventions, these would be done after the vital signs are taken. Nitroglycerin is a vasodilator and will lower the blood pressure. Test-Taking Strategy: Use the steps of the nursing process to eliminate options 1 and 3. From the remaining options, remember always to assess the client first, not the monitoring devices. Therefore, option 4 is correct. Review the signs of toxicity and the nursing interventions if you had difficulty with this question.

Fasting blood glucose: Normal range

60-110 mg/dl

newborn normal blood pressure

65/41

715. A client with angina complains that the anginal pain is prolonged and severe and occurs at the same time each day, most often at rest in the absence of precipitating factors. How would the nurse best describe this type of anginal pain? 1. Stable angina 2. Variant angina 3. Unstable angina 4. Nonanginal pain

715. 2 Rationale: Variant angina, or Prinzmetal's angina, is prolonged and severe and occurs at the same time each day, most often at rest. Stable angina is induced by exercise and relieved by rest or nitroglycerin tablets. Unstable angina occurs at lower and lower levels of activity or at rest, is less predictable, and is often a precursor of myocardial infarction. Test-Taking Strategy: Use the process of elimination, focusing on the data in the question. Noting the strategic words at rest will direct you to option 2. If you had difficulty with this question, review the characteristics of the various types of angina.

717. A client with atrial fibrillation is receiving a continuous heparin infusion at 1000 units/hr. The nurse would determine that the client is receiving the therapeutic effect based on which of the following results? 1. Prothrombin time of 12.5 seconds 2. Activated partial thromboplastin time of 60 seconds 3. Activated partial thromboplastin time of 28 seconds 4. Activated partial thromboplastin time longer than 120 seconds

717. 2 Rationale: Common laboratory ranges for activated partial thromboplastin time are 20 to 36 seconds. Because the activated partial thromboplastin time should be 1.5 to 2.5 times the normal value, the client's activated partial thromboplastin time would be considered therapeutic if it was 60 seconds. Test-Taking Strategy: Use the process of elimination. Option 1 is eliminated because the prothrombin time assesses response to warfarin (Coumadin) therapy. Eliminate option 3 because at 28 seconds the client is receiving no therapeutic effect from the continuous heparin infusion. Eliminate option 4 because this value is beyond the therapeutic range and the client is at risk for bleeding. Review laboratory tests to monitor the effectiveness of heparin therapy if you had difficulty with this question.

726. A client is on nicotinic acid (niacin) for hyperlipidemia and the nurse provides instructions to the client about the medication. Which statement by the client would indicate an understanding of the instructions? 1. "It is not necessary to avoid the use of alcohol." 2. "The medication should be taken with meals to decrease flushing." 3. "Clay-colored stools are a common side effect and should not be of concern." 4. "Ibuprofen (Motrin) taken 30 minutes before the nicotinic acid should decrease the flushing."

726. 4 Rationale: Flushing is a side effect of this medication. Aspirin or a nonsteroidal anti-inflammatory drug can be taken 30 minutes prior to taking the medication to decrease flushing. Alcohol consumption needs to be avoided because it will enhance this side effect. The medication should be taken with meals but this will decrease gastrointestinal upset; taking the medication with meals has no effect on the flushing. Clay-colored stools are a sign of hepatic dysfunction and should be immediately reported to the physician. Test-Taking Strategy: Use the process of elimination. Option 1 can be eliminated because alcohol must be abstained from. Option 2 can be eliminated because taking the medication with meals helps decrease the gastrointestinal symptoms. The clay-colored stools in option 3 is a sign of hepatic dysfunction and should be immediately reported to the physician. Review the client teaching points related to this medication if you had difficulty with this question.

729. Intravenous heparin therapy is prescribed for a client. While implementing this prescription, a nurse ensures that which of the following medications is available on the nursing unit? 1. Protamine sulfate 2. Potassium chloride 3. Aminocaproic acid (Amicar) 4. Vitamin K (AquaMEPHYTON)

729. 1 Rationale: The antidote to heparin is protamine sulfate; it should be readily available for use if excessive bleeding or hemorrhage should occur. Vitamin K is an antidote for warfarin sodium. Aminocaproic acid is the antidote for thrombolytic therapy. Potassium chloride is administered for a potassium deficit. Test-Taking Strategy: Knowledge regarding the various antidotes is needed to answer this question. Remember the antidote to heparin is protamine sulfate. Learn these antidotes if you had difficulty with this question.

730. A client is receiving thrombolytic therapy with a continuous infusion of streptokinase (Streptase). The client suddenly becomes extremely anxious and complains of itching. A nurse hears stridor and on examination of the client notes generalized urticaria and hypotension. Which of the following should be the priority action of the nurse? 1. Administer oxygen and protamine sulfate. 2. Stop the infusion and call the physician. 3. Cut the infusion rate in half and sit the client up in bed. 4. Administer diphenhydramine

730. 2 Rationale: The client is experiencing an anaphylactic reaction to streptokinase, which is allergenic. The infusion should be stopped, the physician notified, and the client treated with epinephrine, antihistamines, and corticosteroids. Test-Taking Strategy: Recall that an allergic reaction and possible anaphylaxis are risks associated with streptokinase therapy. Also, focusing on the signs and symptoms in the question will assist in answering the question. When a severe allergic reaction occurs, the offending substance should be stopped, and lifesaving treatment should begin. Review the adverse effects of this medication if you had difficulty with this question.

731. A client is admitted with pulmonary embolism and is to be treated with streptokinase (Streptase). A nurse would report which of the following assessments to the physician before initiating this therapy? 1. Adventitious breath sounds 2. Temperature of 99.4° F orally 3. Blood pressure of 198/110 mm Hg 4. Respiratory rate of 28 breaths/min

731. 3 Rationale: Thrombolytic therapy is contraindicated in a number of preexisting conditions in which there is a risk of uncontrolled bleeding, similar to the case in anticoagulant therapy. Thrombolytic therapy also is contraindicated in severe uncontrolled hypertension because of the risk of cerebral hemorrhage. Therefore the nurse would report the results of the blood pressure to the physician before initiating therapy. Test-Taking Strategy: Use the process of elimination and focus on the client's diagnosis. Options 1, 2, and 4 may be present in the client with pulmonary embolism but are not necessarily signs that warrant reporting before this therapy is initiated. Review the contraindications associated with the administration of this medication if you had difficulty with this question.

732. The nurse is monitoring a client who is taking digoxin (Lanoxin) for adverse effects. Which findings are characteristic of digoxin toxicity. Select all that apply. 1. Tremors 2. Diarrhea 3. Irritability 4. Blurred vision 5. Nausea and vomiting

732. 2, 4, 5 Rationale: Digoxin (Lanoxin) is a cardiac glycoside. The risk of toxicity can occur with the use of this medication. Toxicity can lead to life-threatening events and the nurse needs to monitor the client closely for signs of toxicity. Early signs of toxicity include gastrointestinal manifestations such as anorexia, nausea, vomiting, and diarrhea. Subsequent manifestations include headache, visual disturbances such as diplopia, blurred vision, yellow-green halos, photophobia, drowsiness, fatigue, and weakness. Cardiac rhythm abnormalities can also occur. The nurse also monitors the digoxin level. Therapeutic levels for digoxin range from 0.5 to 2 ng/mL. Test-Taking Strategy: Specific knowledge regarding the characteristics of digoxin toxicity is needed to answer this question. Recall that the early signs are gastrointestinal manifestations. Next, recall that visual disturbances occur. If you had difficulty with this question review the manifestations associated with digoxin toxicity.

3, 5, 2, 4, 1

A 67-year old patient will be discharged from the hospital in the morning. The health care provider has ordered three new medications for her. Place the following steps of the nursing process in the correct order: 1. The nurse returns to the patient's room and asks her to describe the medications she will be taking at home. 2. The nurse talks with the patient and family about who will be available if the patient has difficulty taking medicines and considers consulting with the health care provider about a home health visit. 3. The nurse asks the patient if she is in pain, feels tired, and is willing to spend the next few minutes learning about her medications. 4. The nurse brings the containers of medicines and information leaflets to the bedside and discusses each medication with her. 5. The nurse considers what she learns from the patient and identifies the patient's nursing diagnosis.

4. Informatics

A critical care nurse is using a computerized decision support system to correctly position her ventilated patients to reduce pneumonia cause by accumulated respiratory secretions. This is an example of which Quality and Safety in the Education of Nurses (QSEN) competency? 1. Patient-centered care 2. Safety 3. Teamwork and collaboration 4. Informatics

3. An adventitious crisis

A grandfather living in Japan worries about his two young grandsons who disappeared after a tsunami. This is an example of: 1. A situational crisis 2. A maturational crisis 3. An adventitious crisis 4. A developmental crisis

3. Basic critical thinking

A nurse has been working on a surgical unit for 3 weeks. A patient requires a Foley catheter to be inserted, so the nurse reads the procedural manual for the institution to review how to insert it. The level of critical thinking that the nurse is using is: 1. Commitment 2. Scientific method 3. Basic critical thinking 4. Complex critical thinking

2. Evaluation

A patient had hip surgery 16 hours ago. During the previous shift the patient had 40 mL of drainage in the surgical drainage collection device for an 8 hour period. The nurse refers to the written plan of care, noting that the health care provider is to be notified when drainage in the device exceeds 100 mL for the day. On entering the room, the nurse looks at the device and carefully notes the amount of drainage currently in it. This is an example of: 1. Planning 2. Evaluation 3. Intervention 4. Diagnosis

4. Implementation

A patient in the emergency department has developed wheezing and shortness of breath. The nurse gives the ordered medicated nebulizer treatment now and in 4 hours. Which standard of practice performed? 1. Planning 2. Evaluation 3. Assessment 4. Implementation

2. Expressing the importance of learning the skill correctly

A patient needs to learn how to administer a subcutaneous injection. Which of the following reflects that the patient is ready to learn? 1. Describing difficulties a family member has had in taking insulin 2. Expressing the importance of learning the skill correctly 3. Being able to see and understand the markings on the syringe 4. Having the dexterity needed to prepare and inject the medication

3. Provide only the information that the patient needs to go home

A patient newly diagnosed with cervical cancer is going home. The patient is avoiding discussion of her illness and postoperative orders. What is the nurse's best plan in teaching this patient? 1. Teach the patient's spouse 2. Focus on knowledge the patient will need in a few weeks 3. Provide only the information that the patient needs to go home 4. Convince the patient that learning about her health is necessary

4. With silence.

A patient newly diagnosed with type 2 diabetes says, "My blood sugar was just a little high. I don't have diabetes." The nurse responds: 1. "Let's talk about something cheerful." 2. "Do other members of your family have diabetes?" 3. "I can tell that you feel stressed to learn that you have diabetes." 4. With silence.

3. Conducting reflective practice.

A patient on a surgical unit develops sudden shortness of breath and a drop in blood pressure. The staff respond, but the patient dies 30 minutes later. The manager on the nursing unit calls the staff involved in the emergency response together. The staff discusses what occurred over the 30-minute time frame, the actions taken, and whether other steps should have been implemented. The nurses in this situation are: 1. Problem solving. 2. Showing humility. 3. Conducting reflective practice. 4. Exercising responsibility.

41. A 6-month-old infant has been brought to the well-child clinic for a check-up. She is currently sleeping. What should the nurse do first when beginning the examination?

A) Auscultate the lungs and heart while the infant is still sleeping.

9. The nurse is conducting an interview in an outpatient clinic and is using a computer to record data. Which is the best use of the computer in this situation? Select all that apply.

A) Collect the patient's data in a direct, face-to-face manner. D) Type the data into the computer after the narrative is fully explored. E) Allow the patient to see the monitor during typing.

A pt has returned from surgery with a tracheostomy tube in place. After about 10 minutes in postoperative recovery, the pt begins to have noisy, increased respirations and an elevated heart rate. What action should the RN take immediately? A. Suction the tracheostomy. B. Readjust the tracheostomy tube and tighten the ties. C. Preform a complete respiratory assessment.

A. Noisy, increased respiration & increased pulse are signs that the pt needs immediate suctioning to clear the airway of secretions. A complete respiratory assessment may then be completed.

107. During an abdominal assessment, the nurse would consider which of these findings as normal? A) The presence of a bruit in the femoral area B) A tympanic percussion note in the umbilical region C) A palpable spleen between the ninth and eleventh ribs in the left midaxillary line D) A dull percussion note in the left upper quadrant at the midclavicular line

B) A tympanic percussion note in the umbilical region Pages: 539-540. Tympany should predominate in all four quadrants of the abdomen because air in the intestines rises to the surface when the person is supine. Vascular bruits are not usually present. Normally the spleen is not palpable. Dullness would not be found in the area of lung resonance (left upper quadrant at the midclavicular line).

113. The nurse is assessing the joints of a woman who has stated, "I have a long family history of arthritis, and my joints hurt." The nurse suspects that she has osteoarthritis. Which of these are symptoms of osteoarthritis? Select all that apply. A) Symmetric joint involvement B) Asymmetric joint involvement C) Pain with motion of affected joints D) Affected joints are swollen with hard, bony protuberances E) Affected joints may have heat, redness, and swelling

B) Asymmetric joint involvement C) Pain with motion of affected joints D) Affected joints are swollen with hard, bony protuberances Page: 608. In osteoarthritis, asymmetric joint involvement commonly affects hands, knees, hips, and lumbar and cervical segments of the spine. Affected joints have stiffness, swelling with hard bony protuberances, pain with motion, and limitation of motion. The other options reflect signs of rheumatoid arthritis.

53. An elderly woman is brought to the emergency department after being found lying on the kitchen floor 2 days, and she is extremely dehydrated. What would the nurse expect to see upon examination?

B) Dry mucous membranes and cracked lips

49. The nurse needs to perform anthropometric measures of an 80-year-old man who is confined to a wheelchair. Which of the following is true in this situation?

B) Height measurements may not be accurate because of changes in bone.

29. Which statement is true regarding the mental status examination?

B) It is usually sufficient to gather mental status information during the health history interview.

16. The nurse is reviewing concepts of cultural aspects of pain. Which statement is true regarding pain?

B) Just as patients vary in their perceptions of pain, so will they vary in their expressions of pain.

36. The nurse would use bimanual palpation technique in which situation?

B) Palpating the kidneys and uterus

The nurse is assigned to care for a client with complete right-sided hemiparesis, the nurse plans care knowing that in this condition:

B- The client has weakness on the right side of the body, including the face and tongue Rationale: Hemiparesis is a weakness of one side of the body that may occur after a stroke. Complete hemiparesis is weakness of the face and tongue, arm and leg on one side. Complete bilateral paralysis does not occur in the condition. The client with right- sided hemiparesis has weakness of the right arm and leg and needs assistance with feeding, bathing and ambulating.

Which of the following is most likely to be effective in meeting a patient's teaching/learning needs preoperatively?

B. Teaching the patient and family. A nurse should determine learning needs preoperatively and teach both the patient and the family before surgery. Using only written instructions does not provide the opportunity for evaluation for learning. Brief verbal instructions are often forgotten. Teaching only the patient limits learning preoperatively because the patient can be anxious and not receptive to new information.

Severe preeclampsia

BP is equal to or greater than 160/110 on two occasions at least six hours apart with woman at bed rest; proteinuria is greater than or equal to 5 grams in a 24-hour urine specimen; oliguria equal to or less than 400 ml in 24-hour period; cerebral or visual disturbances are reported; epigastric pain present; pulmonary edema or cyanosis reported; HELLP syndrome diagnosed

94. The nurse is assessing a patient's apical impulse. Which of these statements is true regarding the apical impulse? A) It is palpable in all adults. B) It occurs with the onset of diastole. C) Its location may be indicative of heart size. D) It should normally be palpable in the anterior axillary line.

C) Its location may be indicative of heart size. Page: 473 | Page: 492. The apical impulse is palpable in about 50% of adults. It is located in the fifth left intercostal space in the midclavicular line. Horizontal or downward displacement of the apical impulse may indicate an enlargement of the left ventricle.

95. During an assessment of an older adult, the nurse should expect to notice which finding as a normal physiologic change associated with the aging process? A) Hormonal changes causing vasodilation and a resulting drop in blood pressure B) Progressive atrophy of the intramuscular calf veins, causing venous insufficiency C) Peripheral blood vessels growing more rigid with age, producing a rise in systolic blood pressure D) Narrowing of the inferior vena cava, causing low blood flow and increases in venous pressure resulting in varicosities

C) Peripheral blood vessels growing more rigid with age, producing a rise in systolic blood pressure Pages: 504-505. Peripheral blood vessels grow more rigid with age, resulting in a rise in systolic blood pressure. Aging produces progressive enlargement of the intramuscular calf veins, not atrophy. The other options are not correct.

99. When using a Doppler ultrasonic stethoscope, the nurse recognizes arterial flow when which sound is heard? A) Low humming sound B) Regular "lub, dub" pattern C) Swishing, whooshing sound D) Steady, even, flowing sound

C) Swishing, whooshing sound Pages: 515-516. When using the Doppler ultrasonic stethoscope, the pulse site is found when one hears a swishing, whooshing sound.

97. When performing a peripheral vascular assessment on a patient, the nurse is unable to palpate the ulnar pulses. The patient's skin is warm and capillary refill time is normal. The nurse should next: A) check for the presence of claudication. B) refer the individual for further evaluation. C) consider this a normal finding and proceed with the peripheral vascular evaluation. D) ask the patient if he or she has experienced any unusual cramping or tingling in the arm.

C) consider this a normal finding and proceed with the peripheral vascular evaluation. Pages: 506-507. It is not usually necessary to palpate the ulnar pulses. The ulnar pulses are often not palpable in the normal person. The other responses are not correct.

83. The nurse knows that auscultation of fine crackles would most likely be noticed in: A) a healthy 5-year-old child. B) a pregnant woman. C) the immediate newborn period. D) association with a pneumothorax.

C) the immediate newborn period. Pages: 436-437. Fine crackles are commonly heard in the immediate newborn period as a result of the opening of the airways and clearing of fluid. Persistent fine crackles would be noticed with pneumonia, bronchiolitis, or atelectasis.

Which of the following is most appropriate after administration of preoperative medications?

C. Placing the patient in bed with the rails up. After administration of preoperative medications, a nurse should instruct a patient not to get up without assistance because medications can cause drowsiness or dizziness. Confirming that the patient has recently voided should be done before preoperative medications are administered. Monitoring vital signs every 15 minutes is not a necessary intervention unless prescribed by the physician. Transporting the patient immediately to the operating room is not necessary unless the patient is called for.

While performing preoperative teaching, the patient asks when she needs to stop drinking water before the surgery. Based on the most recent practice guidelines established by the American Society of Anesthesiologists, the nurse tells the patient that:

C. She can drink clear liquids up to 2 hours before surgery. Practice guidelines for preoperative fasting state the minimum fasting period for clear liquids is 2 hours. Evidenced-based practice no longer supports the long-standing practice of requiring patients to be NPO after midnight.

118. During the history of a 78-year-old man, his wife states that he occasionally has problems with short-term memory loss and confusion: "He can't even remember how to button his shirt." In doing the assessment of his sensory system, which action by the nurse is most appropriate? A) The nurse would not do this part of the examination because results would not be valid. B) The nurse would perform the tests, knowing that mental status does not affect sensory ability. C) The nurse would proceed with the explanations of each test, making sure the wife understands. D) Before testing, the nurse would assess the patient's mental status and ability to follow directions at this time.

D) Before testing, the nurse would assess the patient's mental status and ability to follow directions at this time. The nurse should ensure validity of the sensory system testing by making sure the patient is alert, cooperative, comfortable, and has an adequate attention span. Otherwise, the nurse may obtain misleading and invalid results.

The nurse is assessing the adaptation of the client to changes in the functional status after a stroke. The nurse assesses that the client is adapting most successfully if the client:

D- Consistently uses adaptive equipment in dressing self Rationale: Client's are evaluated as coping successfully with lifestyle changes after a brain attack (stroke) if they make appropriate lifestyle alterations, use the assistance of others, and have appropriate social interactions.

The nurse has instructed the family of a client with a stroke who has homonymous hemianopsia about measures to help the client overcome the deficit. The nurse determines that the family understands the measures to use if the state that they will:

D- Remind the client to turn the head to scan the lost visual field Rationale: Homonymous hemianopsia is loss of half of the visual field. The client with Homonymous hemianopsia should have objects placed in the intact field of vision, and the nurse also should approach the client from the intact side. The nurse instructs the client to scan the environment to overcome the visual deficit and does client teaching from within the intact field of vision: The nurse encourages the use of personal eye glasses, if they are available.

To prevent airway obstruction in the postoperative patient who is unconscious or semiconscious, the nurse

D. positions the patient in a side-lying position. An unconscious or semiconscious patient should be placed in a lateral position to protect the airway from obstruction by the tongue. Deep breathing and elevation of the head of the bed are implemented to facilitate gas exchange when the patient is responsive. Oxygen administration is often used, but the patient must first have a patent airway.

A nurse is preparing to teach a client who has been newly diagnosed with stomach cancer. Which statement should the nurse include in her teaching? 1) Stomach pain is typically a late symptom of stomach cancer. 2) Surgery is commonly a successful treatment for stomach cancer. 3) Chemotherapy and radiation are usually successful treatments for stomach cancer. 4) You may be on TPN for an extended time.

RATIONALE: 1) Stomach pain is typically a late sign of stomach cancer; outcomes are particularly poor when the cancer reaches that point. Surgery, chemotherapy, and radiation have minimal positive effects on stomach cancer. TPN may increase the growth of cancer cells.

A nurse is caring for a client with liver cirrhosis who has developed ascites and requires paracentesis. Relief of which symptom indicates that the paracentesis was effective? 1) Pruritus 2) Dyspnea 3) Jaundice 4) Peripheral neuropathy

RATIONALE: 2) Ascites (fluid buildup in the abdomen) puts pressure on the diaphragm. Paracentesis (surgical puncture of the abdominal cavity to aspirate fluid) is done to remove fluid from the abdominal cavity and thus reduce pressure on the diaphragm. The goal is to improve the client's breathing. Pruritus, jaundice, and peripheral neuropathy are signs of cirrhosis that aren't relieved or treated by paracentesis.

A nurse is caring for a client who requires total parenteral nutrition (TPN). The client asks the nurse why he's getting TPN. The nurse best response is: 1) It adds necessary fluids and electrolytes to the body 2) It gives you complete nutrition by the I.V. route until you can eat again. 3) These tube feedings provide nutritional supplementation. 4) It contains liquid protein to supplement your diet between meals.

RATIONALE: 2) TPN is given I.V. to provide all the nutrients the client needs; it provides more than just fluids and electrolytes. TPN solutions typically provide glucose, amino acids, trace elements and vitamins, and fats. TPN is neither a tube feeding nor a liquid dietary supplement.

Daily abdominal girth measurements are prescribed for a client with liver dysfunction and ascites. To increase accuracy, the nurse should use which landmark?

RATIONALE: 2) The proper technique for measuring abdominal girth involves using the umbilicus as a landmark while encircling the abdomen with a tape measure. Using the xiphoid process, the iliac crest, or the symphysis pubis as a landmark would yield inaccurate measurements.

A client is admitted with possible bowel obstruction. Which intervention is most important for the nurse to perform?

RATIONALE: 2) With a bowel obstruction, abdominal distention occurs. Measuring abdominal girth provides quantitative information about increases or decreases in the amount of distention. Monitoring daily weights provides information about fluid status. An increase In daily weight usually indicates fluid retention. Measuring intake and output provides no information about abdominal distention or obstruction. A client with an obstruction would receive a NPO order.

A client is admitted with possible bowel obstruction. Which intervention is most important for the nurse to perform? 1) Obtain daily weights. 2) Measure abdominal girth. 3) Keep strict intake and output. 4) Encourage the client to increase fluids.

RATIONALE: 2) With a bowel obstruction, abdominal distention occurs. Measuring abdominal girth provides quantitative information about increases or decreases in the amount of distention. Monitoring daily weights provides information about fluid status. An increase In daily weight usually indicates fluid retention. Measuring intake and output provides no information about abdominal distention or obstruction. A client with an obstruction would receive a NPO order.

A nurse is doing preoperative teaching with a client expected to undergo a herniorrhaphy (surgical repair of a hernia). The nurse should instruct the client to:

RATIONALE: 3) After herniorrhaphy, teach the client to avoid activities that increase intra-abdominal pressure, such as coughing, sneezing, or straining with bowel movement. If the client must cough or sneeze, splinting the incision with a pillow is helpful. Encourage the use of analgesics for pain or discomfort. The client should be instructed not to cough, but deep-breathing exercises should be still be preformed q2hrs. Ice may be used to reduce scrotal edema and pain after herniorrhaphy.

A client with a history of long term anti inflammatory use has dark, tarry stools. The nurse knows that this indicates bleeding in the:

RATIONALE: 3) Melena is the passage of dark, tarry stools that contain a large amount of digested blood. It occurs with bleeding from the upper GI tract. Passage of red blood from the rectum indicates lower GI (colon, small intestine, and rectum) bleeding. Bleeding in the lower colon would cause bright red blood in the stool.

When performing a physical assessment, the nurse discovers a client&#039;s urinary drainage bag lying next to him. Based on this finding, the nurse identifies which priority nursing diagnosis?

RATIONALE: 1) The drainage bag shouldn't be placed alongside the client or on the floor because of the increased risk of infection caused by microorganisms. It should hang on the bed in a dependent position. The other nursing diagnoses are not appropriate for this assessment finding.

When performing a physical assessment, the nurse discovers a client&#039;s urinary drainage bag lying next to him. Based on this finding, the nurse identifies which priority nursing diagnosis? - 1. Risk for infection - 2. Reflex urinary incontinence - 3. Impaired comfort - 4. Risk for compromised human dignity

RATIONALE: 1) The drainage bag shouldn't be placed alongside the client or on the floor because of the increased risk of infection caused by microorganisms. It should hang on the bed in a dependent position. The other nursing diagnoses are not appropriate for this assessment finding.

A nurse is teaching a male client how to collect a clean-catch midstream urine specimen. What cleaning technique should the nurse include in her teaching?

RATIONALE: 1) before collecting a clean-catch urine specimen, a male client should clean around the urethral meatus in a circular motion and move several inches down the shaft of the penis. When the penis is cleaned from down the shaft to up toward the urethral meatus, organisms from the skin of the penis are dragged toward the meatus. Scrubbing back and forth repeatedly moves organisms across the urethral meatus, not away from it. Some areas of the skin around the tip of the penis may be missed when cleaning is done in rows.

A client is receiving pancrelipase (Viokase) for the treatment of chronic pancreatitis. Which observation by the nurse best indicates the treatment is effective?

RATIONALE: 1) before starting a feeding, it's essential to ensure that the tube is in the proper location. Aspiration for stomach contents confirms correct placement. Giving the feeding without confirming proper placement puts the client at risk for aspiration. If an x-ray is ordered, it should be done immediately, not in 24 hrs. Clamping the tube provides no information about tube placement.

A nurse is instructing a client how to obtain an accurate clean-catch urine specimen for a urine culture. She should include what instruction? 1

RATIONALE: 1) when obtaining a clean-catch urine specimen, the perineal area should be thoroughly cleaned. The inside of the container is already sterile, so washing it would only contaminate it.

A nurse is instructing a client how to obtain an accurate clean-catch urine specimen for a urine culture. She should include what instruction?

RATIONALE: 1) when obtaining a clean-catch urine specimen, the perineal area should be thoroughly cleaned. The inside of the container is already sterile, so washing it would only contaminate it. Only a small specimen of urine is needed, so it isn't necessary to completely fill the container. The container should be closed as soon as the urine is collected to prevent contamination

A nurse is caring for a client with chronic renal failure. The laboratory results indicate hypocalcemia and hyperphosphatemia. When assessing the client, the nurse should be alert for which of the following? SELECT ALL THAT APPLY. 1) Trousseau's sign 2) Cardiac arrhythmia 3) Constipation 4) Decreased clotting time 5) Drowsiness and lethargy 6) Fractures

RATIONALE: 1, 2, 6. Hypocalcemia is a calcium deficit that causes irritability and repetitive muscle spasms. S/S of hypocalcemia include Trousseau's sign, cardiac arrhythmias, diarrhea, increased clotting times, anxiety, and irritability. The calcium-phosphorus imbalance leads to brittle bones and pathologic fractures.

While undergoing hemodialysis, a client becomes restless and tells a nurse that he has a headache and feels nauseous. Which complication does the nurse suspect?

RATIONALE: 2) Disequilibrium syndrome is caused by a rapid reduction in urea, sodium, and other solutes from the blood. This may lead to cerebral edema and increased intracranial pressure (IICP). S/S of ICCP include HA, nausea, and restlessness as well as vomiting, confusion, twitching, and seizures. Fever and elevated WBC may indicate infection. Popping or ringing in the ears, chest pain, dizziness, or coughing suggests an air embolus. Chest pain, dyspnea, burning at the access site and cramping suggests acute hemolysis.

While preparing a client for an upper GI endoscopy, which interventions should the nurse implement? SELECT ALL THAT APPLY!

RATIONALE: 2) and 4) The client shouldn't eat or drink for 6-12 hrs before the procedure to ensure that his upper GI tract is clear for viewing. The client will receive a sedative before the endoscope is inserted that will help him relax while allowing him to remain conscious. GI tract cleansing and liquid diet are interventions before a lower GI tract procedure such as colonoscopy. Food and fluids must be withheld until the gag reflex returns.

A client is admitted with acute pancreatitis. Which laboratory result should the nurse expect?

RATIONALE: 3) Amylase of 306 units/L 3)Pancreatitis involves activation of pancreatic enzymes, such as amylase and lipase. Therefore, serum amylase and lipase levels are elevated in a client with acute pancreatitis. Serum creatinine levels are elevated with kidney disfunction. Injury or disease of the liver elevated ALT levels. Troponin levels are elevated with heart damage such as myocardial infarction.

A nurse is caring for a client diagnosed with diverticulous. Which should be the nurse expect to institute?

RATIONALE: 3) Diverticulosis is characterized by an out-pouching of the colon. The client needs a high fiber diet and psyllim (bulk laxative) administration to promote normal soft stools. A low fiber diet, decreased fluid intake, bed rest, analgesics, and some antacids can lead to constipation.

After a retropubic prostatectomy, a client needs continuous bladder irrigation. The client has an I.V. of D5W infusing at 40ml/hr and a triple lumen urinary catheter with normal saline solution infusing at 200,l/hr. A nurse empties the urinary catheter drainage bag 3 times during an 8 hr period, for a total of 2780ml. How many milliliters does the nurse calculate as urine? Round to the nearest whole number. ________ ml

RATIONALE: During 8 hrs, 1600ml of bladder irrigation has been infused (200ml x 8hrs = 1600ml/8hrs). The nurse then subtracts this amount of infused bladder irrigation from the total volume in the drainage bag (2780ml - 1600ml = 1180ml) to determine urine output.

A client, with cirrhosis of the liver, develops asciteis. The nurse should expect the physician to write which of the following orders.

RATIONALE: Restrict fluids decreases in the amount fluid present in the body, thus decreasing the amount of fluid, accumulation in the peritoneal space. Other temp. treatments include a restriction of physical activity, a low-sodium diet. And the use of diuretics.

The nurse cares for clients in the emergency department (ED). An 82-year-old client comes to the ED complaining of muscle weakness and drowsiness. The nurse notes decreased deep tendon reflexes and hypotension. Which of the following actions should the nurse take FIRST? 1. Escort the client to an emergency room unit. 2. Ask the client if he has been taking antacids. 3. Assess for Chvostek's sign. 4. Measure client's intake and output

Strategy: "FIRST" indicates priority 1) delegate to other personnel 2) CORRECT— increased intake of magnesium-containing antacids and laxatives can cause hypermagnesemia (> 2.5 mEq/L); depresses CNS and cardiac impulse transmission; discontinue oral Mg, support ventilation, administer loop diuretics or IV calcium, teach about OTC drugs that contain Mg 3) seen with hypocalcemia; tap face just below and anterior to the ear to trigger facial twitching on that side of face 4) renal insufficiency can cause decreased excretion of magnesium; not appropriate for this setting

The nurse in the outpatient clinic has four unscheduled clients waiting to see the physician. Which of the following clients should the nurse see FIRST? 1. A client complaining of a sore throat and nasal drainage. 2. A client with a history of kidney stones complaining of severe flank pain. 3. A client complaining of redness and pain in his left great toe. 4. A client receiving digoxin (Lanoxin) complaining of nausea and vomiting.

Strategy: "FIRST" indicates priority 1) symptoms consistent with viral rhinitis; encourage to gargle with salt water and increase fluid intake 2) second client that should be seen; administer opioid analgesics to prevent shock and syncope 3) indications of acute gout; attack subsides spontaneously in 3 to 4 days; administer colchicine (Colsalide) and NSAIDS 4) CORRECT— early effects of digitalis toxicity; hold medication and monitor client's symptoms

The nurse in the pediatric clinic receives a phone call from the mother of a 3-year-old child. The mother reports that her child has been complaining of a sore throat, has a temperature of 102°F (39°C), and he has suddenly begun drooling. Which of the following suggestions should the nurse make FIRST? 1. "Place a cold water vaporizer in your child's room." 2. "Take your child to the emergency department immediately." 3. "Look into your child's throat and tell me what you see." 4. "Frequently offer your child oral fluids."

Strategy: "FIRST" indicates priority. 1) appropriate action if the child has croup 2) CORRECT— symptoms indicate acute epiglottitis which can be life threatening; drooling occurs because of difficulty swallowing; child may become apprehensive or anxious; transport to hospital sitting in the parent's lap to reduce stress 3) do not inspect the throat unless immediate intubation can be performed if needed 4) transport to the hospital

The nurse on a college campus is informed by the microbiology department that they accidentally received a shipment of highly toxic, contagious bacteria. Which of the following actions should the nurse take FIRST? 1. Determine if there are adequate supplies of antibiotics and antipyretics. 2. Order necessary equipment and supplies. 3. Contact the Red Cross. 4. Identify who was exposed to the shipment.

Strategy: "FIRST" indicates priority. 1) may be required, but not the first action; affected people will most likely be treated in a treatment facility 2) more important to determine who was exposed to the bacteria 3) if exposure is widespread, they may send health care providers; determine scope of problem first 4) CORRECT— assess before implementing; after determining who has been exposed, appropriate treatment can be instituted

The nurse on the medical/surgical unit is approached by an LPN/LVN from a different team. The LPN/LVN expresses concern because one of her patients is diagnosed with COPD and the RN (a new graduate) is giving the patient oxygen at 2 L/min. Which of the following statements by the nurse is MOST appropriate? 1. "I will assess the patient for oxygen toxicity." 2. "Are you concerned about the oxygen or the new graduate's competency?" 3. "Please tell me more about your concerns." 4. "Leave the oxygen in place."

Strategy: "MOST appropriate" indicates discrimination is required to answer the question. 1) client is assigned to another nurse; usurps assigned nurse's authority 2) yes/no question; nontherapeutic; should allow LPN/LVN to express her concerns 3) CORRECT— open-ended statement; therapeutic; allows the LPN/LVN to express specific concerns and enables the nurse to further assess 4) not enough information to make a judgment; assess before implementing

A 25-year-old woman is receiving aminophylline 0.7 mg/kg/h by continuous IV infusion into her left arm. It is MOST important for the nurse to observe her for which of the following? 1. Slowed pulse and reduced blood pressure. 2. Constipation and decreased bowel sounds. 3. Palpitations and nervousness. 4. Difficulty voiding and oliguria.

Strategy: "MOST important" indicates discrimination is required to answer the question. 1) causes rapid pulse and dysrhythmias; decrease intake of colas, coffee, and chocolate because they contain xanthine 2) causes diarrhea, nausea, and vomiting; administer with food or full glass of water 3) CORRECT— effects of aminophylline include nervousness, nausea, dizziness, tachycardia, seizures 4) medication has no effect on the kidneys; encourage intake of 2,000 cc per day to decrease viscosity of airway secretions

A new registered nurse asks the assigned nurse mentor to check on 4 clients who are receiving oxygen therapy. It is MOST important for the nurse mentor to ask the nurse which of the following questions? 1. "Which client should I see first?" 2. "Have you completed your assessment?" 3. "What are your specific concerns?" 4. "Don't you think you should be able to care for the clients?"

Strategy: "MOST important" indicates discrimination may be required to answer the question. 1) nurse mentor should find out about the nurse's specific concerns 2) yes/no question; doesn't allow nurse mentor to assess the nurse's needs 3) CORRECT— clarifies the nurse's concerns and will help the new nurse become a safe practitioner 4) yes/no question; nontherapeutic; does not allow nurse mentor to assess new nurse's concerns

The nurse cares for a client following a scleral buckling. Which of the following nursing actions is MOST important? 1. Remove all reading material. 2. Assess for nausea. 3. Assess drainage from affected eye. 4. Irrigate affected eye every 3 hours.

Strategy: "MOST important" indicates priority. 1) scleral buckling compresses the sclera to repair a detached retina; should take precautions to prevent moving eyes rapidly 2) CORRECT— nausea and vomiting increase intraocular pressure and could cause damage to the area repaired 3) wear eye shield; avoid sneezing, coughing, straining at stool 4) do not irrigate

A patient returns from surgery for a total replacement of the right hip with a large surgical dressing and a Jackson-Pratt drain. Which of the following, if observed by the nurse 2 hours after surgery, necessitates calling the physician? 1. There is a small amount of bloody drainage on the surgical dressing. 2. The patient complains of increased hip pain. 3. A harsh, hollow sound is auscultated over the trachea. 4. The patient's blood pressure is 136/86.

Strategy: "necessitates calling the physician" indicates a complication. 1) expected outcome, complications of total hip replacement include dislocation of prosthesis, excessive wound drainage, thromboembolism, and infection 2) CORRECT— indicates dislocation of prosthesis; other indications include shortening of affected leg, leg rotation, soft popping sound heard when affected leg is moved; maintain abduction, use wedge pillow, avoid stopping, do not sleep on operated side until directed to do so, flex hip only 1/4 circle, never cross legs, avoid position of flexion during sexual activity, walking is excellent exercise, avoid overexertion; in 3 months will be able to resume ADLs, except strenuous sports 3) describes normal breathing sounds 4) within normal limits

The nurse evaluates care for a client who demonstrates manipulative behavior. The nurse should intervene if which of the following is observed? 1. The staff discusses with the client the consequences of his manipulative behavior. 2. The staff establishes limits on the client's manipulative behavior. 3. The staff clarifies the consequences of the client's manipulative behavior. 4. The staff decreases the demands on the client.

Strategy: "nurse should intervene" indicates something is wrong. 1) appropriate that the staff help to client learn to see the consequences of his behavior 2) appropriate; staff should communicate clearly defined expectations and carry out limit-setting 3) appropriate behavior 4) CORRECT— fosters a sense of entitlement

Glomerulonephritis is commonly related to what infection?

Strep. Intervention- prevent and treat strep quickly! Also: Immune dz such as Lupus. Vasculitis, Scarring from: HTN, Diabetic kidney dz. Intervention- control blood sugars and hypertension.

The nurse is reinforcing teaching with a client recently diagnosed with gout. The nurse should encourage the client to minimize the intake of a. alcoholic beverages b. citrus juices c. dairy products d. processed foods

The correct answer is A. A client with gout should minimize the intake of alcoholic beverages that may exacerbate the pain.

The nurse is preparing to administer the measles, mumps, and rubella (MMR) vaccine to a client as prescribed. Which of the following should the nurse have immediately available? a. a latex-free tourniquet b. epinephrine (Adrenaline) 1:1000 c. a commercial cold pack d. methylprednisolone (Solu-Medrol)

The correct answer is B. A client who is receiving injectable medications and vaccines is at risk for an immediate hypersensitivity reaction. The nurse should have epinephrine 1:1000 immediately available to administer to help prevent airway collapse.

The home health nurse is administering an intravenous anti-infective as prescribed to a client with osteomyelitis. Which of the following observations would indicate that the client's peripheral intravenous catheter site needs to be changed before the nurse completes the infusion as prescribed?

The correct answer is B. A client who is receiving medications via a peripheral intravenous site needs to be monitored for indications of infiltration. This is manifested by localized swelling, coolness, pallor and discomfort at the IV site. If an infiltration is present, the infusion is stopped, the catheter is removed and the infusion is restarted at a new site. Blisters under the dressing may indicate reaction to the adhesive but is not an indication that the site needs to be changed.

The home health nurse is checking an adult client with chronic obstructive pulmonary disease (COPD). Which of the following observations would require intervention by the nurse?

The correct answer is B. A client with COPD should have oxygen flowing no higher than 3 liters/minute via nasal cannula to help maintain the respiratory drive. Clubbing of the nail beds is a normal finding for a client with COPD.

The nurse is assessing a 72-year-old male client before the client's annual physical examination. The nurse should understand that which of the following changes is a normal part of the aging process? a. increased rapid eye movement (REM) sleep b. decreased bladder capacity c. increased difficulty remembering events d. decreased muscle tone

The correct answer is B. The normal aging process causes a decrease in the bladder capacity. Memory loss is not considered a normal part of the aging process.

The nurse is assessing a client with bacterial pneumonia. The client has been receiving 0.9% normal saline and an anti-infective via IV piggyback as prescribed for 24 hours. The nurse must intervene if the client's a. oxygen saturation level via pulse oximetry is 92% b. cough is productive of green sputum c. peripheral IV site is cool to touch d. skin is pale

The correct answer is C. A client with bacterial pneumonia who is receiving fluids and medications via IV should be monitored for complications. A cool IV site would require intervention since this indicates an infiltration. The client's sputum color is expected based on the client's diagnosis and length of treatment.

The home health nurse is providing care to an active 2-year-old client who has a tracheostomy. Which of the following actions by the nurse would increase the risk for a charge of negligence?

The correct answer is C. The nurse must remain aware of standards of care for clients with a tracheostomy and take action to minimize the risk for injury to the client. Negligence is defined as failure to provide care that meets the standard and that places the client at risk for injury. Velcro ties should not be used to secure the tracheostomy tube for a 2-year-old client since the client can easily reach and manipulate the Velcro. The neck should be flexed prior to securing the square knot to help ensure adequate room for expansion of the neck during movement.

The nurse is assessing a client who is taking prescribed levothyroxine (Synthroid) and who has a low thyroid-stimulating hormone (TSH) level. The nurse should assess the client's a. nails for thickening b. skin for breakdown c. neck veins for distention d. pedal pulses for symmetry

The correct answer is C. The nurse should understand common manifestations of thyroid disease and risks for a client with thyroid disease. A client who is on Synthroid is at risk for hyperthyroidism. A low TSH is an indication of excess Synthroid and the client is at risk for dysrhythmias and heart failure. The nurse should assess the client's neck veins for distention.

The nurse is conducting a staff development conference regarding death and dying. The nurse should indicate that a school-aged child typically views death as a. a deep sleep b. punishment c. a part of mortality d. final

The correct answer is D. The school-aged child typically views death as final.

4. nurse researcher

The nurses on an acute care medical floor notice an increase in pressure ulcer formation in their patients. A nurse consultant decides to compare two types of treatment. The first the procedure currently used to assess for pressure ulcer risk. The second uses a new assessment instrument to identify at-risk patients. Given this information, the nurse consultant exemplifies which career? 1. clinical nurse specialist 2. nurse administer 3. nurse educator 4. nurse researcher

4. The professional nurse works with colleagues and the patient's family to provide combined expertise in planning care.

The statement that best explains the role of collaboration with others for the patient's plan of care is which of the following? 1. The professional nurse consults the health care provider for direction in establishing goals for patients. 2. The professional nurse depends on the latest literature to complete an excellent plan of care for patients. 3. The professional nurse works independently to plan and deliver care and does not depend on other staff for assistance. 4. The professional nurse works with colleagues and the patient's family to provide combined expertise in planning care.

Cholinergic Crisis

Too much medication N&V, diarrhea, cramps hypotension Tensilon to distinguish from Myasthenia Crisis Atropine to reverse Respiratory support may be needed

4. Talk with him about his favorite hobbies

You are caring for Mr. Smith, who is facing amputation of his leg. During the orientation phase of the relationship, what would you do? 1. Summarize what you have talked about in the previous sessions 2. Review his medical record and talk to other nurses about how he is reacting 3. Explore his feelings about losing his leg 4. Talk with him about his favorite hobbies

3. Move to her bedside, get her attention, and repeat the question while facing her

You are caring for an 80-year old woman, and you ask her a question while you are across the room washing your hands. She does not answer. What is your next action? 1. Leave the room quietly since she evidently does not want to be bothered right now 2. Repeat the question in a loud voice, speaking very slowly 3. Move to her bedside, get her attention, and repeat the question while facing her 4. Bring her a communication board so she can express her needs

4. Defines the principles of right and wrong to provide patient care.

You are participating in a clinical care coordination conference for a patient with terminal cancer. You talk with your colleagues about using the nursing code of ethics for professional registered nurses to guide care decisions. A nonnursing colleague asks about this code. Which of the following statements best describes this code? 1. Improves self-health care 2. Protects the patient's confidentiality 3. Ensures identical care to all patients 4. Defines the principles of right and wrong to provide patient care.

Standard treatment for lithium toxicity is administration of...

normal saline.

A chancre lesions associated with syphilis is...

not painful.

signs of meningeal irritation

nuchal rigidity-stiff neck kernig's sign-when hip flexed to 90 degree complete extension of the knee is restricted and painful. brudzinski's sign-attempts to flex the neck will produce flexion at knee and thigh opisthotonic position-hyperextend of legs and arch of back

para

number of past pregnancies that have gone beyond the period of viability (20 wks gestation or > 500 g)

Clinical manifestations of stroke include...

numbness or weakness on one side of the body; dizziness and loss of balance or coordination; sudden headache (more with hemorrhagic strokes); visual or perception changes; aphasia; apraxia; dysarthria; dysphagia

Method used to elicit tricep reflex

nurse loosely suspends the client's arm in an open hand while tapping the back of the client's elbow

Method used to elicit biceps reflex

nurse places thumb on the muscle inset in the antecubital space and taps the thumb briskly with the reflex hammer

Treatment for physiologic jaundice (hemolytic anemia)

phototherapy with fluorescent lighting- alters nature of bilirubin to aid excretion; eyes and genitals covered

Mechanical obstruction:

physical blockage of lumen of intestine usually seen in the small intestine. hernia, tumors, adhesions, strictures d/t radiation, intussusception, volvus ( twisting of bowel)

Copper (paragard)

recommended for women who had at least 1 child; can be USED FOR 10 yrs -damage to sperm in transit to fallopian tube

s/s of infection:

redness, fever, pain, increased WBC

MG: MYASTHENIA GRAVIS : S/S Complication Crisis

s/s: Risk complications: Swallowing/Arrest Ptosis, diplopia Weakness, dysarthria, dysphagia, difficulty sitting up, Respiratory distress Eye and periorbital muscles most affected- manifested by diplopia, ptosis, ocular palsies Sx least evident in the AM and most evident w/effort as the day proceeds Crisis: Sudden exacerbation of motor weakness putting client at risk for respiratory failure and aspiration: Pneumonia & MYASTHENIC CRISIS: Respiratory, and swallowing muscles too weak. Risk complications: Swallowing/Arrest THYMOMA-A rare neoplasm, usually found in the anterior mediastinum and originating in the epithelial cells of the thymus. Assess an maintain respiratory, swallowing, atelectasis. CHOLINERGIC CRISIS: Flaccid paralysis, respiratory failure, GI symptoms, severe muscle weakness, vertigo. Tx: Atropine

ALS: AMYOTROPHIC LATERAL SCLEROSIS : S/S Complication Crisis

s/s: Weakness, dysarthria, dysphagia No loss of cognitive function Complications/Crisis: respiratory failure. Assess Respiratory function: ABC's, clear lungs. Swallowing: proper food choices & eventual NG tube. Mobility. Skin. Suctioning: difficult chewing, swallowing, drooling, choking. Communication.

TRIGEMINAL NEURALGIA : S/S Complication Crisis

s/s: facial pain. Severe facial pain occurring for brief seconds to mins hundreds of x/day, several x/yryear Usually occurs unilaterally in area of mouth and rises toward ear &eye Triggers: areas on the face may initiate the pain- eating, swallowing, talking Often there is spontaneous remission after years, and then condition recurs with dull ache in between pain episodes. Assess/monitor pts trigger factors, complications of pain, nutrition, hygiene, oral care, anxiety.

Negative symptoms of schizophrenia are those such as...

social withdrawal and failure to communicate with others.

Hegar sign

softening and compressibility of isthmus of uterus

Goodell's sign

softening of the cervix

Hegar's sign

softening of the portion of the cervix between the uterus and vaginal portion of the cervix

While taking Bactrim what side effect would be a concern:

sore throat

The female with histrionic personality disorder is...

southern belle syndrome; overly seductive, excitable, immature, and theatrical in her emotions; forms shallow, short-lived relationships

Carbidopa/levodopa toxicity symptoms

spasmodic eye winking, involuntary twitching of muscles, facial grimaces, severe tongue protrusion

Position assumed by children with congenital heart disease; tetralogy of fallot to relieve hypoxia

squatting position with arms wrapped around the legs

65+ yrs

(Late Adulthood) Integrity V despair

35-65 yrs

(Middle Adulthood) Generativity V stagnation

1 year to 3 yr

(Toddler) Autonomy V Shame & doubt

Ptosis:

drooping of eyelids

Addison's Disease: deficiency in mineralocorticoid secretions

(aldosterone) > increased sodium excretion> dehydrations> hypotension> decreased CO and decrease heart size.

Addison's Disease: deficiency in glucocorticoid secretions

(cortisol) > decrease in gluconeogensis> hypoglycemic and liver glycogen deficiency. Failure to inhibit ACTH and melanocyte-stimulating hormone> increased level of ACTH and hyperpigmentation.

Probable signs of pregnancy

(examiners objective findings) Positive pregnancy test, enlarge abdomen uterus, Gooddells signs, chadwick, hegars signs, ballottement, braxtons hicks contractions

birth to 1 yr

(infancy) Trust V Mistrust

Solitary Play

(infancy) birth to 1 yr alone but enjoys presence of others, interest centered on own activity

Narcissistic personality disorder is summarized by the expression

"It's all about me." Grandiose ideas about their wealth, power, and intelligence; appear nonchalant; rationalize or blame others for their self-centered behavior

Mnemonic for Cholinergic Crisis Symptoms: "SLUDGE"

"SLUDGE" S- SALIVATION L- LACRIMATION (tearing) U- URINATION D- DEFECATION G- GASTRIC UPSET E- EMESIS

Gravida

# of pregnancy

IICP: What is the most common sign of Increased Intracranial Preasure?

#1 sign = Mental status changes indicate brain cells are deprived of O2. Changes may be subtle: - flat affect - change in pts orientation (ie: pt is no longer A&Ox3) - decreased level of attention

12-20 yrs

(Adolescence) Identity V role diffusion

20-35yrs

(Adulthood) Intimacy V Isolation

3-6 yrs

(preschool) Initiative V Guilt

Associative play

(preschool) 3 to 6 yrs No group goal, often follows a leader

Cooperative play

(school age) 6-12 yrs organized rules, leader/follower relationship

6-12yrs

(school-age) industry V inferiority

Parallel PLay

(toddlers) 1-3 yrs plays alongside, not with another characteristic of toddlers

Bowel Obstruction: Dietary modification

**NPO** to prevent obstruction to progress to bowel perforation. TPN & Bowel rest until obstruction resolved.

What does RN do if hematoma develops

- elevate - cool compress if blood is new - warm compress if blood is old

What safety intervention should a nurse ensure for a pt identified as needing Seizure Precautions?

- padded side rails; - all four up to prevent falling out of bed --> during a seizure

Positive Kernig's sign:

- positive when the leg is fully bent in the hip and knee, and subsequent extension in the knee is painful (leading to resistance)

Nuchal rigidity:

- the inability to flex the head forward due to rigidity of the neck muscles. **If flexion of the neck is painful, but full range of motion is present, nuchal rigidity is absent**

HHNKS: Assessment

-Glucose level of >800 md/dl -dry mucous, poor skin turgor -ketosis and acidosis do NOT occur here

The home health nurse is talking with family members about the nutritional needs of a client who is receiving mechanical ventilation. Which of the following statements would be correct for the nurse to make?

. A client on mechanical ventilation is at risk for imbalanced nutrition. The client needs to be on a diet that has increased protein, calories, vitamins and minerals. However, a high carbohydrate intake can cause increased carbon dioxide production and result in a low oxygen saturation level.

Hyperthyroidism Labs

...

ICP: Intracranial Pressure

...

alkaline-ash diet consists of:

...

dysarthria

...

dysplagia

...

Glucose tolerance test is given:

1 h test at 24-28 wks

1 gm of peripads

1 ml of blood loss

Following hydatidiform mole the client should be instructed not to become pregnant for at least...

1 year because rising HCG level will stimulate cancer cell growth.

triple in weight should be at

1 yr

Common Preoperative Medications:

1) Benzodiazepines and barbiturates: for sedation and amnesia 2) Anticholinergics: to reduce secretions 3) Opioids: to decrease intraoperative anesthetic requirements and pain 4) Additional drugs include antiemetics, antibiotics, eye drops, and regular prescription drugs

First stage of labor

1-10 cm dilation

Guillain-Barre frequently develops...

1-3 weeks following an upper respiratory or GI infection

Permanent tooth eruption is complete by...

12 yeras of age.

Ovulation usually occurs...

12-18 days after the first day of the menstrual cycle; client's temperature decreases and then sharply rises at the time of ovulation

Fetal heart tones can be heard with a fetoscope at approximately...

18-20 weeks

BSE

18-20 yrs repeat monthly

What are some common Nursing Interventions to reduce risk/avoid post-op complications?

1. Turn & reposition the pt to promote circulation and reduce the risk of skin breakdown, especially over boney prominences. Initially position pt in a Lateral recumbant position until arousal from anesthesia, then position pt in Semi or Fowler position to reduce breathing effort. 2. Encourage coughing & deep breathing. This helps clear anestetics from the body, lowers risk of pulmonary/fat emboli, and hypostatic pneumonia associated with secretion buildup in the airways. 3. Encourage use of Incentive Spirometer. 4. Monitor In's and Out's. Hydration and protein rich nutrition promotes healing and provides energy to meet the needs of the pts increased metabolism associated with surgery. 5. Promote early ambulation. Early post-op exercise and ambulation significantly reduce the risk of thromboembolism.

Therapeutic Levels of Lithium

1.0-1.5

normal specific gravity

1.010-1.020

Mg+

1.3-2.1

Gastric ulcer Pain:

1/2-1 hrs pc meal; relieved by vomiting, food does not help

FM

10 movement in 20 minutes to 2 hrs twice a day is good indicator. > 3 movement in 1 hour should be reported

Normal FHR

120-160

newborn pulse rate

120-160

Normal FHR

120-160 must obtain Baseline

Blood pressure when pregnany

140/90

Alpha-fetoprotein levels can be done on mother's blood between...

16 and 20 weeks gestation.

A toddler's head circumference is about....

19 inches by 15 months and about 20 inches by 2 years of age. Brain growth increases to 90% of adult size.

anterior fontanelle close

1yr- 18 months

The toddler is approximately one-half his adult height by age...

2 1/2 years.

Between ages of 6 and 12, children grow an average of...

2 inches per year; by 12 should be approximately 5 feet tall

45. The nurse is providing care for a 68-year-old woman who is complaining of constipation. What concern exists regarding her nutritional status?

A) The absorption of nutrients may be impaired.

Inferior

2,3 avf

Duodenal ulcer Pain

2-3 hrs pc; food intake relieves pain

Signs of withdrawal related to stimulant abuse...

agitation, disorientation, insomnia, depression, suicidal ideation

To prevent isoimmunization, the mother should be given Rhogam during pregnancy as early as...

20 weeks or postpartally within the first 48-72 hours

CK

20-200

tympany

air filled structures (stomach)

HCO3

22-26

weight gain during pregnancy

24-28 lbs first trimester 2-4 second 12-14 third 8-12

Narcotics withdrawal seen in infants occur

24-72 hrs

Cholinergic Crisis

Caused by overmedication with anticholinesterase. Treatment: hold medication and give atropine if ordered.

25. A client with rapid rate atrial fibrillation asks a nurse why the physician is going to perform carotid sinus massage. Which of the following would be reflective of a correct explanation provided by the nurse? 1. The vagus nerve slows the heart rate. 2. The diaphragmatic nerve slows the heart rate. 3. The diaphragmatic nerve overdrives the rhythm. 4. The vagus nerve increases the heart rate, overdriving the rhythm.

25. 1 Rationale: Carotid sinus massage is one maneuver used for vagal stimulation to decrease a rapid heart rate and possibly terminate a tachydysrhythmia. The others include inducing the gag reflex and asking the client to strain or bear down. Medication therapy often is needed as an adjunct to keep the rate down or maintain the normal rhythm. Options 2, 3, and 4 are incorrect descriptions of this procedure. Test-Taking Strategy Knowledge of anatomy and physiology alone may be sufficient to answer this question. Eliminate options 3 and 4 because a rapid rate dysrhythmia would need to be slowed. Recalling the functions of the vagus nerve and the diaphragmatic nerve will direct you to option 1. The vagus nerve affects heart rate. The diaphragmatic nerve affects respiration. If you are unfamiliar with the functions of these nerves, review this content.

Infant stage

28 days to 1 year

When the L/S ration reaches...

2:1, the lungs are considered to be mature.

Ventricular Tach

3 or more PVC occuring at 100 bpm

Habitual

3 or more spontaneous abortion NC: cerclage ( encircling cerivx with suture)

GCS total

3-15

Complication of spinal shock can occur with a client with a SCI and can last for...

3-6 weeks

Sty

Caused: Staphylococcal Warm compress, Ab, I&D

K+

3.5-4.5

newborn respiratory rate

30-60 per min

30. A nurse is evaluating a client's response to cardioversion. Which of the following observations would be of highest priority to the nurse? 1. Blood pressure 2. Status of airway 3. Oxygen flow rate 4. Level of consciousness

30. 2 Rationale: Nursing responsibilities after cardioversion include maintenance first of a patent airway, and then oxygen administration, assessment of vital signs and level of consciousness, and dysrhythmia detection. Test Taking Strategy Use the process of elimination, noting the strategic words highest priority. Use the ABCs—airway, breathing, and circulation—to direct you to option 2. Review care of the client following cardioversion if you had difficulty with this question.

if UTI is suspected how many mls of fluid should the RN encourage daily?

3000 ml unless contraindicated ie CHF pt

mammogram test

35-39 annually

PCO2

35-45

During the prenatal period, a weight gain of approximately...

36 pounds is allowable, and weight reduction is discouraged.

Preterm is a delivery that occurs prior to...

37 weeks gestation.

703. A client is having frequent premature ventricular contractions. A nurse would place priority on assessment of which of the following? 1. Sensation of palpitations 2. Causative factors, such as caffeine 3. Precipitating factors, such as infection 4. Blood pressure and oxygen saturation

4 Rationale: Premature ventricular contractions can cause hemodynamic compromise. The shortened ventricular filling time with the ectopic beat leads to decreased stroke volume and, if frequent enough, to decreased cardiac output. The client may be asymptomatic or may feel palpitations. Premature ventricular contractions can be caused by cardiac disorders, states of hypoxemia, or by any number of physiological stressors, such as infection, illness, surgery, or trauma, and by intake of caffeine, nicotine, or alcohol. Test-Taking Strategy: Note the strategic words priority on assessment. Use the ABCs—airway, breathing, and circulation—to direct you to option 4. Review the effects of premature ventricular contractions if you had difficulty with this question.

associative play is at what age

4 yo

Glycosylated hemoglobin (HbA1c): Normal Range (non fasting test)

4-6%

6. The nurse is performing a health interview on a patient who has a language barrier, and no interpreter is available. Which is the best example of an appropriate question for the nurse to ask in this situation?

A) "Do you take medicine?"

4. Automobile accidents, suicide, and substance abuse

A couple is with their adolescent daughter for a school physical and state they are worried about all the safety risks affecting this age. What is the greatest risk for injury for an adolescent? 1. Home accidents 2. Physiological changes of aging 3. Poisoning and child abduction 4. Automobile accidents, suicide, and substance abuse

Chalazion (inflammatory cyst)

Caused: duct obstruction I&D

Therapeutic level magnesium sulfate

4.8-9.6 mg/dl

AV node bpm

40-60

43. A nurse is assessing a client with an abdominal aortic aneurysm. Which of the following assessment findings by the nurse is probably unrelated to the aneurysm? 1. Pulsatile abdominal mass 2. Hyperactive bowel sounds in the area 3. Systolic bruit over the area of the mass 4. Subjective sensation of "heart beating" in the abdomen

43. 2 Rationale: Not all clients with abdominal aortic aneurysm exhibit symptoms. Those who do may describe a feeling of the "heart beating" in the abdomen when supine or being able to feel the mass throbbing. A pulsatile mass may be palpated in the middle and upper abdomen. A systolic bruit may be auscultated over the mass. Hyperactive bowel sounds are not related specifically to an abdominal aortic aneurysm. Test-Taking Strategy Use the process of elimination and note the strategic word unrelated. Note that options 1, 3, and 4 are comparative or alike in that they identify a circulatory component. Review the signs of abdominal aortic aneurysm if you had difficulty with this question.

45. Cardiac magnetic resonance imaging (MRI) is prescribed for a client. The nurse identifies that which of the following is a contraindication for performance of this diagnostic study? 1. Client has a pacemaker. 2. Client is allergic to iodine. 3. Client has diabetes mellitus. 4. Client has a biological porcine valve.

45. 1 Rationale: The magnetic fields used for magnetic resonance imaging (MRI) can deactivate the pacemaker. Options 2, 3, and 4 are not contraindications for an MRI. Test-Taking Strategy Focus on the name of the test and note the strategic word magnetic. Remember that the magnetic fields of the MRI can deactivate the pacemaker. Review the contraindications for an MRI if you had difficulty with this question.

How long can breast milk last

48 hrs refrigerated 2 wks in a refrigerator freezer 2 months for deep freezer

hearing and vision test

4yr, 5 yr then annually

Average weight gain is about...

5 pounds per year; 3 year old about 37 pounds; 4 year old 42 pounds; 5 year old 47 pounds

milk production requires and increase in_____ calories

500

Colostomy irrigations for sigmoid colostomy: procedure

500-1500 ml luke warm water, insert catheter 8 cm (3 inch), hang irrigating container at shoulder height (18-20 inch above stoma); water flow in 5-10 min, wait 10-15 min to allow return. leave sleeve on for 30-45 min. clean with soap and water

SA node bpm

60-100

ph

7.35-7.45

PO2

80-100

FBS for pregnancy

80-100mg/dl

pincer grasp is at what age

9 months

9. A client who has developed severe pulmonary edema would most likely exhibit which of the following? 1. Mild anxiety 2. Slight anxiety 3. Extreme anxiety 4. Moderate anxiety

9. 3 Rationale: Pulmonary edema causes the client to be extremely agitated and anxious. The client may complain of a sense of drowning, suffocation, or smothering. Test-Taking Strategy Use the process of elimination. Noting the strategic word severe will direct you to option 3. Review the clinical manifestations associated with severe pulmonary edema if you had difficulty with this question.

Cl normal range

95-105

Temperatre

98.6

Preeclampsia plan implementation

< 6gram of sodium, report 3 lbs weight gain over 24 hrs (magnesium sulfate, hydralazine (apresoline) Valium, procardia

Uveitis (inflammation of iris, ciliary body, choroid)

Caused: local or systemic infection Warm Compress, Dark glasses, Ab, analgesics, sedatives

1. A safe environment promotes patient activity.

A 62-year old woman is being discharged home with her husband after surgery for a hip fracture from a fall at home. When providing discharge teaching about home safety to his patient and her husband, the nurse knows that: 1. A safe environment promotes patient activity. 2. Assessment focuses on environmental factors only. 3. Teaching home safety is difficult to do in the hospital setting. 4. Most accidents in the older adult are caused by lifestyle factors.

4. Clear the area around the child to protect child from injury.

A child in the hospital starts to have a grand mal seizure while playing in the playroom. What is your most important nursing intervention in this situation? 1. Begin cardiopulmonary respiration. 2. Restrain the child to prevent injury. 3. Place a tongue blade over the tongue to prevent aspiration. 4. Clear the area around the child to protect child from injury.

2. "Tell me what bothers you the most about this experience."

A child who has been in a house fire comes to the emergency department with her parents. The child and parents are upset and tearful. During the nurse's first assessment for stress the nurse says: 1. "Tell me who I can call to help you." 2. "Tell me what bothers you the most about this experience." 3. "I'll contact someone who can help get you temporary housing." 4. "I'll sit with you until other family members can come help you get settled."

2. After 6 weeks when she adjusts to the child's respiratory status and reestablishes the entitlement checks

A crisis intervention nurse working with a mother whose Down syndrome child has been hospitalized with pneumonia and who has lost her entitlement check while the child is hospitalized can expect the mother to regain the stability after how long? 1. After 2 weeks when the child's pneumonia begins to improve 2. After 6 weeks when she adjusts to the child's respiratory status and reestablishes the entitlement checks 3. After 1 month when the child goes home and the mother gets help from a food pantry 4. After 6 months when the child is back in school

Keratitis (inflammation of cornea)

Caused: virus, spread of systemic dz Ab, hot compress, steroids

4. Psychomotor domain

A patient needs to learn to use a walker. Which domain is required for learning this skill? 1. Affective domain 2. Cognitive domain 3. Attentional domain 4. Psychomotor domain

1. Telling approach

A patient with chest pain is having an emergency cardiac catheterization. Which teaching approach does the nurse use in this situation? 1. Telling approach 2. Selling approach 3. Entrusting approach 4. Participating approach

A nurse is assessing a pt who has been identified as a seizure risk. When assessing the pt risk factors, the nurse assesses for status epilepticus. What is SE?

A state of continuous seizure activity, or a condition in which seizures recur in rapid succession, without return to consciousness between seizures.

124. The nurse knows that determining whether a person is oriented to his or her surroundings will test the functioning of which of these structures? A) Cerebrum B) Cerebellum C) Cranial nerves D) Medulla oblongata

A) Cerebrum Pages: 621-622 | Page: 660. The cerebral cortex is responsible for thought, memory, reasoning, sensation, and voluntary movement. The other options structures are not responsible for a person's level of consciousness.

101. The nurse is percussing the seventh right intercostal space at the midclavicular line over the liver. Which sound should the nurse expect to hear? A) Dullness B) Tympany C) Resonance D) Hyperresonance

A) Dullness Page: 541. The liver is located in the right upper quadrant and would elicit a dull percussion note.

The telemetry unit nurse is reviewing laboratory results for a patient who is scheduled for an operative procedure later in the day. The nurse notes on the laboratory report that the patient has a serum potassium level of 6.5 mEq/L, indicative of hyperkalemia. The nurse informs the physician of this laboratory result because the nurse recognizes hyperkalemia increases the patient's operative risk for:

A) Hyper/hypokalemia increases the patient's risk for cardiac problems. A decrease in the hematocrit and hemoglobin level may indicate the presence of anemia or bleeding. An elevated WBC occurs in the presence of infection. Abnormal urine constituents may indicate infection or fluid imbalances.

Upon assessment, a patient reports that he drinks 5-6 bottles of beer every evening after work. Based upon this information, the nurse is aware that the patient may require:

A) Patients with a larger habitual intake of alcohol require larger doses of anesthetic agents and postoperative analgesics, increasing the risk for drug-related complications.

Clients receiving magnesium sulfate should have a...

Foley catheter inserted to monitor output hourly; should be assessed for hypotension and respiratory distress

The nurse is providing teaching to a patient regarding pain control after surgery. The nurse informs the patient that the best time to request pain medication is:

A) The question states that the patient is being instructed on when to "request" pain medication. If a pain medication is ordered PRN, the patient should be instructed to ask for the medication before the pain becomes severe.

The nurse is preparing to send a patient to the operating room for an exploratory laparoscopy. The nurse recognizes that there is no informed consent for the procedure on the patient's chart. The nurse informs the physician who is performing the procedure. The physician asks the nurse to obtain the informed consent signature from the patient. The nurse's best action to the physician's request is to:

A) The responsibility for securing informed consent from the patient lies with the person who will perform the procedure. The nurse's best action is to inform the physician that it is his responsibility to obtain the signature.

120. During the assessment of an 80-year-old patient, the nurse notices that his hands show tremors when he reaches for something and his head is always nodding. There is no associated rigidity with movement. Which of these statements is most accurate? A) These are normal findings resulting from aging. B) These could be related to hyperthyroidism. C) These are the result of Parkinson disease. D) This patient should be evaluated for a cerebellar lesion.

A) These are normal findings resulting from aging. Page: 659. Senile tremors occasionally occur. These benign tremors include an intention tremor of the hands, head nodding (as if saying yes or no), and tongue protrusion. Tremors associated with Parkinson disease include rigidity, slowness, and weakness of voluntary movement. The other responses are incorrect.

A client who is started on metformin and glyburide would have initially present with with symptoms?

A) symptoms of hyperglycemia included polydispia, polyuria, and weightloss. Metformin and sulfonylureas are commonly ordered medications. Weight gain, tiredness, and bradycardia are symptoms of hypothyroidism. Irritability, diaphoresis, and tachycardia are symptoms of hypoglycemia. Symptoms of Crohn's disease include diarrhea, abdominal pain, and weight loss.

Select all that apply. Advantages of laser surgery include diminished

A, B, C, D, & E (All of the above) Laser surgery offers the benefits of diminished bleeding, swelling, tissue damage, and postoperative pain and infection.

What classification is Pepcid and what does it treat?

H2 antagonist (blocks histamine receptor) Treats dyspepsia, GERD, PUD, esohagitits

The nurse has given the client with Bell's Palsy instructions on preserving muscle tone in the face and preventing denervation. The nurse determines that the client needs additional information if the client states that he or she will:

A- Expose the face to cold and drafts

The client with a stroke has residual dysphagia. When the diet order is initiated, the nurse avoids doing which of the following?

A- Giving the client thin liquids Rationale: before the client with dyshagia is started on a diet, the gag and swallow reflexes must have returned. The client is assisted with meals as needed and is given ample time to chew and swallow. Food is placed on the unaffected side of the mouth. Liquids are thickened to avoid aspiration.

physiologic jaundice

caused by immature hepatic function, resolving cephalohematoma; last 5-7 days no tx necessary

breast milk jaundice (early onset)

caused by poor milk intake; tx is frequent breastfeeding

Syphilis

primary lesion (chancre) located on internal or external genitalia

Precipitous labor: plan

ritodrine [Yutopar], terbutaline [ brethinel]

INTRACRANIAL TUMORS : disease discription

CNS Abresses, tumors of brain and spinal cord due to: Brain Trauma Extradural brain abscesses are related with osteomyelitis- following mastoidititis, sinusitis or even sinusitis surgery

Bowel obstruction : disease discription

Blockage in small intestine or colon that prevents food and fluid from passing through. Tissue death & perforation of intestine can lead to severe infection and shock. Can be life-threatening.

Lochia alba

white or clear discharge, can last several weeks following delivery

Acute PTSD occurs...

within 6 months of the event.

What rehab exercises differ for client with homonymous hemianopsia during Acute versus the Rehab phase? Where would you place this pts plate of food?

Acute Phase: Put plate of food in pt's line of vision. Rehab Phase: Help pt to retrain their eye. Put plate of food on table in front of them, but out of line of vision. Teach family to remind pt to turn head to scan the lost visual field.

Delayed PTSD occurs...

within 6 months or more after the event.

Phase 2 labor

active labor (4-7 cm dilation)

Physical Assessment of Renal System- AUSCULTATION:

AUSCULTATION: With a stethoscope the abdominal aorta and renal arteries are auscultated for a bruit (an abnormal murmur), which indicates impaired blood flow to the kidneys.

Treatment of HIV/AIDS during pregnancy

AZT, Zidovudine, non-nucleoside reverse transcriptase inhibitors, protease inhibitors, HAART, bactrim for PCP

What is decorticate posturing?

Abnormal flexion: Hands pulled to chest and hyper-extended. Internal rotation and adduction of the arms with flexion of the elbows, wrists & fingers. "flexor - toward the cord"

Glaucoma:Diagnosis

Abnormal increase in intraocular pressure leading to visual disability and blindness; obstruction of outflow of aqueous humor

What is the high and low score of a Glasgow Coma Scale indicate?

Above 15 = Non-neurological impaired 8 or less = Coma 3 or less is = likely brain death

CNVIII

Acoustic (Vestibulocochlear); hearing and equilibrium

Trigeminal Neuralgia (Tic Douloureux): Nursing consideration

Administer: Carbamazepine (tegretol) Chew on opposite side, avoid eye rubbing

Parkinsons Disease: Tx

Adminsiter dpaminergics (sinemet) dopamine agonist (pramipexole) Antocholnergic (benzotropine treheyphenidyl) Antiviral *amantaddine)

1. Function independently

Advanced practice registered nurses generally: 1. Function independently 2. Function as unit directors 3. Work in acute care settings 4. Work in the university setting.

Clients with schizophrenia are best known for their odd appearance and behavior, summarized by the 4 A's

Affect--described as flat, blunted, or inappropriate; autism--preoccupation with self and a retreat into fantasy; association--loosely joined unrelated topics; ambivalence--having simultaneous opposing feelings

3. "Would you like for me to sit down with you for a few minutes so you can talk about this?"

After a health care provider has informed a patient that he has colon cancer, the nurse enters the room to find the patient gazing out the window in thought. The nurse's first response is which of the following? 1. "Don't be sad. People live with cancer every day." 2. "Have you thought about how you are going to tell your family?" 3. "Would you like for me to sit down with you for a few minutes so you can talk about this?" 4. "I know another patient whose colon cancer was cured by surgery."

When doing a corneal reflex, we need to remember what key factor?

Age slows down the reflex. Contact lenses can affect the corneal reflex as well.

Parkison-like effect

Akinesia, changes in gain, dyskinesia MEDS: ABC (artane,

Biophosphonates: Names

Alendronate (fosamax) Risedronae (Actonel) Ibandronate (boniva)

3. Assessment

An 18-year-old woman is in the emergency department with fever and cough. The nurse obtains her vital signs, auscultates her lung sounds, listens to her heart sounds, determines her level of comfort, and collects blood and sputum samples for analysis. Which standard of practice is performed? 1. Diagnosis 2. Evaluation 3. Assessment 4. Implementation

Compazine is a(an)______ and results in _______

An antiemetic and is calming and sedating (putting pt at risk for aspiration)

4. Allows the patient time to express himself or herself and ask questions

And older adult is being started on a new antihypertensive medication. In teaching the patient about the medication, the nurse: 1. Speaks loudly 2. Presents the information once 3. Expects the patient to understand the information quickly 4. Allows the patient time to express himself or herself and ask questions

Librium

AntiAnxiety (benzodiazepine): Cigarette smoking increase clearance of drug, alcohol increases CNS depression.

Anticholinergic

Anticholinergics are a class of medications that inhibit parasympathetic

APGAR

Appearance: 0=blue pale 1=body pink extremities blue 2=completely pink Pulse: 0=absent 1=<100 2=>100 Grimace: 0=no response 1= weak cry grimace 2=cry vigorous Activity: 0=flaccid 1=some flexion or extremities 2=active motion general flexion Respiratory effort: 0=absent 1=slow and irregular 2= good strong loud cry

The nurse has given instructions to the client with Parkinson's disease about maintaining mobility. The nurse determines that the client understands the directions if the client sates that he or she will:

C- Rock back and forth to start movement with bradykinesia

Kawasaki Dz Side effect

Aspirin is used to treat the inflammation and fever. Tinnitus and ringing buzzing in the ear is a side effect

IICP: What are some expected changes that may be seen when assessing pts w/ compression of oculomotor nerves as a result of IICP?

Assess PERRLA, extraocular movements: - dilation of pupil on the same side as the mass lesion, - sluggish or no response to light - inability to move the eye upward - ptosis (drooping) of the eyelid

Reyes Syndrome:

acute metabolic encephalopathy of childhood that causes hepatic dysfunction with links of aspirin with viral illness.

Bone Marrow Depression

Assessment: Anemia (low Hct), Thrombocytopenia, Leukopenia. NC: Monitor CBC, Protect from infection and injury

1. Prepare for an influx of patients

At 3 am the emergency department nurse hears that a tornado hit the east side of town. What action does the nurse take first? 1. Prepare for an influx of patients 2. Contract the American Red Cross 3. Determine how to restore essential services 4. Evacuate patients per the disaster plan

what is the antidote for a cholenergic medication?

Atropine

Meds to Tx Sinus Brady

Atropine .5mg IV

speed shock

caused by rush of IV fluid administered; med races to blood-rich heart and brain and floods them w/toxic levels of med

Glaucoma:Implementation

Avoid: atropine. heavy lifting straining at stool.

39. When performing a physical examination, safety must be considered to protect the examiner and the patient against the spread of infection. Which of these statements describes the most appropriate action the nurse should take when performing a physical examination?

B) Wash hands before and after every physical patient encounter.

Physical Assessment of Renal System- CREATININE CLEARANCE:

CREATININE CLEARANCE: Because almost all creatinine in the blood is normally excreted by the kidneys, creatinine clearance is the most accurate indicator of renal function. The result of a creatinine clearance test closely approximates that of the GFR.

A pediatric nurse is preparing a child for cleft palate repair surgery. The nurse recognizes that this type of surgery is categorized as:

B) Cleft palate repair considered constructive surgery because the goal is to restore function in congenital anomalies. Reconstructive surgery serves to restore function to traumatized or malfunctioning tissues and includes plastic surgery or skin grafting. Transplant surgery replaces organs or structures that are diseased or malfunctioning, such as a liver or kidney transplant. Palliative surgery is not a curative and seeks to relieve or reduce the intensity of an illness, such as debridement or necrotic tissue.

123. During an assessment of a 22-year-old woman who has a head injury from a car accident 4 hours ago, the nurse notices the following change: pupils were equal, but now the right pupil is fully dilated and nonreactive, left pupil is 4 mm and reacts to light. What does finding this suggest? A) Injury to the right eye B) Increased intracranial pressure C) Test was not performed accurately D) Normal response after a head injury

B) Increased intracranial pressure Pages: 662-663. In a brain-injured person, a sudden, unilateral, dilated, and nonreactive pupil is ominous. Cranial nerve III runs parallel to the brainstem. When increasing intracranial pressure pushes the brainstem down (uncal herniation), it puts pressure on cranial nerve III, causing pupil dilation. The other responses are incorrect.

116. During the neurologic assessment of a "healthy" 35-year-old patient, the nurse asks him to relax his muscles completely. The nurse then moves each extremity through full range of motion. Which of these results would the nurse expect to find? A) Firm, rigid resistance to movement B) Mild, even resistance to movement C) Hypotonic muscles as a result of total relaxation D) Slight pain with some directions of movement

B) Mild, even resistance to movement Page: 637. Tone is the normal degree of tension (contraction) in voluntarily relaxed muscles. It shows a mild resistance to passive stretch. Normally, the nurse will notice a mild, even resistance to movement. The other responses are not correct.

What is the best technique to use to best assess the patient's skin temperature? Use the:

B) dorsal surface of the hand because the skin is thinner than on the palms.

A nursing student is caring for a client with a stroke who is experiencing unilateral neglect. The nurse would intervene if the student plans to use which of the following strategies to help the client adapt to this deficit?

B- Approaches the client from the unaffected side. Rationale: The nurse teaches the client to scan the environment to become aware of that half of the body and approaches the client form the affected side to increase awareness further.

The nurse is assessing the motor function of an unconscious client. The nurse would plan to use which of the following to test the client's peripheral response to pain?

B- Nail bed pressure Rationale: Motor testing in the unconscious client can be done only by testing response painful stimuli. Nail bed pressure tests a basic peripheral pressure on the orbital rim, or squeezing the clavical or sternoleidomastoid muscle.

The nurse is planning to institute seizure precautions for a client who is being admitted from the emergency department. Which of the following measures would the nurse avoid in planning for the client's safety?

B- Putting a padded tongue blade at the head of the bed Rationale: seizure precautions may vary from agency to agency but the generally have some common features. Usually an airway, oxygen, and suctioning equipment are kept available at the bedside. The side rails of the bed are padded, and the bed is kept in the lowest position. The client has an intravenous access in place to have a readily accessible route if anticonvulsant medications must be administered. The use of padded tongue blades is highly controversial, and they should not be kept at bedside. Forcing a tongue blade into the mouth during a seizure more likely will harm the client who bites down during seizure activity. Risks include blocking the airway from improper placement, chipping the client's teeth.

The nurse is caring for the client who begins to experience seizure activity while in bed. Which of the following actions by the nurse would be contraindicated?

B- Restraining the Client's Limbs Rationale: Nursing Actions during a seizure include providing for privacy, loosening restrictive clothing, removing the pillow and raising the side rails in the bed, and placing the client on one side with the head flexed forward, if possible, to allow the tongue to fall forward and facilitate drainage. The limbs are never restrained because the strong muscle contractions could cause the client harm. If the client is not in bed when seizure activity begins, the nurse lowers the client to the floor, if possible protects the head from injury, and moves furniture that may injure the client. Other aspects of care are as described for the client who is in bed.

The client with Parkinson's disease has a nursing diagnosis of falls, Risk for related to an abnormal gait documented in the nursing care plan. The nurse assesses the client, expecting to observe which type of gait?

B- Shuffling and propulsive

A patient has the following preoperative medication order: morphine 10 mg with atropine 0.4 mg IM. The nurse informs the patient that this injection will

B. decrease oral and respiratory secretions, thereby drying the mouth. Atropine, an anticholinergic medication, is frequently used preoperatively to decrease oral and respiratory secretions during surgery, and the addition of morphine will help to relieve discomfort during the preoperative procedures. Antiemetics decrease nausea and vomiting during and after surgery, and scopolamine and some benzodiazepines induce amnesia. An actual sleep state is rarely induced by preoperative medications unless an anesthetic agent is administered before the patient is transported to the operating room.

Conscious sedation is being considered for a patient undergoing a cervical dilation and endometrial biopsy in the health care provider's office. The patient asks the nurse, "What is this conscious sedation?" The nurse's response is based on the knowledge that conscious sedation

B. enables the patient to respond to commands and accept painful procedures. Conscious sedation is a moderate sedation that allows the patient to manage his or her own airway and respond to commands, and yet the patient can emotionally and physically accept painful procedures. Drugs are used to provide analgesia, relieve anxiety and/or provide amnesia. It can be administered by personnel other than anesthesiologists, but nurses should be specially trained in the techniques of conscious sedation to carry out this procedure because of the high risk of complications resulting in clinical emergencies.

20. When the nurse is evaluating the reliability of a patient's responses, which of these statements would be correct? The patient:

B. provided consistent information and therefore is reliable.

ulcerative colitis: assessment

BLOOD, pus, mucus in stool, abdominal pain predefecation, involves the rectum and left colon

Hemolytic disease of newborn (pathologic jaundice)

Baby's rh antigen enter mother; mother produces antibody; antibodies re-enter baby and causes hemolysis and jaundice

Probable signs of pregnancy include...

Ballottement, Chadwick's sign, Goodell's sign, Hegar's sign, positive pregnancy test (rising Hcg levels), uterine enlargement

GUILLIAN-BARRE' SYNDROME : S/S Complication Crisis

Begins in extremities: weakness, paralysis, respiratory failure Progressive ascending muscle weakness of the limbs with: symmetric flaccid paralysis; paresthesias or numbness; loss of tendon reflexes. Autonomic nervous system involvement causes postural hypotension, dysrrhythmias, facial flushing, diaphoresis, and urinary retention. Pain occurring with the slightest of movements is a common feature. Crisis: will progress to medical emergency, likely respiratory failure.

Cranial Nerve Disorders

Bell's palsy (CN VII), unilateral paralysis of the facial muscles. Trigeminal neuralgia (CN V), chronic disease severe facial pain.

Acoustic Neuroma: Analysis

Benign tumor of the eighth cranial nerve

Major Nursing Concern for Guillian-Barre syndrome (polyneuritis)

Breathing Problems

Preoperatively the nurse should teach the parents of an infant with cleft lip/cleft palate to feed the infant using a...

Breck feeder or flanged nipple, slowly to prevent aspiration, and burp frequently to prevent gastric distention.

The PACU has received a semiconscious patient from the operating room and reviews the chart for orders related to positioning of the patient. There are no specific orders on the chart related to specific orders for the patient's position. In this situation, in what position will the nurse place the patient?

C) If the patient is not fully conscious, place the patient in the side-lying position, unless there is an ordered position on the patient's chart.

122. The nurse is caring for a patient who has just had neurosurgery. To assess for increased intracranial pressure, what would the nurse include in the assessment? A) Cranial nerves, motor function, and sensory function B) Deep tendon reflexes, vital signs, and coordinated movements C) Level of consciousness, motor function, pupillary response, and vital signs D) Mental status, deep tendon reflexes, sensory function, and pupillary response

C) Level of consciousness, motor function, pupillary response, and vital signs Pages: 660-661. Some hospitalized persons have head trauma or a neurologic deficit from a systemic disease process. These people must be monitored closely for any improvement or deterioration in neurologic status and for any indication of increasing intracranial pressure. The nurse should use an abbreviation of the neurologic examination in the following sequence: level of consciousness, motor function, pupillary response, and vital signs.

The nurse is preparing a patient for a colonoscopy. The nurse is familiar with the colonoscopy procedures at the hospital and is aware that which type of anesthesia os commonly used for his procedure?

C) Moderate sedation/analgesia is also known as conscious sedation or procedural sedation and is used for short-term and minimally invasive procedures such as a colonoscopy.

110. During an assessment the nurse notices that a patient's umbilicus is enlarged and everted. It is midline, and there is no change in skin color. The nurse recognizes that the patient may have which condition? A) Intra-abdominal bleeding B) Constipation C) Umbilical hernia D) An abdominal tumor

C) Umbilical hernia Page: 537. The umbilicus is normally midline and inverted, with no signs of discoloration. With an umbilical hernia, the mass is enlarged and everted. The other responses are incorrect.

15. An elderly Mexican-American woman with traditional beliefs has been admitted to an inpatient care unit. A culturally-sensitive nurse would:

C) further assess the patient's cultural beliefs and offer the patient assistance in contacting a curandero or priest if she desires.

Common test for renal caliculi?

CT scan

In the absence of postoperative vomiting, GI suctioning, and wound drainage, the physiologic responses to the stress of surgery are most likely to cause

C. fluid overload. Secretion and release of aldosterone and cortisol from the adrenal gland and ADH from the posterior pituitary as a result of the stress response cause fluid retention during the first 2 to 5 days postoperatively, and fluid overload is possible during this time. Aldosterone causes renal potassium loss with possible hypokalemia, and blood coagulation is enhanced by cortisol.

Ulcerative Colitis

CHRONIC symptoms Chronic inflammation of the inner-most lining of large intestine (colon) and rectum in CONTINUOUS STRETCHES of colon, only. Can be debilitating or life-threatening. NO CURE, w/ occasional remission.

Medication for Paget's disease

Calcitonin (calcimar) Etidroante disodium (EHDP) Mithramycin (mithracin)

Appendicitis

Cause: Inflammation of the appendix r/t obstruction of lumen by stool, tumors or foreign bodies. S/S: N/V, fever, pain in RLQ, rebound tenderness, abd. muscle guarding Tx: Surgery Critical: Perforation/rupture will likely lead to peritonitis

Peritonitis etiology/ cause

Cause: Localized or generalized inflammatory process of the peritoneum Primary - blood borne organisms enter peritoneal cavity Secondary - abd. Organs perforate/rupture &amp; release contents into peritoneal cavity (ex. Appendix rupture) **Can be fatal!**

Progesterone (mirena)

Change q yr; check for string after each menses and before intercourse

MG: MYASTHENIA GRAVIS : disease discription

Chronic neuromuscular disorder characterized by fatigue and severe weakness of skeletal muscles Occurs with remissions and exacerbations. Causes decrease in muscle's ability to contract, despite sufficient acetylcholine.

PD: PARKINSON'S DISEASE : S/S Complication Crisis

Classic s/s: tremor at rest, muscle rigidity, bradykinesia. Complications: risk for fall, aspiration, urinary retention/UTI, dysphagia, oculogyric crisis: fixed lateral and upward gaze.

Nonpenetrating-contusions

Cold compresses, analgesics

Critical complication of IICP:

Coma

Parasympathetic Effects:

Constricts Pupils,Stimulate saliva, slows heartbeat, constricts bronchi, stimulates peristalsis, stimulate bile, contracts bladder

1. Caregiver 2. Autonomy and accountability 3. Patient advocate 4. Health promotion

Contemporary nursing requires that the nurse has knowledge and skills for a variety of professional roles and responsibilities. Which of the following are examples? (Select all that apply.) 1. Caregiver 2. Autonomy and accountability 3. Patient advocate 4. Health promotion 5. Lobbyist

19. When planning a cultural assessment, the nurse should include which component?

D) Health-related beliefs

Nonpenetrating-abasions

Eye patch for 24 hours

ulcers:

excavation formed in the mucosal wall, caused by erosion that my extend to muscel layers or through the muscle to the peritoneum (h. pylori)

Penetrating-pointed or sharp objects

Cover with patch; refer to surgeon

Three type of skeletal tongs are used for traction of a client with a spinal cord injury

Crutchfield, Gardner-Wells, Vinke

ICP: What are some expected changes in vital signs when a patient shows signs of increased ICP?

Cushing's triad (3 sx) = may present over time or present suddenly. 1. Widening pulse pressure= SPB increases, but DBP stays same. 2. Bradycardia 3. Hypertension Temperature will also increase if hypothalamus is impacted.

The Glasgow Coma scale is a 15 point scale that is used to measure neurological status, what does it measure responses to?

Eyes Open, Verbal Response, Motor Response

Celiac Disease:

inborn error of wheat and rye metabolism

119. In assessing a 70-year-old patient who has had a recent cerebrovascular accident, the nurse notices right-sided weakness. What might the nurse expect to find when testing his reflexes on the right side? A) Lack of reflexes B) Normal reflexes C) Diminished reflexes D) Hyperactive reflexes

D) Hyperactive reflexes Hyperreflexia is the exaggerated reflex seen when the monosynaptic reflex arc is released from the influence of higher cortical levels. This occurs with upper motor neuron lesions (e.g., a cerebrovascular accident). The other responses are incorrect

103. The nurse suspects that a patient has a distended bladder. How should the nurse assess for this condition? A) Percuss and palpate in the lumbar region. B) Inspect and palpate in the epigastric region. C) Auscultate and percuss in the inguinal region. D) Percuss and palpate the midline area above the suprapubic bone.

D) Percuss and palpate the midline area above the suprapubic bone. Pages: 539-540. Dull percussion sounds would be elicited over a distended bladder, and the hypogastric area would seem firm to palpation.

102. Which structure is located in the left lower quadrant of the abdomen? A) Liver B) Duodenum C) Gallbladder D) Sigmoid colon

D) Sigmoid colon Page: 530. The sigmoid colon is located in the left lower quadrant of the abdomen.

121. While the nurse is taking the history of a 68-year-old patient who sustained a head injury 3 days earlier, he tells the nurse that he is on a cruise ship and is 30 years old. The nurse knows that this finding is indicative of: A) a great sense of humor. B) uncooperative behavior. C) inability to understand questions. D) decreased level of consciousness.

D) decreased level of consciousness. Pages: 660-661. A change in consciousness may be subtle. The nurse should notice any decreasing level of consciousness, disorientation, memory loss, uncooperative behavior, or even complacency in a previously combative person. The other responses are incorrect.

106. The physician comments that a patient has abdominal borborygmi. The nurse knows that this term refers to: A) a loud continuous hum. B) a peritoneal friction rub. C) hypoactive bowel sounds. D) hyperactive bowel sounds.

D) hyperactive bowel sounds. Pages: 539-540. Borborygmi is the term used for hyperperistalsis when the person actually feels his or her stomach growling.

117. When the nurse asks a 68-year-old patient to stand with feet together and arms at his side with his eyes closed, he starts to sway and moves his feet farther apart. The nurse would document this finding as a(n): A) ataxia. B) lack of coordination. C) negative Homans' sign. D) positive Romberg sign.

D) positive Romberg sign. Page: 638. Abnormal findings for Romberg test include swaying, falling, and widening base of feet to avoid falling. Positive Romberg sign is loss of balance that is increased by closing of the eyes. Ataxia is uncoordinated or unsteady gait. Homans' sign is used to test the legs for deep vein thrombosis.

The nurse is admitting a client with Guillian- Barre syndrome to the nursing unit. The client has an ascending paralysis to the level of the waist. Knowing the complication of the disorder, the nurse brings which of the following items into the client's room?

D- Electrocardiographic monitoring electrodes and intubation tray

While a nurse is caring for a patient who is scheduled to have surgery in 2 hours, the patient states, "My doctor was here and told me a lot of stuff I didn't understand and then I signed a paper for her." To fulfill the role of advocate, what is the best nursing action?

D. Call the physician to return and clarify information for the patient. Examples of nursing advocacy include questioning doctors' orders, promoting patient comfort, and supporting patient decisions regarding health care choices.

A nurse has requested and gotten permission to observe a surgical procedure of interest in the hospital in which the nurse is employed. While the patient is being draped, the nurse notices that a break in sterile technique occurs. Which of the following actions on the nurse's part is most appropriate?

D. Point out the observation immediately to the personnel involved. Any break in sterile technique in the operating room should be immediately pointed out and remedied.

A physician is performing a sterile procedure at a patient's bedside. Near the end of the procedure, the nurse thinks that the physician has contaminated a sterile glove and the sterile field. The nurse should

D. point out the possible break in surgical asepsis and provide another set of sterile gloves and fresh sterile field. It is the responsibility of the nurse to point out any possible break in surgical asepsis when others are unaware that they have contaminated the field. Reporting the physician is not indicated, nor does it protect the patient. Asking the physician may lead to infection if the physician is unaware of the break in technique that the nurse believes may have happened. Saying nothing does not protect the patient and is negligence on the part of the nurse.

Abruptio Placentae: postpartal complications

DIC: gums IV site. PE, UTI

Acoustic Neuroma: Assessment

Deafness-partial, initially twitching grimacing of facial muscles dizziness

Respiratory distress syndrome (RDS)

labored respiration after several minutes or hours of normal respiration

ALS: AMYOTROPHIC LATERAL SCLEROSIS : disease discription

Degeneration of motor neurons in brain stem and spinal cord: brain's messages don't reach the muscles No cure.

Elevated BUN and Na

Dehydration

Hypothyroidism S/S

Description: insufficient circulation of thyroid hormones resulting in a hypometabolic state. S/S: fatigue, lethargy, personality and mental changes, mental dullness, decreased cardiac output, anemia, constipation, dry skin, cold intolerance, weight gain, menstrual problems- heavy menstruation

Meds to Tx for Sinus Tach

Digoxin, beta blocker, Dilitazem

Meniere's Disease

Dilation of the membrane of the labyrinth thats causes unbalance and hearing loss. Severe vertigo

Meds tx for A-flutter

Diltiazem, Digoxin, Beta Blocker, Atropine for slow V. rate

OA

occiput anterior; back of baby's head is directly to the front of the mother's pelvis

Auditory Impairment

Dizziness, ringing in ear, loss of balance Safety and injury prevention

Cranial nerve 7 comes out of the temple and runs all the way down to the corner of the mouth. If there are problems with this nerve, what might we see?

Drooping of the corner of the mouth (Bell's Palsy)

Drug Use in Pre-Op setting: Benzodiazepines and barbiturates

Drug used in pre-op setting for sedation and amnesia

Drug Use in Pre-Op setting: Opioids

Drug used in pre-op setting to decrease intraoperative anesthetic requirements and pain.

Drug Use in Pre-Op setting: Anticholinergics

Drug used in pre-op setting to reduce secretions.

Anticholinergic Effect

Dry mouth, dsphagia, impotence Sugarless lozenges, void before taking medication

Bacterium most commonly causing UTI's?

E.coli

Papilledema

Edema and inflammation of the optic nerve at its point of entrance into the retina.

GERD: dietary interventions

Elevate head of bed and sit up after eating. Decrease high fat foods. Drink fluids BETWEEN meals not with meals. Avoid: milk at bedtime & late night snacking/meals, chocolate, peppermint, caffiene, tomato & orange juice. Weight loss. ***NPO status after surgery until gag reflex returns***

Renal Impairment

Elevated BUN & creatinine, Decreased Hct, ALtered K & Na, Edema Fluid and diet restrictions, dialysis

Common interventions related to: UTI/ Cystitis

Empty bladder/bowel regularly & completely; Avoid stagnant urine in the bladder or urethra. Drink water prior to intercourse to promote urination & empty bladder after intercourse. Clean perineal area: front to back. Drink large amt fluids daily.

Meds for Vfib

Epinephrine 1mg repeat 3-5 min Vasopressin 40U wait 10 min before epinephrine Amiodarone 300 mg then 150 mg lidocaine 1-1.5mg/kg 0.5-.75 mg/kg Mg 1-2 g (ETOH/ Torsades)

3. A problem-solving approach that integrates best current evidence with clinical practice

Evidence-based practice is defined as: 1. Nursing care based on tradition 2. Scholarly inquiry of nursing and biomedical research literature 3. A problem-solving approach that integrates best current evidence with clinical practice 4. Quality nursing provided in an efficient and economically sound manager

Ulerative Bowel: Dietary modification

Expect pt to have physician orders for: **NPO** w/ TPN Bowel rest until ulcers have healed.

T or F Nauseas & vomitting should not be treated until pt acutally vomits.

F- n/v should be treated aggressively; preventing comfort,safety, and compliance w/tx regimen

Positive signs of pregnancy

FHB fetoscope 20 wks, doppler 10-12 wks

BELL'S PALSY : disease discription

Facial nerve inflammation; Peripheral facial paralysis due to CN VII motor dysruption; affects one side of face. Inflammation, edema, ischemia, demyelination of nerve, causing sensory and motor loss. Outbreak of herpes vesicles in or around ear; Caused by a reactivation of herpes simplex virus, although other infections (e.g., syphilis or Lyme disease) are sometimes implicated

CNVII

Facial; movement of facial muscles; facial expression; tear formation; salivation; taste sensation in anterior tongue

Precipitous labor

occurs between 20 and 37 wks gestation, uterine contractions and cervical changes

T or F while caring for pt w/suspected appendicitis you would give enema or laxatives in prep for sx?

False - peristalsis can cause appendix to ruputure

Hyperopia

Farsightedness- light rays refract behind the retina. Pt have problem seeing things close

Toxic Shock Syndrome: (TSS) signs and symptoms

Fever 102F/ 38.9C; hypotension;Rash caused by S. aureas

Reye's syndrome: assessment

Fever, increase ICP, decreased LOC, coma, decreased hepatic fxn

Burns

First Degree: epidermal only second Degree: epidermal and dermis third degree: destruction of all skin layer 4th degree: muscles (fascia) bone

The most diagnostic test for all stages of treponema pallidum (syphilis)

Florescent treponemal antibody (FTA-Abs)

The name of the scale used globally to assess a person's consciousness

Glasgow Coma Scale

CNIX

Glossopharyngeal; taste sensation in post third of tongue

Celiac Disease: Nursing consideration

Gluten-free diet (no wheat, oats, rye, barley)

Hypothyroidism Complications

Goiter Decreased metabolic rate Myxedema due to a deficiency of thyroid hormone (adult form) --Myxedema Coma Cretinism (infant form) Atherosclerosis Hyperthyroid (if too much Rx) Hypoglycemia Thyroiditis (Hashimoto's)

GTPAL or TPAL

Gravida, Term, Preterm, abortion, living children

Guillain-Barre Syndrome: Assessment

Guillain-Barre syndrome is a serious disorder that occurs when the body's defense (immune) system mistakenly attacks part of the nervous system. This leads to nerve inflammation that causes muscle weakness.Usually begin in lower extremities

Positioning for Head Injury pt:

HOB elevated 30-45 to increase venous return

measures should an RN take when placing a client in seizure precautions?

Have suction, airway &amp; oxygen at bedside. status epilepticus -a state of continuous seizure activity or a condition in which seizures recur in rapid succession without return to consciousness between seizures.

1. Moving from an acute illness to a health promotion, illness prevention model

Health care reform will bring changes in the emphasis of care. Which of following models is expecting from health care reform? 1. Moving from an acute illness to a health promotion, illness prevention model 2. Moving from illness prevention to a health promotion model 3. Moving from an acute illness to a disease management model 4, Moving from a chronic care to an illness prevention model

Hirschsprung's Disease: Plan

High calories, protein diet. Low-fiber diet. Measure abdominal at umbilicus. Let partent know colostomy is usually temporary, closed when child is 17-22lbs.

Addison's Disease: plan

High: protein, carbohydrate, sodium diet Low: potassium diet. monitor for hypo-glycemic & natremia.

Hirschsprung's Disease: Dx

Hirschsprung's disease is a blockage of the large intestine due to improper muscle movement in the bowel. It is a congenital condition, which means it is present from birth.

Hypothyroidism Labs

History and PE Thyroid function tests: elevated TSH, Low T3/T4 Thyroid scan Radioactive Iodine Uptake test

1. Completing incident reports when appropriate 2. Completing incident reports for a near miss 3. Communicating product concerns to an immediate supervisor

How does the nurse support a culture of society? (Select all that apply.) 1. Completing incident reports when appropriate 2. Completing incident reports for a near miss 3. Communicating product concerns to an immediate supervisor 4. Identifying the person responsible for an incident

Eclampsia

Hypertension, proteinuria tonic-clonic seizures

CNXII

Hypoglossal; tongue movement

What are some expected changes that may be seen when assessing pts w/ Brain Herniation as a result of IICP?

IICP Brain Herniation: - Unilateral dilated pupil. - sluggish, equal pupil response.

What are some expected changes that may be seen when assessing pts w/ Opitic Nerve dysfunction (II, IV, VI) as a result of IICP?

IICP Optic Nerve Dysfunction: - Blurred vision - diplopia: double vision - changes in extraocular eye movements

What is the difference between and ischemic & hemorrhagic stroke?

ISCHEMIC STROKE- results from inadequate blood flow to the brain from partial or complete occlusion of an artery. They are further divided into thrombotic and embolic. **Warning sign is usually a TIA and happens during or after sleep, slower progression, and recurrence is common. HEMORRHAGIC STROKE- results from bleeding into the brain tissue itself (intracerebral or intraparenchymal hemorrhage) or into the subarchnoid space or ventricles (subarachnoid hemorrhage or intraventricular hemorrhage). **Warning sign is usually a headache and happens during activity, sudden onset and fatality more likely with presence of coma.

1. Answer the call light promptly. 4. Answer questions honesty. 5. Demonstrate competence when doing treatments.

Identify behaviors that foster the development of trust. (Select all that apply.) 1. Answer the call light promptly. 2. Call the patient by first name unless requested otherwise. 3. Do all the care as quickly as possible and leave the room as the patient can rest. 4. Answer questions honesty. 5. Demonstrate competence when doing treatments.

Assessment of CNV

Identify location of stimulus; check jaw strength

4. The nurse explains the procedure for giving a tube feeding to a second nurse who has floated to the unit to assist with care.

In which of the following examples is the nurse not applying critical thinking skills in practice? 1. The nurse considers personnel experience in performing intravenous (IV) line insertion and ways to improve performance. 2. The nurse uses a fall risk inventory scale to determine a patient's fall risk. 3. The nurse observes a change in a patient's behavior and considers which problem is likely developing. 4. The nurse explains the procedure for giving a tube feeding to a second nurse who has floated to the unit to assist with care.

Bells Palsy (facial paralysis): Assessment

Inability to close eye, speech difficulty, loss of taste. distortion of one side of face

Gout:

Inborn error of purine metabolism

Hyperthyroidism Complications

Increased metabolic rate r/t increased circulating thyroid hormone Thyroid Storm- may result in mania or heart failure

Renal Failure: Assessment

Increased protein and albumin losses in urine that leads to protein deficiency

Meningitis: Causes

Infections (viral, bacteria, fungal) Neurosurgical procedure, basilar-skull fracture Otitis media, mastoiditis

Etiology Pathophysiology Cranial Nerve Disorders

Inflammation, Infection, Viral

CVA: CEREBROVASCULAR ACCIDENT : disease discription

Ischemia of brain tissue: Hemorrhage, thrombus, embolus. Medical Emergency

Symptoms of stage 1 (6-8 hours after last use)

anxiety, anorexia, tremors, nausea and vomiting, depression, headache, increased BP, tachycardia, and profuse sweating

Liver Impairment

Jaundice, Light stool dark urine, elevated AST (SGOT) ALT (SGPT) bilirubin, altered PTT Small frequent meals, good skin care

Components of a neurological examination

LOC, pupillary evaluation, neuromuscular response, vital signs

Hypothyroidism Nursing Interventions

LOOK AT S/S- then select appropriate action. vital signs warm environment teach importance of lifelong Rx regiment

Huntington's Disease: drug therapy

Librium, haldol, thorazine

Cluster C personality disorders includes...

anxious, fearful behavior

Inflammatory Bowel Dietary interventions

Low Residue/ NO Fiber diet - A high residue diet may irritate an inflamed mucosa. Avoid HOT, spicy foods &amp; pepper (increase peristalsis). Increase Protein, Decrease Lactose. NO Alcohol, carbonated beverages, tea, coffee, broth.

Gout: Nursing Consideration

Low-purine diet (no fish no organ meat)

The use of SSRIs with...

MAOIs, selective MAOIs, tryptophan, and St. John's wort is contraindicated. Serotonin syndrome can occur.

A client with a spinal cord injury requires this for turning

MD order

Hematocrit

Man 42-52% women 35-47% child 3-12 yo) 35-45%

Following amniocentesis, the client should be told to report...

any cramping or bleeding and avoid lifting objects heavier than 5 pounds for several days.

Incompetent cervix is corrected by performing a...

McDonalds' cerclage or Shirodkar procedure to close the cervix.

Visual Fields

Measurement of range of vision (perimetry) Normally 50 angle superiorly, 60 medially, 70 inferior, 90 lateral.

Induction of labor criteria

Mother cannot have CPD fetus mush have vertex presentation and engaged mothers cervix must ripened

CVA: CEREBROVASCULAR ACCIDENT : S/S Complication Crisis

Motor changes: opposite side, balance, coordination, gait, proprioception Sensory Changes: Aphasia, Agnosia, Apraxia, Visual problems, hemianopsia Cognitive Changes: impaired memory, disoriented Paralysis, difficulty swallowing, talking, memory, pain. Assessment includes: glasgow coma scale/LOC

3. Deflect your eyes downward to show respect

Mr. Sakda emigrated from Thailand. When taking care of him, you note that he looks relaxed and smiles but seldom looks at you directly. How do you respond? 1. Use therapeutic communication to assess for increased anxiety 2. Sit down and position yourself closer so you are at eye level 3. Deflect your eyes downward to show respect 4. Continue to maintain eye contact

2. Coach her to give herself positive messages about her ability to do this

Mrs. Jones states that she gets anxious when she thinks about giving herself insulin. How do you use your understanding of intrapersonal communication to help with this? 1. Provide her the opportunity to practice drawing up insulin 2. Coach her to give herself positive messages about her ability to do this 3. Bring her written material that clearly describes the steps of insulin administration 4. Use therapeutic communication to help her express her feeling about giving herself an injection

Treatment of opiate overdose...

Narcan

Myopia

Nearsightedness- light rays refract at a point in front of the retina. Pt have problem seeing things far away

Pulseless electrial activity (PEA)

No pulse but u see rhythm if rate is < 60 bpm give atropine with epi. If >60 give epi or vasopressin 40U

Addison's Disease:

occurs when the adrenal gland fails to secrete sufficient mineralocorticoids, glucocorticoids and androgens.

Meningitis: Assessment

Nuchal rigidity-stiff neck Kernig's sign- when hip flexed 90 knee is restristed and painful Brudzinskis- attempt to flex the neck will produce flexion at knee and thigh Opisthotonic position- Legs hyperextended forming arc with the trunk

3. In-service education

Nurses at a community hospital are in an education program to learn how to use a new pressure-relieving device for patients at risk for pressure ulcers. This is which type of education? 1. Continuing education 2. Graduate education 3. In-service education 4. Professional Registered Nurse Education

Risk factors for Cholelithiasis/cholecystitis

Obesity, sedentary lifestyle, women, 50 yo, increased level of cholesterol

GCS: Best motor response

Obeys=6; localizes pain=5; withdraws=4; abnormal flexion=3; extends=2; no response=1

Clinical Manifestations Bell's palsy

One-sided facial paralysis, Loss of corneal reflex, Loss or impairment of taste, Increased tearing from lachrymal gland

GCS: Verbal response

Oriented=5; confused conversation=4; inappropriate words=3; incomprehensible words=2; no response=1

3rd degree AV block

P wave independent of QRS

Guthrie blood test

PKU (phenylketonuira) absense of enzyme needed to metabolize essential amino acid phenyalalanine

1st degree Heart block

PR interval >.20 e/t rheumatic feer, digoxin, beta blocker, inferior MI, Increase vagal tone

2nd degree AV block: type 1or mobitz1

PR interval prolongs until QRS dropped e/t inferior MI, digitalis toxitiy, increased vagal tone, myocarditis,

SEIZURE : disease discription

Paroxysmal, uncontrolled electrical discharge of neurons in the brain that interrupts normal function.

Perinatal asphyxia

Perinatal asphyxia or neonatal asphyxia is the medical condition resulting from deprivation of oxygen to a newborn infant that lasts long enough during the birth process to cause physical harm, usually to the brain.

Bells Palsy (facial paralysis): Analysis

Peripheral involvement of the seventh cranial nerve

pernicious anemia

Pernicious anemia is a decrease in red blood cells that occurs when the body cannot properly absorb vitamin B12 from the gastrointestinal tract. Vitamin B12 is necessary for the proper development of red blood cells.

Treatment of cord prolapse

Place client inf Trendelenburg or knee-chest position, rapid infusion of normal saline or LR, oxygen

GUILLIAN-BARRE' SYNDROME : disease discription

Polyneuritis: peripheral nerve disease; autoimmune inflammatory response to prior infection. Acute immune-mediated polyneuropathy d/t damage to myelin sheath of Peripheral Nerves.

Diabetic Assessment:

Polyuria, polydipsia, polyphagia

What allergy would you assess for in pts being given Pancrelipase?

Pork - used as enzyme supplement to aid in break down of food so that it can be utilized for energy, cell growth and repair

Acoustic Neuroma: Nursing consideration

Pre and Post operative care for posterior fossa craniotomy. assist with turning head and neck

Pre-Op patients should be screened for possible critical allergies?

Pre-Op pts should be screened for critical allergies: Latex, Iodine, and allergies to anesthesia that may result in Malignant Hyperthermia.

What is psoriasis?

Psoriasis is a common skin condition that causes skin redness and irritation. Most people with psoriasis have thick, red skin with flaky, silver-white patches called scales.

A client admitted for acute pyelonephritis is about to start antibiotic therapy. Which symptom would be expected in this client?

RATIONAL: 2) The client may complain of pain on the affected side because the kidney is enlarged and might have formed an abscess. Hypertension is associated with chronic pyelonephritis. Pain may radiate down the ureters or to the epigastrium. The client would have tenderness with deep palpation over the CVA.

The nurse is developing a care plan for a client with hepatic encephalopathy. Which of the following should the nurse include? 1

RATIONALE: 1) Hepatic encephalopathy is a degenerative disease of the brain that is a complication of cirrhosis. For the client with hepatic encephalopathy, the nurse may administer the laxative lactulose to reduce ammonia levels in the colon. Protein intake is usually restricted to reduce serum ammonia levels until the client's mental status begins to improve. Sedatives are avoided because they can cause respiratory or circulatory failure. Bed rest is encouraged because physical activity increases metabolism, leading to an increased production of ammonia.

The nurse is developing a care plan for a client with hepatic encephalopathy. Which of the following should the nurse include? 1) Administering a lactulose enema as ordered. 2) Encouraging a protein rich diet 3) Adminis.tering sedatives as needed. 4) Encouraging ambulation at least 4 times a day.

RATIONALE: 1) Hepatic encephalopathy is a degenerative disease of the brain that is a complication of cirrhosis. For the client with hepatic encephalopathy, the nurse may administer the laxative lactulose to reduce ammonia levels in the colon. Protein intake is usually restricted to reduce serum ammonia levels until the client's mental status begins to improve. Sedatives are avoided because they can cause respiratory or circulatory failure. Bed rest is encouraged because physical activity increases metabolism, leading to an increased production of ammonia.

A nurse is caring for a client with end stage renal disease. Which nursing diagnosis has priority?

RATIONALE: 2) Excess Fluid Volume is a top priority nursing diagnosis for a client with end stage renal disease because the kidney can no longer remove fluid and wastes. The other diagnoses may also apply, but they don't take priority.

During a health history, which statement by a client indicates a risk of renal calculi? -

RATIONALE: 4) Renal calculi are commonly composed of calcium. Diets high in calcium may predispose a person to renal calculi. Milk and milk products are high in calcium. Cola soft drinks don't contain ingredients that would increase the risk of renal calculi. Jogging and increased stress aren't considered risk factors for renal calculi formation.

A client who recently had abdominal surgery tells the nurse he felt a popping sensation in his incision during a coughing spell, following by severe pain. The nurse anticipates an evisceration. Which supplies should she bring to the client's room?

RATIONALE: 4) Sterile saline solution and sterile dressings 4) Saline solution is isotonic, or close to body fluids in content, and is used along with sterile dressings to cover an eviscerated wound (a wound that opened, allowing the intestines to protrude outside the body) and keep it moist.

A nurse is instructing a client with oxalate renal calculi. What foods should the nurse urge the client to eliminate from his diet?

RATIONALE: 4) To reduce the formation of oxalate calculi, urge the client to avoid foods high in oxalate, such as spinach, rhubarb, and asparagus. Other oxalate- rich foods to avoid include tomatoes, beets, chocolate, cocoa, celery, and parsley. Citrus fruits, molasses, dried apricots, milk, cheese, ice cream, sardines and organ meats do NOT produce oxalate and do NOT need to be omitted from the client's diet.

A client returns from the operating room after extensive abdominal surgery. He has 1,000ml of lactated Ringer's solution infusing via central line. The physician orders the I.V. fluid to be infused at 125ml/hr plus the total output of the previous hour. The drip factor of the tubing is 15gtt/min. The client's output for the previous hour was 75m. via Foley catheter, 50ml via NG tube, and 10ml via Jackson-Pratt tube. How many drops per minute should the nurse set the I.V. flow rate at to deliver the correct amount of fluid? Record as a whole number. ______ ggt/minute.

RATIONALE: 65ggt/min. First calculate the volume to be infused in milliliters: 75ml + 50ml + 10ml = 135ml total output for the previous hour; 135ml + 125ml ordered as a constant flow = 260ml to be infused over the next hour. Next, used the formula: Volume to be infused/ Total minutes to be infused x Drip Factor = Drops per min. In this case, 260ml divided by 60min x 15 ggt/min = 65 ggt/min

A client in acute renal failure becomes severely anemic and the physician prescribes 2 units of packed red blood cells. A nurse should plan to administer each unit:

RATIONALE: Infusing a unit of RBCs over 1 to 3 hours is standard practice.

Plasmapheresis (plasma exchange)

Removal of patients plasma and plasma components Produces a temporary reduction in the antibodies Does not treat the underlying abnormality

Physical Assessment of Renal System- INSPECTION: Skin, Mouth, Face & extremities, Abdomen, Weight, General State of Health

Renal System- INSPECTION: SKIN: pallor, yellow-gray cast, excoriations, changes in turgor, bruises, texture (e.g., rough, dry skin) MOUTH: stomatitis, ammonia breath odor FACE & EXTREMITIES: generalized edema, peripheral edema, bladder distention, masses, enlarged kidneys ABDOMEN: striae, abdominal contour for midline mass in lower abdomen (may indicate urinary retention) or unilateral mass (occasionally seen in adult, indicating enlargement of one or both kidneys from large tumor or polycystic kidney) WEIGHT: weight gain secondary to edema; weight loss and muscle wasting in renal failure GENERAL STATE OF HEALTH: fatigue, lethargy, and diminished alertness

What is the PPE transmission precaution for meningococcal meningitis?

Respiratory Isolation = Droplet precautions: Gloves, Gown, Mask within 3 ft of pt.

Common causes/ risk factors: Atelectasis

Respiratory complication that may be the result of a blocked airway, diminished surfactant, or mucus plug. Recent general anesthesia, shallow breathing, respiratory muscle weakness and immobility are common risk factors.

Definition: Atelectasis

Respiratory complication when the alveoli within the lung becomes deflated, resulting in a complete or partial collapse of a lung.

Parkinson's disease Complications

Respiratory tract infections. Urinary tract infections. Skin breakdown. Falls. Adverse affects of medications.

Describe extrapyramidal signs and symptoms:

Restlessness or desire to keep moving, rigidity, tremors, pill rolling, mask-like face, shuffling gait, muscle spasms, twisting motions, difficulty speaking or swallowing, loss of balance control

Problems with hemolysis occur if the mother is...

Rh negative and the fetus is Rh positive.

Acute complication of cerebral edema:

Seizure

Cluster A personality disorders includes...

odd, eccentric behavior

Akinesia

Slowed or delayed movement that affects chewing, speaking, eating

What do we do to check function of the optic nerve?

Snellen's Chart: Field of vision, visual acuity and structures (external, internal, red reflex and optic disc.) Reading a newspaper or magazine. Holding up fingers

Bowel Perforation: s/s

Spillage of gastric or duodenal contents into peritoneal cavity. S/S: Sudden, severe pain unrelated to intensity & location at admittance, Rigid, boardlike abdomen, shallow, grunting respirations

CNXI

Spinal accessory; movement of trapezius and sternocleidomastoid muscles

GCS: Eye opening

Spontaneous=4; to speech=3; to pain=2; no response=1

Sedatives used in labor

Stadol (butorphanol) and Nubain (nalbuphine); provide pain relief with little suppression of FHT

An elderly client is brought to the emergency department complaining of acute back pain. The client denies any chronic illness, allergies, or previous hospitalizations. Which of the following is the BEST initial response for the nurse to make to this client? 1. "We'll get this pain under control in no time." 2. "Are you sure you've never been in the hospital?" 3. "Did you fall, lift something heavy, or turn the wrong way?" 4. "On a scale of 1 to 10, with 10 being the worst, rate the pain you are experiencing."

Strategy: "BEST" indicates priority. 1) false reassurance; nurse should complete assessment 2) confrontational response; pain assessment is priority 3) should first assess intensity of pain as well as location 4) CORRECT— assessment, is objective and clear, and responds directly to client's complaint; gives information for further intervention

The nurse cares for an older client diagnosed with terminal lung cancer. When told about the diagnosis, the client becomes very angry. He curses, throws objects, and hits the nurse tech and LPN/LVN when they attempted provide care for him. It is MOST important for the nurse to take which of the following actions? 1. Inform client that injury or risk of injury to staff is not acceptable. 2. Send the staff out of the room. 3. Administer prescribed antianxiety with full glass of water. 4. Report signs/symptoms to physician immediately.

Strategy: "FIRST" indicates priority 1) CORRECT— set limits on client's behavior; staff has the right to work in a safe environment 2) gives client the power; speak calmly to client, help to verbalize feelings, use nonthreatening body language 3) nurse should use least restrictive interventions to assist the client to regain control 4) passing the buck; it is the nurse's responsibility to care for the client

The nurse cares for a client receiving chlordiazepoxide (Librium). It is MOST important for the nurse to observe for which of the following? 1. Skeletal muscle spasms and insomnia. 2. Anorexia and dry mouth. 3. Diarrhea and euphoria. 4. Drowsiness and confusion.

Strategy: Think about each answer. 1) dystonia is side effect of antipsychotics; insomnia caused by SSRIs 2) Ritalin causes anorexia; dry mouth is side effect of tricyclic antidepressants 3) not caused by Librium 4) CORRECT— antianxiety and sedative/hypnotic used to treat anxiety and alcohol withdrawal; causes drowsiness and sedation; use caution when driving or operating equipment; confusion may indicate immediate n

The nurse cares for a client at term in labor. The client's blood pressure is 182/88 and fetal heart rate (FHR) is 132-134 with minimal beat-to-beat variability. Her bloody show is dark red and there is more bleeding than anticipated. Her abdomen is firm between contractions and she complains of back pain. The nurse understands that the client is at risk for which of the following? 1. Placenta previa. 2. Abruptio placenta. 3. Miscarriage. 4. Imminent delivery.

Strategy: Think about each answer. 1) placenta is implanted near or over the cervical os; symptoms include painless, sudden, profuse bleeding in third trimester 2) CORRECT— premature separation of placenta; painful vaginal bleeding, abdomen is tender, painful, tense, possible fetal distress; prepare for immediate delivery 3) occurs before 20-24 weeks of pregnancy; indications are persistent uterine bleeding and cramp-like pain 4) symptoms are classic signs of abruption

Following the administration of meperidine HCl (Demerol) for an adult client, the nurse expects which of the following? 1. The client states that he feels better. 2. The client is talking with visitors. 3. The client appears to be physically relaxed. 4. The client is no longer crying or moaning.

Strategy: Think about how each answer relates to pain. 1) client may express pain relief, but in reality may still be experiencing pain 2) client may still be in pain 3) CORRECT— nonverbal cues are the best indication of pain relief 4) not best indication of relief of pain

The Glasgow Coma Scale

System for assessing the degree of consciousness, or impairment in the critically ill and for predicting the duration + outcome of coma.

T or F Antacids promote premature dissolving of enteric-coated meds

T - separate administration of other meds by 1-2 hours

T or F cardiac enzymes are ordered for GI pts?

T - they will rule out if s/s are due to cardiac issue rather than GI issue

When teaching a patient about taking Pepcid what should you include:

Take 1 hr before meals (causes acid-producing parietal cells of stomach to be less responsive to stimulation-blocks 90% of acid secretion)

The nurse is teaching a health promotion class to a group of clients. The nurse should recognize that the risk for developing pneumonia is increased if a client has

The correct answer is A. Risk factors for pneumonia include altered mental status, diabetes mellitus, chronic lung disease, and cancer. The client with moderate Alzheimer's disease has altered mental status.

Snellen Test

Test visual Acuity. Pt stands 20 ft reads letters.

In the surgical setting, where is the center of the sterile field?

The center of the sterile field is the site of the surgical incision.

The nurse is assessing a client who had a closed liver biopsy. The nurse should understand that the client is at increased risk for a. altered peripheral tissue perfusion b. sensory-perceptual alteration c. altered urinary elimination d. ineffective airway clearance

The correct answer is A. A client who had a liver biopsy is at risk for hemorrhage. Altered peripheral tissue perfusion is the nursing diagnosis associated with hemorrhage.

The home health nurse is checking an adult client who is receiving small-bore nasogastric (NG) tube feedings. The nurse should immediately report that the client has

The correct answer is A. A client who is receiving a NG feeding is at risk for aspiration pneumonia which may be indicated by the presence of a dry cough. Hypoactive bowel sounds do not require immediate action.

The home health nurse has reinforced teaching with a pregnant client who is starting heparin sodium as prescribed. Which of the following statements by the client would indicate a correct understanding of the teaching?

The correct answer is A. A client who starts heparin sodium should carry a medical alert with them at all times in case of injury or bleeding. Food precautions exist with warfarin sodium and an electric razor is recommended.

The nurse is preparing to change the dressing on a client's triple lumen central venous catheter. Which of the following actions would be correct for the nurse to take? a. removing the client's old dressing using sterile gloves b. lowering the client's head before removing the dressing c. applying triple-antibiotic ointment to the catheter insertion site d. putting on a face mask after opening the prepackaged dressing kit

The correct answer is D. The nurse should put on a face mask when the dressing is changed on a client's central venous catheter. It would be appropriate to put on the mask after the dressing kit has been opened. The client's head should be turned away from the catheter insertion site when the dressing is removed.

The nurse is present when an adult client stops eating, grabs the throat and falls to the floor. The nurse should immediately a. open the client's mouth b. get a stretcher c. start abdominal thrusts d. determine if the client can speak

The correct answer is D. The question should increase the nurse's suspicion that the client has an obstructed airway. The nurse should immediately determine if the client can speak.

1. Diagnostic reasoning.

The nurse sits down to talk with a patient who lost her sister 2 weeks ago. The patient reports she is unable to sleep, feels very fatigued during the day, and is having trouble at work. The nurse asks her to clarify the type of trouble. The patient explains she can't concentrate or even solve simple problems. The nurse records the results of the assessment, describing the patient as having ineffective coping. This is an example of: 1. Diagnostic reasoning. 2. Competency. 3. Inference. 4. Problem solving.

2. Clarifying

The nurse states, "When you tell me that you're having a hard time living up to expectations, are you talking about your family's expectations?" The nurse is using which therapeutic communication technique? 1. Providing information 2. Clarifying 3. Focusing 4. Paraphrasing

4. Feedback

The nurse summarizes the conversation with the patient to determine if the patient has understood him or her. This is what element of the communication process? 1. Referent 2. Channel 3. Environment 4. Feedback

Placenta Previa

The placenta is implanted in the lower uterine segment, usually near or over the cervical opening; placenta may partially or totally cover cervical os. Painless bleeding

The length at birth (average 19-21 inches) increases by...

one inch per month during the first six months of life.

Amyotrophic Lateral Sclerosis (ALS) Supportive care

Treat symptoms. NG tubes. Mechanical ventilation.

Parkinsons Disease: Assessment

Tremors (pill rolling motion) akinesia (loss of automation), ridgidity, motorized propulsive gait. slurred speech, dysphagia.

TRIGEMINAL NEURALGIA : disease discription

Trigeminal Nerve- degeneration/ pressure. Chronic disease of trigeminal nerve (cranial nerve V) causing severe facial pain The maxillary and mandibular divisions of nerve are effected

CNV

Trigeminal; facial sensation, corneal reflex, mastication

T or F Macrobid should be given with milk?

True

Variable decelerations

V-shaped transitory decreases in the fetal heart rate that occur anytime during the contraction; can also occur when no contraction is present

Syphilis blood test

VDRL

True or False an anti-diarrheal is contraindicated w/a bowel obstruction:

True - colitis, N/V, and diarrhea should not be suppressed if underlying cause is not known

T or F calcium stones are alkaline:

True - this pt would need acid ash diet

Post surgery for stone removal hematuria is expected T or F

True bright red blood would be cause for concern

CNX

Vagus; pharyngeal contraction; symmetrical movement of vocal cords and soft palate; movement and secretion of thoracic and abdominal viscera

Mithramycin (mithracin)

Tx for paget's disease SD: Facial flushing GI upset NC: monitor BUN, liver, renal test. Platelet and PT times

Etidroante disodium (EHDP)

Tx for paget's disease: SE: Diarrhea NC: dont give with food, milk or antacids (reduce bone absorption)

Calcitonin (calcimar)

Tx for paget's disease; SE:increase urinary frequency NC: Give at bedtime (HS)

Trigeminal Neuralgia (Tic Douloureux): Analysis

Type of neuralgia involving one ore more branches of the 5 CN. Causs infection of sinuses, teeth, mouth or irritation of nerve from pressure

Physical Assessment of Renal System- URODYNAMIC TESTS:

URODYNAMIC TESTS: study the storage of urine within the bladder and the flow of urine through the urinary tract to the outside of the body.

TMP-SMZ(Bactrim) may be prescribed for a pt with what?

UTI

Astigmatism

Uneven curvature of cornea causing blurring of vision.

BELL'S PALSY : S/S Complication Crisis

Unilateral weakness of facial muscles Pain around ear Unilateral inability to close eye Drooping of mouth Inability to smile, frown, whistle paralysis that distorts smiling, eye closure, salivation, and tear formation on the affected side. Distinguishing it from the facial paralysis associated with some strokes, which affect the muscles of the mouth more than those of the eye or forehead. Complication: Corneal abrasion or ulceration Residual facial weakness

Thermal: Hot metals, liquids

Use of goggles to protect the cornea, patching analgesic

Tensilon Test

Used to diagnose MG and to differentiate between myasthenic crisis and cholinergic crisis.

Plasmapheresis

Used to remove antibodies. Often done when respiratory involvement has occurred, or before planned surgery.

Is occluded ligated and severed; bands; clip to prevent sperm passage

Vas deferens

1. Results in neurophysiological response.

When teaching a patient about the negative feedback response to stress, the nurse indicates which of the following to describe the benefits of this stress response? 1. Results in neurophysiological response. 2. Reduces body temperature 3. Causes a person to be hypervigilant 4. Reduces level of consciousness to conserve energy.

3. 18 inches to 4 feet from the patient.

When the nurse takes the patient's history, he or she sits: 1. Next to the patient. 2. 4 to 12 feet from the patient. 3. 18 inches to 4 feet from the patient. 4. 12 inches to 3 feet from the patient.

When is a lumbar puncture not indicated?

When the patient has a possible brain tumor.

3. Shifting quickly from subject to subject.

When working with an older adult, the nurse remembers to avoid: 1. Touching the patient. 2. Allowing the patient to reminisce. 3. Shifting quickly from subject to subject. 4. Asking the patient how he or she feels.

2. "Why do you always put so much salt on your food?"

Which of the following statements would be most likely to block communication? 1. "You look kind of tired today." 2. "Why do you always put so much salt on your food?" 3. "It sounds like this has been a hard time for you." 4. "If you use your oxygen when you walk, you may be able to walk farther."

Terbutaline:

a beta-receptor agonist, is approved by the Food and Drug Administration for inhibiting preterm uterine contractions.

Addison's Disease: treatment

administer lifelong exogenous replacement:(take with food or milk) glucocorticoids- prednisone, hydrocortisone mineralcorticoids-gludrocortisone (Florinef) avoid stress.

Myasthenia gravis: plan

administer medication before meals (anticholinesterases, corticosteriods, immunosuppresent). No use of streptomycin or neomycin (they produce muscle weakness)

Front-line treatment of spinal cord injury

administration of high-dose steroids within eight hours of injury

Tx for syphilis

adminster penicillin which crosses placenta treating fetus

Conduct disorder in children gives way to an...

adult diagnosis of antisocial personality disorder.

Quadriparesis or quadriplegia:

affecting all four limbs

Hemiparesis or hemiplegia:

affecting both limbs on one side

Diparesis, diplegia, paraparesis, or paraplegia:

affecting both upper OR both lower limbs

Monoparesis or Monoplegia:

affecting one limb

Contractility

ability of cardiac cells to respond to an impulse by contracting

Automaticity

ability of the heart to initiate impulses regularly and spontaneously without initiation

Conductivity

ability to respond to a cardiac impulse by transmitting the impulse along cell membrane

Excitability

ability to respond to a stimulus by initiating a cardiac impulse

Tracheoesophageal fistula (TEF)

abnormal opening between trachea and esophagus

bradykinesia

abnormal slowness of voluntary movements and speech

Hirschsprung disease (congenital aganglionic megacolon)

absence of nerve stimulation to the bowel, which produces normal peristalsis

Symptoms of substance abuse include...

absenteeism, decline in school or work performance, frequent accidents, increased isolation, slurred speech, tremors

Betamethasone

accelerates surfactant production in preterm labor.

Teratogenic agents include...

accutane, alcohol, cytomegalovirus, herpes, LSD, rubella virus, syphillis, tetracycline, toxoplasmosis

Pancreatitis: assessment

abdominal pain, n/v vomiting 24/48 hrs after heavy meal or alcohol ingestion; pain relief with position change. RUQ/ LUQ pain radiates to back.

The client taking Antabuse (disulfiram) should avoid...

alcohol or substances containing alcohol; this can produce headache, nausea and vomiting, tachycardia, chest pain, convulsions, cardio-respiratory collapse, and death

Tranxene is given for the treatment of...

alcohol withdrawal

Duodenal ulcer & Gastric Ulcer Risk factors:

alcohol, smoking, stress, NSAIDS (gastric)

Uric acid stone (excess purine) would require what kind of diet?

alkaline ash

In preparation for a cerebral angiography, what do you need to ask the patient before the test?

allergies to Iodine - contrast media (dye)

If pt is scheduled for an IVP what allergy should you assess for?

allergy to iodine or seafood

a pt who is being prescribed TMP-SMZ should be assessed for what allergy?

allergy to sulfas

Presumptive signs of pregnancy include...

amenorrhea, breast sensitivity, Chadwick's sign, fatigue, fingernail changes, urinary frequency, weight gain

Ferning test/nitrazine testing is used to confirm...

amniotic fluid.

Cardiac Output

amount of blood ejected from left ventricles in the the aorta every minute

If you want to view a pts gallbladder and ducts what would be ordered?

an ERCP - gag is paralyzed so NPO until returns

Somatoform disorder is characterized by the...

appearance of physical symptoms for which there is no apparent organic or physiological cause. Client continuously seeks medical treatment for a physical complaint even though he has been told there is no evidence of physical illness.

Appendicitis: surgery

appendectomy ( places in fowlers position Post-op)

Assessment of CNX

ask client to say "Ah"; uvula should rise midline; check ability to swallow

posterior fontanelle close

at 2-3 months

Signs of overdose related to stimulant abuse...

ataxia, hyperpyrexia, respiratory distress, seizures, cardiovascular collapse, coma, death

Atrial flutter

atria depolarize @ 250-350 b/min Ventricles <300 bpm A/V ratio of 3:1 Saw tooth P wave

Sinus Tachycardia

atrial >100bpm ventricular >150 bpm

PR interval:

atrial depolorization & AV delay

Med tx for 2nd degree av block type 1

atropine 0.5 mg iv, o2

Ortolani click

audible sound that is made when the affected hip is abducted

Cholelithiasis/cholecystitis: diet plan

avoid fried food, pork, cheese, alcohol low fat high protein, carbs skim milk

ulcers: plan

avoid oversecretion and hypermotility in the GI tract. eat 3 small frequent meals, stop smoking (inhibits ulcer repair)

Cluster C disorders include...

avoidant, dependent, and obsessive-compulsive personality disorders, which are characterized by anxious, fearful behavior.

Post crainiotomy, cold application should be used for...

periorbital edema and bruising.

Symptoms of autonomic hyperreflexia or dysreflexia include...

bradycardia, cold, dry, pale skin below the lesion level, headache, hypertension, nasal congestion, piloerection, profuse sweating

if pt is on acid-ash diet to decrease pH of urine what will it include:

bread, cereal, whole grains, cranberries, legumes, tomatoes,oysters, fish, poultry, pastries

lumpectomy

breast, axillary nodes, and the major and minor pectoral muscles are preserved

Lochia rubra

bright red bleeding, occurs after delivery and lasts approximately 3 days

bells palsy

peripheral involvement of the 7th cranial nerve

Glaucoma: 5 danger signs

brow arching halos around lights blur vision diminished peripheral vision Headache or eye pain

Diverticular disease: plan

bulk laxative (metamucil), high fiber diet, avoid food with seed, increase fluid, anticholinergic (bentyl)

A client with a herpes lesion will be delivered...

by C-section because there is a possibilty of transmission of the infection to the fetus with direct contact to lesions.

inevitable abortion

cervical dilation cannot be prevented; persistent hemorrhage, cervical dilation and effacement NC: D&C

At 4 years, the presschooler can...

balance on one foot for 5 seconds, walk heel to toe, catch a ball, throw a ball overhand, skin and hop on one foot, use scissors, lace shoes, copy a square, add three parts to stick figure; understands time in relation to daily events, prizes independence, takes pride in accomplishments, enjoys entertaining, shares family secrets, imaginary friend; uses sentences, 1,500 words, egocentrism

Diagnosis of hirschusprung disease (congenital aganglionic megacolon)

barium enema, rectal biopsy, anorectal manometry (pressure of internal anal sphincter, fails to relax)

Clients with mild preeclampsia are treated with...

bed rest and a low sodium diet.

A Bishop score evaluates cervical readiness for labor based on five factors

cervical softness, cervical effacement, dilation, fetal position, and station.

chadwicks sign

cervix becomes bluish color,

Prevention Toxic Shock Syndrome

change tampons 3-6 hrs; do not use superabsorbent tampons.

Presumptive sign of pregnancy

changes felt by woman: amenorrhea, n/v, breast sensitivity fatigue, quickening

probable sign of pregnancy

changes observed by examiner: uterine enlargement, positive urine test, Hegars sign, chadwick sign

Sublimation

channeling of unacceptable thoughts and behaviors into socially acceptable behaviors

Persons with antisocial personality disorder are often described as...

charming, smooth talking, and extremely intelligent; do not feel remorse for wrongs committed

Assessment of CNIII, CNIV, CNVI

check pupil costriction; accommodation and convergence as object brought near eyes; strength of lid closure

Assessment of CNVII

check symmetry of facial expressions; muscle strength; identify sweet, sour, and salty taste on anterior of tongue

Most important assessment if pyleonephritis is suspected?

check temperature

Diverticular disease: assessment

cramping pain in left lower quadrant of abdomen relieved by passage of stool or flatus; constipating alternating with diarrhea

Subtypes of schizophrenia

catatonic, disorganized, paranoid, residual, undifferentiated

Pathologic jaundice (hemolytic anemia)

caused by blood antigen incompatibility; anemia caused due t hemolysis of large number of erythrocytes

Treatment of vasospasms associated with strokes include the use of...

calcium channel blockers.

Treatment of MgSO4 toxicity

calcium gluconate

types of urinary tract caliculi

calcium oxalate (30-45%) calcium phosphate (8-10%) struvite (10-15%) cystine (1-2%)

what to do if Prolapsed cord

call for help, push against presenting part to relieve pressure on cord, place in trendelenburg or knee chest position

nystatin tx for:

candida

Signs of overdose related to sedative-hypnotic abuse...

cardiovascular and respiratory depression, seizures, shock, coma, death

V-fib

choatic heart it not pumping, just shaking, no CO, no Pulse 300-600 bpm, no P wave

Systemic Lupus Erythematosus (SLE)

chronic systemic inflammatory disease of connective tissue that involves skin, joints, serous membranes, kidneys, hematologic system.

Cholelithiasis: If pt presents w/ acute pain related to stones lodged in duct, what urine and feces characteristics are expected?

clay colored stools and dark amber urine

Bipolar I

client experiences periods of acute mania and major depression

A T-cell count in HIV less than 200 indicates...

client is at risk for opportunistic diseases

Stage 1 labor

closed cervix to 10 cms dilation of cervix

A toddler's anterior fontanel is...

closed.

Bp is inaccurately low

cuff is too long or too wide brachial artery is above the heart

Bp is inaccurately high

cuff is too short or too narrow or brachial artery is below the heart

Late decelerations

drops in the fetal heart rate late in the contraction are caused by utero-placental insufficiency. Decelerations are U-shaped and mirror the contractions.

Side effects of MgSO4

drowsiness and hot flashes

Perceptive (sensorineural loss)

dt disorder of the organ of Corti or the auditory nerve. sounds being transmitted but not able to read the message

Tx for Vtach if pulseless

defib, vasopressin, epi, lido, amiodaron

TX for Vfib

defibrillation monophasic 360J or biphasic 120-200J if unknown 200 J

Myasthenia gravis: diagnosis

deficeny of ACTH at myoneural junction; based on administration of anticholinesterase (neostigmine); positive if increase in muscle strength

Parkinson's

deficiency in dopamine, which impairs coordination and autonomic function

Parkinsons Disease: Diagnosis

deficieny of dopamine; increased acetylcholine levels; Does not lead to paralysis

Positive sign

definite sign of pregnancy: FHR (8-12 wks) auscultation 18-20 wks,

>1.020

dehydration, olguria, blood in urine

Symptoms of DHD in the older child...

delays in walking, presence of extra gluteal folds, positive Trendelenburg sign when weight bearing

Symptoms of stage 4 (3-5 days after last use)

delirium tremens including confusion, insomnia, agitation, hallucinations, and uncontrolled tachycardia

third stage of labor

delivery of the placenta

Bipolar: Assessment of mania

delusion of grandeur, cheerful, euphoria, flight of ideas, easily distractable, inappropriate dress, excessive make and jewelry, cant stop moving to eat

Positive symptoms of schizophrenia are those such as...

delusions and hallucinations.

dull

dense fluid filled tissue

Symptoms associated with GAD...

depression, somatization or development of phobias, worries excessively over everything, retreats from anxiety-producing situations, self-medication with drugs or alcohol

descending or sigmoid colostomy:

descending or sigmoid colon (usually permanent. descending produce firm stool. sigmoid product solid stool.

calcium rich foods include:

dairy products, lentils, fish w/ fine bones, dried fruits, nuts, chocolate, cocoa, Ovaltine

oxalate rich foods include:

dark roughage, spinach, rhubarb, asparagus, cabbage, tomatoes, beets, nuts, celery, parsley, runner beans, chocolate, instant coffee, Ovaltine, tea, worcestershire sauce

Meniere's Disease: treatment

decompression of the endolymphatic sac with teflon shunt Labyrinthectomy-removal of labyrinth can cause Bell's palsy

coombs test

detect antibodies attached to circulating erythrocytes; performed on cord sample. (Rh or ABO factors)

Oxytocin challenge test (contraction stress test)

determine fetal response to contractions; 90-120 minutes; Pitocin used and increased every 15 minutes until three contractions in 10 minutes

Non-stress test

determine fetal response to cyclical periods of rest and activity; fetal monitor applied for approximately 90 minutes; press response button each time baby moves

hemoglobin

determines the amount of hemoglobin in 100 ml of blood

Conversion

development of physical symptoms in response to inner conflict

Diagnosis of meningitis

diagnosed by lumbar puncture where the CSF is analyzed for organisms.

anterior fontanel close

diamond shaped close 18 months (junction of sagital, coronal, and frontal sutures)

Risk factor for colorectal cancer:

diet high in fat, high in protein, low on fiber

Dystocia

difficult or extremely painful labor and delivery

dysarthia

difficult poorly articulated speech resulting from interference in the control over the muscles of speech

dysphasia

difficulty swallowing

dysgraphia

difficulty writing

Treatment for HELLP

early delivery of fetus

Intestinal ostomies for fecal diversion: drainage and description

first 1-2 days: mucus and serousanguineous begins to work 3-6 days post op

Fourth stage:

first 4 hrs after delivery

Intestinal ostomies for fecal diversion: observation and condition of stoma

first few days appears beefy-red and swollen; gradually swelling receds and color is pink or red; notify physician if stoma is dark blue "blackish or purple" my indicate insufficient blood supply.

S/s of pyleonephritis are the same as UTI except for ____

flank pain and/or pain at the costovertebral angle

Following infratentorial crainiotomy, position the client...

flat on either side. Use this position for 24-48 hours after surgery. If bone flap has been removed keep the client off of the operative side.

brudzinski's sign

flexing of the hip and knee when neck is flexed

Characteristic of Proteinuria:

foamy urine due to excess protein

Dumping Syndrome: assessment

food into the jejunum before it has had the chance to begin the digestive process. Diaphoresis, weakness, diarrhea, and feeling of fullness

Nerve block: Epidural block

for uterine labor pain; does not suppress FHR, not complete anesthesia; able to assist with pushing but relatively free of pain; maternal hypotension is complication; increase fluid after block; shivering is expected

What can you do to reverse SVT?

form a vagal or valsalva maneuver, ice to the face, holding breath and then bearing down, massaging the carotid artery, IV adenosine (adenocard or adenoscan

3 phases of labor: increment, acme/peak, decrement

increment: beginning of the contraction until its peak acme/peak:strongest intensity decrement:diminishing intensity

solitary play is at what age

infant

macrosomia

large for gestational age but may have immature organ systems

Newborns of diabetic mothers tend to be...

large for gestational age. At birth they appear pudgy, ruddy, and lethargic.

Symptoms of major depression include...

lasting at least two weeks; feelings of worthlessness, diminished ability to concentrate, anorexia, sleep disturbances, and recurrent thoughts of death or suicide

Fetal bradycardia <110-120 bpm

late sign of fetal hypoxia, prolonged cord compression, maternal supine hypotensive syndrome

Symptoms of DHD in the infant 2-3 months of age...

laxity in the hip joint and the presence of the Ortolani click, shortening of the affected limb (Allis sign) and asymmetrical thigh and gluteal folds

Presumptive signs of pregnancy are those that...

lead the client to believe she is pregnant but that are not conclusive.

LOA

left occiput anterior; back of baby's head is anterior to the pelvis and tilted to the left side

Normal ICP

less than 20 mm Hg

station:

level of presenting part of fetus in relation to imaginary line between ischial spines (zero station) (-1 to -5=floating) engagement is 0 or below

Tx for Vtach with pulse

lidocaie and cardiovert

At 1-3 months, the infant can...

lift the head, grasp and hold objects for a brief period of time, and roll from side to back. The infant can focus on objects. The infant smiles, recognizes primary caregiver, and vocalizes cooing.

Bipolar: Mood-stabilizer Medications

lithium, valproic acid, carbamazepine

s/s of IV infection:

local- redness, pain, drainage @ site systemic - fever, chills, elevated WBC, shock

Symptoms of MS include...

loss of bowel and bladder control, blurry vision, paralysis

Reye's syndrome: mediation and tx

mannitol to decrease ICP, Diuretics (lasix) to decrease CSF production, Dilantin for anticonvulsants, FFP Vitamin K platelet transfusion for overt or covert bleeding

Abruptio Placentae (premature separation of placenta) caused by

maternal HTN and cocaine abuse.

Complications of MgSO4

maternal hypotension, oliguria, apnea

quickening

maternal perception of fetal movement occurring between 16-20 wks

Ovulation:

maturation and release of egg from ovary; generally occurs 14 days before beginning of next menses

hematocrit

measerues the percentage or RBS per fluid volume of blood

Fundal height EDB

measurement of fundal height from the top of symphysis pubis to the top of the fundus with a flexible tape. at symphis 12-14 wks at umbilicus 20 wk

antacid:

medication that neutralizes acid that's already been made

Brethine (terbutaline sulfate)

medication used to stop contractions; bronchodilator contraindicated in clients with CVD because it causes tachycardia and in clients with diabetes because it elevated blood glucose levels

Antipsychotic medication carries the risk of...

neuroleptic malignant syndrome.

Akathesia

pacing and rocking back and forth when sitting

ectopic pregnancy: diagnosis

pain due to implantation of egg outside of uterus; caused by PID, congenital anomalies of fallopian tubes

Brudzinski's Sign:

pain with resistance and involuntary flex of hip/knee when neck is flexed to chest when lying supine

Placenta previa produces...

painless bleeding.

Meckel's Diverticulum: assessment

painless rectal bleeding, abdominal pain, Hematochezia(jelly-like stool)

Condylomata lesions are...

painless warts.

s/s of phlebitis:

palpable cord, pain, redness

If serum amylase and lipase are high what might this indicate?

pancreatitis

Appendicitis: Assessment

periumbilical abdominal pain shifts to RLQ at McBurney's point (located between umbilicus and the right iliac crest), localized tenderness, muscles gaurding, low-grade fever

delusions

persisten false beliefs grandeur: belief that one is special, persecutory:belief that one is victim of a plot ideas of reference: belief that environment events are directed toward the self. sees people talking and believe they are talking about them

Oppositional defiant disorder is characterized by...

persistent patterns of negativistic, hostile, and defiant behavior; does not violate the rights of others; argumentative, uncooperative, annoying, and spiteful

from the beginning of labor until the cervix is completely dilated is the first stage:

phase 1 (0-3cm) phase 2 (4-7cm_ phase 3 (8 -10cm)

First stages of labor: beginning of labor to complete cervical dilation

phase 1 (latent): 0-3 cm phase 2 (active) 4-7 cm phase 3 (transition) 8-10

During a manic episode, the client experiences...

profound changes in mood described as elevated, expansive, or irritable; delusions of grandeur, flight of ideas, increased motor activity, increased risk taking and promiscuity, use of profanity, uncontrolled spending, failing to sleep or eat for long periods of time

effacement

progressive thinning and shortening of cervix (0-100%)

para

refers to past pregnancies that reached viability 20-22 wks or later

attitude

relationship of fetal parts to each other usually flexion head and extremities on chest and abdomen to accommodate to shape of uterine cavity

Fetal lie

relationship of fetus to long axis of mother; determined by performing Leopold's maneuvers

lie:

relationship of spine of feus to spine of mother: longitudinal (parallel), transverse (right angle), oblique ( slight angle off a true transverse lie)

Station

relationship of the presenting part to the maternal ischial spines (0 station is at the ischial spines)

A viral load of less than 400 copies/mL indicates the client is...

relatively free of circulating virus

Bowel surgery purpose for intestinal obstruction:

removal of disease portion of bowel; creation of an outlet for passage of stool when there is an obstruction or need for bowel rest.

Billroth II

removal of distal segment of stomach and antrum

Traditional cholecystectomy-

removal of gall bladder through a high abdominal incision

laparoscopic laser cholecystectomy:

removal of gallbladder by a laser through laparoscope; laparoscope is attached to video camera and procedure is viewed through monitor; four small puncture holes made in abdomen

At 19-24 months, the toddler can...

run, jump in place, throw a ball overhand, kick a ball forward, walk up and down steps with two feet on each step, build a tower of four to six cubes, copy a vertical line, wash and dry hands, gains bowel control, helps dress himself. Understandable speech, can combine three or four words, has vocabulary of 300 works, can name pictures.

Pt has severe pain in periumbilical area that gets increasing worse and then goes away, what may have just happened?

ruptured appendix

INTRACRANIAL TUMORS : S/S Complication Crisis

s/s: Depends on location: visual, motor, neurologic, GI HEADACHE AND SEIZURE cognitive dysfunction, muscle weakness, sensory losses, aphasia, increased ICP, cerebral edema, obstruction of CSF pathways. Crisis prevention: Prevent Intracranial Pressure, remove tumor if possible.

MENINGITIS : S/S Complication Crisis

s/s: Severe HA, fever, delirium, Nuchal Rigidity: stiff neck Kernig's Sign: from bent leg/knee to strait is painful Brudzinski's Sign: pain; resistance and involuntary flex of hip/knee when neck is flexed to chest when lying supine Photo/Phonophobia; Increased ICP Edema and inflammation of the optic nerve Purpuric rash on the skin and mucous membranes Assess for IICP: LOC, VS, Eyes, Motor function Crisis: COMA- Acute complication from IICP SEIZURES- Acute cerebral edema/ IICP

The client with schizotypal personality disorder is...

shy, aloof, and withdrawn; display a more bizarre way of thinking; are sensitive to the reactions of and possible rejection of others; avoid social situations

if pt needs a blood transfusion what must be obtained?

signed consent

Bipolar: plan

simplify the environment and decrease environment stimuli, assign to sign room, keep nosie level down, limit people, distract and redirect energy,

Bradykinesia

slowed movement

Treatment of MS includes...

supportive care and medications (steroids, Immuran, interferons such as Rebif, glatiramer acetate (Copaxone), baclofen (Lioresal), and mitoxantrone (Novantrone))

Pill

suppresses ovulation by suppressing production of FSH and LH; effectiveness decreased if used with barbiturates, phenytoin, antibiotics

Venereal Disease Research Lab (VDRL) and Rapid plasma reagin (RPR) are screening tests done for...

syphilis.

TORCH:

t-toxoplasmosis ( avoid eating undercooked meat and cat litter) o-other (syphilis, varicella/shingles, group b, hepatitis a & b, AIDS r-rubella c-cytomegalovirus H-herpes

Signs of cannabis abuse...

tachycardia, increased appetite, euphoria, slowed perception of time

Posterior

tall r waves in v1-v2

In children with hirschusprung disease, the use of...

tap water, concentrated salt solutions, soap solutions, or phosphate preparations is discouraged because frequent use of nonisotonic solution can lead to water intoxication and the dilution of serum electrolytes

Detached Retina: preoperative care

teach postop need to avoid straining at stool, stooping

EGD pt what must be checked every 15 to 30 mins for 1-2 hrs and why

temp must be checked a spike could mean possible perforation

Glaucoma:Angle-closure (closed angle)

the angle of the iris obstruct aqueous humor

If pt has renal insufficiency what would you assess for?

this is end-stage kidney disease, kidneys aren't functioning; assess for psychosocial changes - depression, anxiety, ability to cope, suicide, withdraw from loved ones

ways to promote Nonnursing women

tight fitting bra for 72 hrs, ice pack minimize breast stimulation

Avoidant personality disorder is used to describe clients who are...

timid, withdrawn, and hypersensitive to criticism; feel socially inadequate, so they avoid situations in which they might be rejected

Milia

tiny, white bumps that occur across the newborn's nose

When is a lumbar puncture indicated?

to determine if there is an infection in the spine (CNS) such as meningitis.

Colostomy irrigations for sigmoid colostomy: purpose

to stimulate emptying of colon of gas, mucus, feces at scheduled times to avoid need for appliance. usually begin 5-7 post op. performed after meal same time each day.

parellel play is at what age

toddlers

At 9-12 years, the child uses...

tools and equipment well, follows direction, is enthusiastic at work and play, looks for ways to earn money; loves secrets, participates in cooperative play or activites; play activity is mostly with same-sex groups, enjoy board games, video games, music, and sporting activities that are shared with others

TORCHS

toxoplasmosis, rubella, cytomegalovirus, herpes, syphillis

Tx for @nd degree av block type 2

trancutaneous pacer, temprary pacer, or permanent pacer, atropine

Phase 3 labor

transition (8-10 cm dilation)

Early decelerations

transitory drops in the fetal heart rate caused by head compression

transverse colostomy

transverse colon (usually temporary). ostomy return is firm, very foul odor, pouch is worn continuously

What is empiric treatment?

treatment that is started before C & S comes back with definitive cause of infection.

Placenta Previa: plan/implementation

trendelenburg position for at least 72 hrs. No vaginal rectal exam unless delivery would not be a problem.

posterior fontanel close

triangular shaped closes 6-8 wks

metronidazole (flagyl) tx for:

trichomonas vaginilis

In the first year of life, body weight...

triples and length increases by 50%

Maalox & Mylanta commonly interfere w/absorption of other meds. T or F

true - especially when kidneys have failed

When feeding someone with a stroke or brain injury it may be better to have them ________________ while eating.

tuck their chin.

acoustic neuroma

tumor of the 8th cranial nerve

During initiation of lithium therapy, levels should be drawn...

twice weekly and then every 2-3 months during long-term therapy.

Assessment of CNVIII

use Weber and Rinne test for hearing loss

Anteroseptal

v1-v3

42. The nurse is assessing an 80-year-old male patient. Which assessment findings would be considered normal?

C) The presence of kyphosis and flexion in the knees and hips

72. The nurse is taking the history of a patient who may have a perforated eardrum. What would be an important question in this situation? A) "Do you ever notice ringing or crackling in your ears?" B) "When was the last time you had your hearing checked?" C) "Have you ever been told you have any type of hearing loss?" D) "Was there any relationship between the ear pain and the discharge you mentioned?"

D) "Was there any relationship between the ear pain and the discharge you mentioned?" Pages: 327-328. Typically with perforation, ear pain occurs first, stopping with a popping sensation, and then drainage occurs.

18. When providing culturally competent care, nurses must incorporate cultural assessments into their health assessments. Which statement is most appropriate to use when initiating an assessment of cultural beliefs with an elderly American Indian patient?

D) "What cultural or spiritual beliefs are important to you?"

At 2-3 years, the toddler can...

balance on one foot, jump with both feet, take a few steps on tiptoe, ride a tricycle, build tower of eight cubes, copy vertical and horizontal lines, has day time bladder control; can give first and last name, name a friend, and one color; 900 words, plays beside another child (parallel play), nesting toys, picture books, push-pull toys, riding toys, pounding boards, sand, soap bubbles, talking toys, balls, dolls, and dress-up

Hyperventilation per mechanical ventilation is used to...

constrict cerebral blood vessels in the client with increased ICP. Decreases PCO2 to 27-30

Giardia lamblia: source

contaminated water

True Labor signs:

contractions at regular interval with increasing frequency, duration and intensity, discomfort radiates from back around the abdomen, contraction do no decrease with rest, cervix effaced and dilated.

False labor:

contractions irregular; discomfort is usually abdomen, contraction lessen with activity or rest, cervical not changed

Somatoform disorders include...

conversion disorder, hypochondriasis, pain disorder, somatization disorder

Variable decelerations are caused by...

cord compression. Prolapsed cord or nuchal cord.

Naegeles rule

count back 3 months from first day of last menstrual period and add 7 days and 1 year

Preparing infant for phototherapy

cover eyes and genitals; increase fluids and feedings to facilitate excretion of bilirubin through GI tract and urinary system

By 6 months of age, new foods are added to the infants diet, including...

crackers, melba toast, rice cereal, vegetables, fruits, meat, and egg yolk.

Developmental hip dysplasia (DHD) is found more often in groups that use...

cradle boards or papoose boards for carrying the infant (such as Native Americans) than those groups that carry the infant on the back or on the hips (such as Asians)

At 12 months, the infant...

cruises well, can walk with one hand being held, begins to take first steps alone, can sit down from a standing position unassisted, can turn pages in a book, and recognizes familiar pictures like animals. Use of pincer grasp. Six to eight deciduous teeth. Shows affection (blows kisses on request), explores, seeks security blanket or favorite toy when upset. Plays alone and enjoys mobiles, busy boxes, soft cuddle toys, and soft picture books.

Vagotomy

cutting the vagus nerve ( decrease the HCl secretion)

Hiatal Hernia: tx

cytoprotective agents: protect the lining of the GI tract (carafate). Surgery to tighten cardac sphincter (fundoplication)

Cesarean Delivery assessment:

dystocia (difficult labor) previous c-section, breech presentation, active gonorrhea or herpes type 2 infection, prolapsed cord, planceta previa, abruptio placenta

Phase 1 labor

early labor or prodromal (0-3 cm dilation)

Fetal tachycardia >160 bpm

early sign of fetal hypoxia, nonreassuring sign

Ballottement

easy flexion of the uterus when the examiner's finger pushes against the uterus and detects the presence of the fetus by return impact

Ulcer: plan/implementation

eat 3 meals/day; avoid extreme temp; avoid coffee, alcohol caffeinated beverages, milk, cream, STOP smoking

Atrial Fibrillation

ectopic foci firing making atrium depolarize @ rates of 350-600bpm P:QRS ratio is (many:1) RIsk for emboli

Caput succedaneum

edema that crosses the suture line on the baby's scalp

Bipolar: diagnosis

elation of grandiosity can be a defense against underlying depression or feelings of low-self esteem.

Symptoms of EA and TEF include...

presence of maternal polyhydramnios (excessive amniotic fluid); excessive mucus and drooling in the newborn; coughing, choking, and cyanosis with the first feeding; x-ray blind pouch at each end widely separated with no connection (EA) or presence of abnormal connection between trachea and esophagus (TEF)

What is Steven-Johnson's Syndrome?

severe allergic reaction to meds flu-like symptoms, followed by a painful red or purplish rash leads to top layer of skin necrosising if pt develops lesions in the lungs, death may result.

1. A client with no history of cardiovascular disease comes to the ambulatory clinic with flu-like symptoms. The client suddenly complains of chest pain. Which of the following questions would best help a nurse discriminate pain caused by a noncardiac problem? 1. "Can you describe the pain to me?" 2. "Have you ever had this pain before?" 3. "Does the pain get worse when you breathe in?" 4. "Can you rate the pain on a scale of 1 to 10, with 10 being the worst?"

1. 3 Rationale: Chest pain is assessed by using the standard pain assessment parameters (e.g., characteristics, location, intensity, duration, precipitating and alleviating factors, and associated symptoms). Options 1, 2, and 4 may or may not help discriminate the origin of pain. Pain of pleuropulmonary origin usually worsens on inspiration. Test-Taking Strategy Use the process of elimination, focusing on the subject, pain resulting from a noncardiac problem. The three incorrect options, although appropriate to use in practice, are general assessment questions only. Option 3 will discriminate between a cardiac and noncardiac cause of pain. Review pain assessment measures for the client with a cardiovascular problem if you had difficulty with this question.

Crohn's Dz:

ACUTE Lining (and deaper) of the digestive tract have PATCHES of inflammation/ulceration that occur anywhere in the digestive tract and often spreads deep into the layers of affected tissues. S/S: Relapsing/Remitting- Severe diarrhea, abdominal pain, malnutrition, blood in stool, ulcers.

21. A client is at risk for pulmonary embolism and is on anticoagulant therapy with warfarin sodium (Coumadin). The client's prothrombin time is 20 seconds, with a control of 11 seconds. How would the nurse interpret these results? 1. Client needs to have test repeated. 2. Client results are within the therapeutic range. 3. Client results are higher than the therapeutic range. 4. Client results are lower than the needed therapeutic level.

21. 2 Rationale: The therapeutic range for prothrombin time is 1.5 to 2 times the control for clients at high risk for thrombus. Based on the client's control value, the therapeutic range for this individual would be 16.5 to 22 seconds. Therefore the result is within the therapeutic range. Test-Taking Strategy: Use the process of elimination. Look at the control value. Remembering that the purpose of anticoagulant therapy is to prolong clotting times will assist in eliminating options 3 and 4. Eliminate option 1, because there is no basis for repeating the test. Because the prothrombin value identified in the question is not even double the control, select option 2 from the remaining options. Review the therapeutic prothrombin level for a client at risk for pulmonary embolism if you had difficulty with this question.

21. A nurse is caring for a client with unstable ventricular tachycardia. The nurse instructs the client to do which of the following, if prescribed, during an episode of ventricular tachycardia? 1. Lie down flat in bed. 2. Remove any metal jewelry. 3. Breathe deeply, regularly, and easily. 4. Inhale deeply and cough forcefully every 1 to 3 seconds.

21. 4 Rationale: Cough cardiopulmonary resuscitation (CPR) sometimes is used in the client with unstable ventricular tachycardia. The nurse tells the client to use cough CPR, if prescribed, by inhaling deeply and coughing forcefully every 1 to 3 seconds. Cough CPR may terminate the dysrhythmia or sustain the cerebral and coronary circulation for a short time until other measures can be implemented. Options 1, 2, and 3 will not assist in terminating the dysrhythmia. Test-Taking Strategy To answer this question, you must be familiar with the treatment for unstable ventricular tachycardia. Remember that cough CPR sometimes is used in the client with unstable ventricular tachycardia. Review the concept of cough CPR if you are not familiar with it.

22. A client is having frequent premature ventricular contractions. A nurse would place priority on assessment of which of the following? 1. Sensation of palpitations 2. Causative factors, such as caffeine 3. Precipitating factors, such as infection 4. Blood pressure and oxygen saturation

22. 4 Rationale: Premature ventricular contractions can cause hemodynamic compromise. The shortened ventricular filling time with the ectopic beat leads to decreased stroke volume and, if frequent enough, to decreased cardiac output. The client may be asymptomatic or may feel palpitations. Premature ventricular contractions can be caused by cardiac disorders, states of hypoxemia or by any number of physiological stressors, such as infection, illness, surgery, or trauma, and by intake of caffeine, nicotine, or alcohol. Test-Taking Strategy Note the strategic words priority on assessment. Use the ABCs—airway, breathing, and circulation—to direct you to option 4. Review the effects of premature ventricular contractions if you had difficulty with this question.

23. A client has developed atrial fibrillation, with a ventricular rate of 150 beats/min. The nurse should assess the client for which associated signs or symptoms? 1. Flat neck veins 2. Nausea and vomiting 3. Hypotension and dizziness 4. Hypertension and headache

23. 3 Rationale: The client with uncontrolled atrial fibrillation with a ventricular rate more than 100 beats/min is at risk for low cardiac output because of loss of atrial kick. The nurse assesses the client for palpitations, chest pain or discomfort, hypotension, pulse deficit, fatigue, weakness, dizziness, syncope, shortness of breath, and distended neck veins. Test-Taking Strategy Use the process of elimination. Flat neck veins are normal or indicate hypovolemia, so eliminate option 1. Nausea and vomiting (option 2) are associated with vagus nerve activity and do not correlate with a tachycardic state. From the remaining options, think of the consequences of falling cardiac output to direct you to option 3. Review the effects of atrial fibrillation if you had difficulty with this question.

24. Which of the following would be an expected outcome of nesiritide (Natrecor) administration? 1. Client will have an increase in urine output. 2. Client will have an absence of dysrhythmias. 3. Client will have an increase in blood pressure. 4. Client will have an increase in pulmonary capillary wedge pressure.

24. 1 Rationale: Nesiritide is a recombinant version of human B-type natriuretic peptide, which vasodilates arteries and veins. It is used for the treatment of decompensated heart failure, increases renal glomerular filtration, and increases urine output. Options 2, 3, and 4 are incorrect. Test-Taking Strategy: Use the process of elimination. Nesiritide does not have antidysrhythmic properties. Dysrhythmias may be a side effect of the medication, so option 2 should be eliminated. Eliminate option 3 because the medication is a vasodilator and causes a decrease in blood pressure. Eliminate option 4 because the medication decreases pulmonary capillary wedge pressure (PCWP). Review the effects of this medication if you had difficulty with this question.

Nursing care for the client with schizophrenia include...

providing a quiet, supportive environment; establishing a trusting relationship; administering antipsychotic medication; observing for side effects of medication; assisting with ADLs; attending to client's physical needs, including nutrition and hydration

31. A nurse is performing cardiopulmonary resuscitation on a client who has had a cardiac arrest. An automatic external defibrillator is available to treat the client. Which of the following activities will allow the nurse to assess the client's cardiac rhythm? 1. Hold the defibrillator paddles firmly against the chest. 2. Apply adhesive patch electrodes to the chest and move away from the client. 3. Apply standard electrocardiographic monitoring leads to the client and observe the rhythm. 4. Connect standard electrocardiographic electrodes to a transtelephonic monitoring device.

31. 2 Rationale: The nurse or rescuer puts two large adhesive patch electrodes on the client's chest in the usual defibrillator positions. The nurse stops cardiopulmonary resuscitation and orders anyone near the client to move away and not touch the client. The defibrillator then analyzes the rhythm, which may take up to 30 seconds. The machine then indicates if defibrillation is necessary. Test-Taking Strategy Use the process of elimination. If you are not familiar with this piece of equipment, look first at the word automatic in the name. This implies that a person is not as involved in the process as with a conventional defibrillator and will help eliminate option 1. Because standard electrocardiogram monitoring leads do not play an active role once resuscitation is underway (options 3 and 4), you can eliminate these comparative or alike options. Review the procedure related to the use of an automatic external defibrillator if you had difficulty with this question.

32. A nurse employed in a cardiac unit determines that which of the following clients is the least likely to have implantation of an automatic internal cardioverter-defibrillator (AICD)? 1. A client with syncopal episodes related to ventricular tachycardia 2. A client with ventricular dysrhythmias despite medication therapy 3. A client with an episode of cardiac arrest related to myocardial infarction 4. A client with three episodes of cardiac arrest unrelated to myocardial infarction

32. 3 Rationale: An automatic internal cardioverter-defibrillator (AICD) detects and delivers an electrical shock to terminate life-threatening episodes of ventricular tachycardia and ventricular fibrillation. These devices are implanted in clients who are considered high risk, including those who have survived sudden cardiac death unrelated to myocardial infarction, those who are refractive to medication therapy, and those who have syncopal episodes related to ventricular tachycardia. Test-Taking Strategy Use the process of elimination and note the strategic words least likely. Ventricular dysrhythmias that induce syncope or occur while the client is on medication are likely to be true indications for the AICD, so eliminate options 1 and 2 first. From the remaining options, the main difference is whether or not the cardiac arrest was related to myocardial infarction. Of these two, the one most likely to be responsive to AICD would be the client without myocardial infarction because those dysrhythmias are spontaneous. Review the indications for the use of an AICD, if you had difficulty with this question.

33. A nurse is caring for a client who has just had implantation of an automatic internal cardioverter-defibrillator. The nurse immediately would assess which of the following items based on priority? 1. Anxiety level of the client and family 2. Presence of a Medic-Alert card for the client to carry 3. Knowledge of restrictions of postdischarge physical activity 4. Activation status of the device, heart rate cutoff, and number of shocks it is programmed to deliver

33. 4 Rationale: The nurse who is caring for the client after insertion of an automatic internal cardioverter-defibrillator needs to assess device settings, similar to after insertion of a permanent pacemaker. Specifically, the nurse needs to know whether the device is activated, the heart rate cutoff above which it will fire, and the number of shocks it is programmed to deliver. Options 1, 2, and 3 are also nursing interventions but are not the priority. Test-Taking Strategy Use Maslow's Hierarchy of Needs theory. Option 4 is the option that identifies the physiological need. Review care to the client following insertion of an automatic internal cardioverter-defibrillator if you had difficulty with this question.

38. Postoperatively, a nurse is caring for a client who had a percutaneous insertion of an inferior vena cava filter and was on heparin therapy before surgery. The nurse would inspect the surgical site most closely for evidence of which of the following? 1. Bleeding and infection 2. Thrombosis and infection 3. Bleeding and wound dehiscence 4. Wound dehiscence and evisceration

38. 1 Rationale: After inferior vena cava filter insertion, the nurse inspects the surgical site for bleeding and signs and symptoms of infection. Otherwise, care is the same as for any other postoperative client. Test-Taking Strategy Use the process of elimination. Because inferior vena cava filters are inserted percutaneously through a deep vein, options 3 and 4 are eliminated because no abdominal incision is made. From the remaining options, noting that the client has been on anticoagulant therapy before surgery because of the high risk of pulmonary embolism will direct you to option 1. Review care of the client following insertion of an inferior vena cava filter if you had difficulty with this question.

41. A nurse is assessing the neurovascular status of a client who returned to the surgical nursing unit 4 hours ago after undergoing aortoiliac bypass graft. The affected leg is warm, and the nurse notes redness and edema. The pedal pulse is palpable and unchanged from admission. How would the nurse correctly interpret the client's neurovascular status? 1. The neurovascular status is normal because of increased blood flow through the leg. 2. The neurovascular status is moderately impaired, and the surgeon should be called. 3. The neurovascular status is slightly deteriorating and should be monitored for another hour. 4. The neurovascular status is adequate from an arterial approach, but venous complications are arising.

41. 1 Rationale: An expected outcome of surgery is warmth, redness, and edema in the surgical extremity because of increased blood flow. Therefore, options 2, 3, and 4 are incorrect interpretations. Test-Taking Strategy Use the process of elimination. Option 2 can be eliminated because the pedal pulse is unchanged from admission. Venous complications from immobilization resulting from surgery would not be apparent within 4 hours, so eliminate option 4. From the remaining options, think about the effects of sudden reperfusion in an ischemic limb. There would be redness from new blood flow and edema from the sudden change in pressure in the blood vessels. Review the expected assessment findings following this surgical procedure if you had difficulty with this question.

88. During palpation of the anterior chest wall, the nurse notices a coarse, crackling sensation over the skin surface. On the basis of these findings, the nurse suspects: A) tactile fremitus. B) crepitus. C) friction rub. D) adventitious sounds.

B) crepitus. Page: 424. Crepitus is a coarse, crackling sensation palpable over the skin surface. It occurs in subcutaneous emphysema when air escapes from the lung and enters the subcutaneous tissue, as after open thoracic injury or surgery.

6. A nurse notes bilateral 12 edema in the lower extremities of a client with myocardial infarction who was admitted 2 days ago. The nurse would plan to do which of the following next? 1. Order daily weights starting on the following morning. 2. Review the intake and output records for the last 2 days. 3. Request a sodium restriction of 1 g/day from the physician. 4. Change the time of diuretic administration from morning to evening.

6. 2 Rationale: Edema, the accumulation of excess fluid in the interstitial spaces, can be measured by intake greater than output and by a sudden increase in weight. Diuretics should be given in the morning whenever possible to avoid nocturia. Strict sodium restrictions are reserved for clients with severe symptoms. Test-Taking Strategy Use the process of elimination, noting the strategic word next. Use the steps of the nursing process to prioritize. Option 2 is the only option that addresses assessment of data. Review care of the client with a myocardial infarction if you had difficulty with this question.

693. A client admitted to the hospital with chest pain and history of type 2 diabetes mellitus is scheduled for cardiac catheterization. Which of the following medications would need to be withheld for 48 hours before and after the procedure? 1. Regular insulin 2. Glipizide (Glucotrol) 3. Repaglinide (Prandin) 4. Metformin (Glucophage)

693. 4 Rationale: Metformin (Glucophage) needs to be withheld 48 hours before and after cardiac catheterization because of the injection of contrast medium during the procedure. If the contrast medium affects kidney function, with metformin in the system, the client would be at increased risk for lactic acidosis. The medications in options 1, 2, and 3 do not need to be withheld 48 hours before or after cardiac catheterization. Test-Taking Strategy: Use the process of elimination. Eliminate options 2 and 3 first. Although these medications may be withheld on the morning of the procedure because of the client's NPO status, there is no indication for withholding the medication the day prior to and the day postprocedure. Regular insulin may be administered if elevated blood glucose levels from infused intravenous solutions occur on the day of the procedure. Review preprocedure and postprocedure interventions if you had difficulty with this question.

694. A client is in sinus bradycardia with a heart rate of 45 beats/min, complains of dizziness, and has a blood pressure of 82/60 mm Hg. Which of the following should the nurse anticipate will be prescribed? 1. Defibrillate the client. 2. Administer digoxin (Lanoxin). 3. Continue to monitor the client. 4. Prepare for transcutaneous pacing.

694. 4 Rationale: Hypotension and dizziness are signs of decreased cardiac output. Transcutaneous pacing provides a temporary measure to increase the heart rate and thus perfusion in the symptomatic client. Digoxin will further decrease the client's heart rate. Defibrillation is used for treatment of pulseless ventricular tachycardia and ventricular fibrillation. Continuing to monitor the client delays necessary intervention. Test-Taking Strategy: Use the process of elimination. Eliminate option 3 because the client is symptomatic and requires intervention. Option 2 is eliminated because digoxin will further decrease the client's heart rate. Defibrillation is used for treatment of pulseless ventricular tachycardia and ventricular fibrillation; therefore eliminate option 1. Review the indications for transcutaneous pacing if you had difficulty with this question.

696. A client with myocardial infarction suddenly becomes tachycardic, shows signs of air hunger, and begins coughing frothy, pink-tinged sputum. Which of the following would the nurse anticipate when auscultating the client's breath sounds? 1. Stridor 2. Crackles 3. Scattered rhonchi 4. Diminished breath sounds

696. 2 Rationale: Pulmonary edema is characterized by extreme breathlessness, dyspnea, air hunger, and the production of frothy, pink-tinged sputum. Auscultation of the lungs reveals crackles. Rhonchi and diminished breath sounds are not associated with pulmonary edema. Stridor is a crowing sound associated with laryngospasm or edema of the upper airway. Test-Taking Strategy: Use the process of elimination. Recalling that fluid produces sounds that are called crackles will assist you in eliminating options 1, 3, and 4. If you had difficulty with this question, review the manifestations found in pulmonary edema.

697. A client with myocardial infarction is developing cardiogenic shock. Because of the risk of myocardial ischemia, for which of the following should the nurse carefully assess the client? 1. Bradycardia 2. Ventricular dysrhythmias 3. Rising diastolic blood pressure 4. Falling central venous pressure

697. 2 Rationale: Classic signs of cardiogenic shock as they relate to this question include low blood pressure and tachycardia. The central venous pressure would rise as the backward effects of the severe left ventricular failure became apparent. Dysrhythmias commonly occur as a result of decreased oxygenation and severe damage to greater than 40% of the myocardium. Test-Taking Strategy: Use the process of elimination and focus on the strategic words myocardial ischemia. Recall that ischemia makes the myocardium irritable, producing dysrhythmias. Also, knowledge of the classic signs of shock helps eliminate the incorrect options. Review the clinical manifestations associated with cardiogenic shock if you had difficulty with this question.

698. A client who had cardiac surgery 24 hours ago has a urine output averaging 20 mL/hr for 2 hours. The client received a single bolus of 500 mL of intravenous fluid. Urine output for the subsequent hour was 25 mL. Daily laboratory results indicate that the blood urea nitrogen level is 45 mg/dL and the serum creatinine level is 2.2 mg/dL. Based on these findings, the nurse would anticipate that the client is at risk for which of the following? 1. Hypovolemia 2. Acute renal failure 3. Glomerulonephritis 4. Urinary tract infection

698. 2 Rationale: The client who undergoes cardiac surgery is at risk for renal injury from poor perfusion, hemolysis, low cardiac output, or vasopressor medication therapy. Renal insult is signaled by decreased urine output and increased blood urea nitrogen and creatinine levels. The client may need medications to increase renal perfusion and possibly could need peritoneal dialysis or hemodialysis. No data in the question indicate the presence of hypovolemia, urinary tract infection, or glomerulonephritis. Test-Taking Strategy: Use the process of elimination. Eliminate options 3 and 4 first because no data indicate infection or inflammation. Noting that the urine output is inadequate will assist you in eliminating option 1. Review the complications associated with cardiac surgery if you had difficulty with this question.

105. A patient has hypoactive bowel sounds. The nurse knows that a potential cause of hypoactive bowel sounds is: A) diarrhea. B) peritonitis. C) laxative use. D) gastroenteritis.

B) peritonitis. Page: 561. Diminished or absent bowel sounds signal decreased motility from inflammation as seen with peritonitis, with paralytic ileus after abdominal surgery, or with late bowel obstruction.

7. A nurse is conducting a health history of a client with a primary diagnosis of heart failure. Which of the following disorders reported by the client is unlikely to play a role in exacerbating the heart failure? 1. Atrial fibrillation 2. Nutritional anemia 3. Peptic ulcer disease 4. Recent upper respiratory infection

7. 3 Rationale: Heart failure is precipitated or exacerbated by physical or emotional stress, dysrhythmias, infections, anemia, thyroid disorders, pregnancy, Paget's disease, nutritional deficiencies (thiamine, alcoholism), pulmonary disease, and hypervolemia. Test-Taking Strategy Use the process of elimination and note the strategic word unlikely. Remembering that heart failure is exacerbated by factors that increase the workload of the heart will assist you in eliminating options 1, 2, and 4. Review the precipitating factors associated with heart failure if you had difficulty with this question.

700. A client is wearing a continuous cardiac monitor, which begins to sound its alarm. A nurse sees no electrocardiographic complexes on the screen. Which of the following should be the priority action of the nurse? 1. Call a code blue. 2. Call the physician. 3. Check the client status and lead placement. 4. Press the recorder button on the electrocardiogram console.

700. 3 Rationale: Sudden loss of electrocardiographic complexes indicates ventricular asystole or possibly electrode displacement. Accurate assessment of the client and equipment is necessary to determine the cause and identify the appropriate intervention. Options 1, 2, and 4 are unnecessary. Test-Taking Strategy: Use the steps of the nursing process. Always assess the client directly before taking any action. Option 3 is the only option that addresses assessment. Review care of the client on a cardiac monitor if you had difficulty with this question.

702. A client has frequent bursts of ventricular tachycardia on the cardiac monitor. What should the nurse be most concerned about with this dysrhythmia? 1. It can develop into ventricular fibrillation at any time. 2. It is almost impossible to convert to a normal rhythm. 3. It is uncomfortable for the client, giving a sense of impending doom. 4. It produces a high cardiac output that quickly leads to cerebral and myocardial ischemia.

702. 1 Rationale: Ventricular tachycardia is a life-threatening dysrhythmia that results from an irritable ectopic focus that takes over as the pacemaker for the heart. The low cardiac output that results can lead quickly to cerebral and myocardial ischemia. Clients frequently experience a feeling of impending doom. Ventricular tachycardia is treated with antidysrhythmic medications, cardioversion (client awake), or defibrillation (loss of consciousness). Ventricular tachycardia can deteriorate into ventricular fibrillation at any time. Test-Taking Strategy: Use the process of elimination and note the strategic words most concerned. Option 2 is incorrect and is eliminated first. From the remaining options, focusing on the strategic words will direct you to option 1 because this option identifies the life-threatening condition. Review the concerns associated with ventricular tachycardia if you had difficulty with this question.

704. A client has developed atrial fibrillation, with a ventricular rate of 150 beats/min. The nurse should assess the client for which associated signs or symptoms? 1. Flat neck veins 2. Nausea and vomiting 3. Hypotension and dizziness 4. Hypertension and headache

704. 3 Rationale: The client with uncontrolled atrial fibrillation with a ventricular rate more than 100 beats/min is at risk for low cardiac output because of loss of atrial kick. The nurse assesses the client for palpitations, chest pain or discomfort, hypotension, pulse deficit, fatigue, weakness, dizziness, syncope, shortness of breath, and distended neck veins. Test-Taking Strategy: Use the process of elimination. Flat neck veins are normal or indicate hypovolemia, so eliminate option 1. Nausea and vomiting (option 2) are associated with vagus nerve activity and do not correlate with a tachycardic state. From the remaining options, think of the consequences of falling cardiac output to direct you to option 3. Review the effects of atrial fibrillation if you had difficulty with this question.

712. A nurse is assessing the neurovascular status of a client who returned to the surgical nursing unit 4 hours ago after undergoing aortoiliac bypass graft. The affected leg is warm, and the nurse notes redness and edema. The pedal pulse is palpable and unchanged from admission. How would the nurse correctly interpret the client's neurovascular status? 1. The neurovascular status is normal because of increased blood flow through the leg. 2. The neurovascular status is moderately impaired, and the surgeon should be called. 3. The neurovascular status is slightly deteriorating and should be monitored for another hour. 4. The neurovascular status is adequate from an arterial approach, but venous complications are arising.

712. 1 Rationale: An expected outcome of aortoiliac bypass graft surgery is warmth, redness, and edema in the surgical extremity because of increased blood flow. Therefore options 2, 3, and 4 are incorrect interpretations. Test-Taking Strategy: Use the process of elimination. Option 2 can be eliminated because the pedal pulse is unchanged from admission. Venous complications from immobilization resulting from surgery would not be apparent within 4 hours, so eliminate option 4. From the remaining options, think about the effects of sudden reperfusion in an ischemic limb. There would be redness from new blood flow and edema from the sudden change in pressure in the blood vessels. Review the expected assessment findings following this surgical procedure if you had difficulty with this question.

62. The physician reports that a patient with a neck tumor has a tracheal shift. The nurse is aware that this means that the patient's trachea is: .

B) pushed to the unaffected side.

714. A nurse is caring for a client who had a resection of an abdominal aortic aneurysm yesterday. The client has an intravenous infusion with a rate of 150 mL/hr, unchanged for the last 10 hours. The client's urine output for the last 3 hours was 90, 50, and 28 mL (28 mL most recent). The client's blood urea nitrogen level is 35 mg/dL and serum creatinine level is 1.8 mg/dL, measured this morning. Which of the following actions should the nurse take next? 1. Call the physician. 2. Check the urine specific gravity. 3. Check to see if the client had a sample for serum albumin level drawn. 4. Put the intravenous line on a pump so that the infusion rate is sure to stay stable.

714. 1 Rationale: Following abdominal aortic aneurysm resection or repair, the nurse monitors the client for signs of renal failure. Renal failure can occur because often much blood is lost during the surgery and, depending on the aneurysm location, the renal arteries may be hypoperfused for a short period during surgery. The nurse monitors hourly intake and output and notes the results of daily blood urea nitrogen and creatinine levels. Urine output lower than 30 to 50 mL/hr is reported to the physician. Test-Taking Strategy: Focus on the information in the question and the abnormal assessment data. This question indicates elevations in blood urea nitrogen and creatinine levels and a significant drop in hourly urine output. These assessment findings should direct you to option 1. Review the complications associated with this surgical procedure if you had difficulty with this question.

716. A nurse in a medical unit is caring for a client with heart failure. The client suddenly develops extreme dyspnea, tachycardia, and lung crackles and the nurse suspects pulmonary edema. The nurse immediately asks another nurse to contact the physician and prepares to implement which priority interventions? Select all that apply. 1. Administering oxygen 2. Inserting a Foley catheter 3. Administering furosemide (Lasix) 4. Administering morphine sulfate intravenously 5. Transporting the client to the coronary care unit 6. Placing the client in a low Fowler's side-lying position

716. 1, 2, 3, 4 Rationale: Pulmonary edema is a life-threatening event that can result from severe heart failure. In pulmonary edema, the left ventricle fails to eject sufficient blood, and pressure increases in the lungs because of the accumulated blood. Oxygen is always prescribed, and the client is placed in a high Fowler's position to ease the work of breathing. Furosemide, a rapid-acting diuretic, will eliminate accumulated fluid. A Foley catheter is inserted to measure output accurately. Intravenously administered morphine sulfate reduces venous return (preload), decreases anxiety, and also reduces the work of breathing. Transporting the client to the coronary care unit is not a priority intervention. In fact, this may not be necessary at all if the client's response to treatment is successful. Test-Taking Strategy: Note the strategic words priority interventions and focus on the client's diagnosis. Recall the pathophysiology associated with pulmonary edema and use the ABCs—airway, breathing, and circulation—to help determine priority interventions. Review priority interventions for the client with pulmonary edema if you had difficulty with this question.

718. A nurse provides discharge instructions to a postoperative client who is taking warfarin sodium (Coumadin). Which statement, if made by the client, reflects the need for further teaching? 1. "I will take my pills every day at the same time." 2. "I will avoid alcohol consumption." 3. "I have already called my family to pick up a Medic-Alert bracelet." 4. "I will take Ecotrin (enteric-coated aspirin) for my headaches because it is coated."

718. 4 Rationale: Ecotrin is an aspirin-containing product and should be avoided. Alcohol consumption should be avoided by a client taking warfarin sodium. Taking prescribed medication at the same time each day increases client compliance. The Medic-Alert bracelet provides health care personnel emergency information. Test-Taking Strategy: Use the process of elimination and note the strategic words need for further teaching. These words indicate a negative event query and ask you to select an option that is an incorrect statement. Recalling that warfarin (Coumadin) is an anticoagulant and that Ecotrin is an aspirin-containing product will direct you to option 4. Review client teaching points related to warfarin if you had difficulty with this question.

721. A nurse is monitoring a client who is taking propranolol (Inderal). Which assessment data would indicate a potential serious complication associated with propranolol? 1. The development of complaints of insomnia 2. The development of audible expiratory wheezes 3. A baseline blood pressure of 150/80 mm Hg followed by a blood pressure of 138/72 mm Hg after two doses of the medication 4. A baseline resting heart rate of 88 beats/min followed by a resting heart rate of 72 beats/min after two doses of the medication

721. 2 Rationale: Audible expiratory wheezes may indicate a serious adverse reaction, bronchospasm. β-Blockers may induce this reaction, particularly in clients with chronic obstructive pulmonary disease or asthma. Normal decreases in blood pressure and heart rate are expected. Insomnia is a frequent mild side effect and should be monitored. Test-Taking Strategy: Use the process of elimination, eliminating options 3 and 4 because these are expected effects from the medication. Note the strategic words potential serious complication. These strategic words will direct you to option 2. Review the adverse effects of this medication if you had difficulty with this question.

723. A client is diagnosed with an acute myocardial infarction and is receiving tissue plasminogen activator, alteplase (Activase, tPA). Which of the following is a priority nursing intervention? 1. Monitor for renal failure. 2. Monitor psychosocial status. 3. Monitor for signs of bleeding. 4. Have heparin sodium available.

723. 3 Rationale: Tissue plasminogen activator is a thrombolytic. Hemorrhage is a complication of any type of thrombolytic medication. The client is monitored for bleeding. Monitoring for renal failure and monitoring the client's psychosocial status are important but are not the most critical interventions. Heparin is given after thrombolytic therapy, but the question is not asking about follow-up medications. Test-Taking Strategy: Use the process of elimination and note the strategic word priority. Remember, bleeding is a priority. Review care of the client on tissue plasminogen activator if you had difficulty with this question.

Patients may experience which problem 24-48 hrs post-op as a result of anesthetics? A. colitis B. Stomatitis C. Paralytic ileus D. Gastrocolic reflux

C. After surgery, pts are clients are at risk for paralytic ileus as a result of anesthesia.

3. Analyticity 4. Self-Confidence

A nurse has worked on an oncology unit for 3 years. One patient has become visibly weaker and states. "I feel funny." The nurse knows how patients often have behavior changes before developing sepsis when they have cancer. The nurse asks the patient questions to assess thinking skills and notices the patient shivering. The nurse goes to the phone, calls the physician, and begins the conversation by saying, "I believe that your patient is developing sepsis. I want to report symptoms I'm seeing." What examples of critical thinking concepts does the nurse show? (Select all that apply.) 1. Experience 2. Ethical 3. Analyticity 4. Self-Confidence 5. Risk taking

2. Advocate

A nurse is caring for a patient with end-stage lung disease. The patient wants to go home on oxygen and be comfortable. The family wants the patient to have a new surgical procedure. The nurse explains the risk and benefits of the surgery to the family and discusses the patient's wishes with the family. The nurse is acting as the patient's: 1. Educator 2. Advocate 3. Caregiver 4. Case manager

The primary goal of the circulating nurse during preparation of the operating room, transferring and positioning the patient, and assisting the anesthesia team is

A. avoiding any type of injury to the patient. The protection of the patient from injury in the operating room environment is maintained by the circulating nurse by ensuring functioning equipment, preventing falls and injury during transport and transfer, monitoring asepsis, and being with the patient during anesthesia induction.

2. Safety

A nurse is caring for an older-adult couple in a community-based assisted living facility. During the family assessment he notes that the couple has many expired medications and multiple medications for their respective chronic illnesses. They note that they go to two different health care providers. The nurse begins to work with the couple to determine what they know about their medications and helps them decide on one care provider rather than two. This is an example of which Quality and Safety in the Education of Nurses (QSEN) competency? 1. Patient-centered care 2. Safety 3. Teamwork 4. Informatics

3. The patient will perform breast self-examination correctly on herself before the end of the teaching session.

A nurse is going to teach a patient how to perform breast self-examination. Which behavioral objective does the nurse set to best measure the patient's ability to perform the examination? 1. The patient will verbalize the steps involved in breast self-examination within 1 week. 2. The nurse will explain the importance of performing breast self-examination once a month. 3. The patient will perform breast self-examination correctly on herself before the end of the teaching session. 4. The nurse will demonstrate breast self-examination on a breast model provided by American Cancer Society.

2. A learning objective

A nurse is planning a teaching session about healthy nutrition with a group of children who are in first grade. The nurse determines that after the teaching session the children will be able to name three examples of foods that are fruits. This is an example of: 1. A teaching plan 2. A learning objective 3. Reinforcement of content 4. Enhancing the children's self-efficacy

4. Health promotion and illness prevention

A nurse is teaching a group of young college-age women the importance of using sunscreen when going out in the sun. What type of content is the nurse providing? 1. Simulation 2. Restoring health 3. Coping with impaired function 4. Health promotion and illness prevention

1. A nurse explain to the NAP the approach to use in getting the patient up and why the patient has activity limitations. 3. The nurse sees the NAP preparing to help a patient out of bed, goes to assist, and thanks the NAP for her efforts to get the patient up early. 4. The nurse is in Patient B's room to check an intravenous (IV) line and collects the urine specimen while in the room.

A nurse is working with a nursing assistive personnel (NAP) on a bust oncology unit. The nurse has instructed the NAP on the tasks that need to be performed, including getting patient A out of bed, collecting a urine sample from patient B, and checking vital signs on patient C, who is scheduled to go home. Which of the following represent(s) successful delegation? (Select all that apply.) 1. A nurse explain to the NAP the approach to use in getting the patient up and why the patient has activity limitations. 2. A nurse is asked by a patient to help her to the bathroom; the nurse leaves the room and directs the NAP to assist the patient instead. 3. The nurse sees the NAP preparing to help a patient out of bed, goes to assist, and thanks the NAP for her efforts to get the patient up early. 4. The nurse is in Patient B's room to check an intravenous (IV) line and collects the urine specimen while in the room. 5. The nurse offers to support the NAP when needed but allows her to complete patient care tasks without constant oversight.

When administering low-molecular-weight heparin (LMWH) after an operation, a nurse should

A. explain that the drug will help prevent clot formation in the legs. Unfractionated heparin or LMWH is given as a prophylactic measure for venous thrombosis and pulmonary embolism. These anticoagulants work by inhibiting thrombin-mediated conversion of fibrinogen to fibrin. LMWH is injected subcutaneously with no relationship to meals. It has a more predictable dose response and less risk of bleeding complications. It does not require anticoagulant monitoring and dosage adjustments.

1. How to use an inhaler during an asthma attack

A nurse needs to teach a young woman newly diagnosed with asthma how to manage her disease. Which of the following topics does the nurse teach first? 1. How to use an inhaler during an asthma attack 2. The need to avoid people who smoke to prevent asthma attacks 3. Where to purchase a medical alert bracelet that says she has asthma 4. The importance of maintaining a healthy diet and exercising regularly

3. Chronic illness

A nurse observes that a patient whose home life is chaotic with intermittent homelessness, a child with spina bifida, and an abusive spouse appears to be experiencing an allostatic load. As a result, the nurse expects to detect which of the following while assessing the patient? 1. Posttraumatic stress disorder 2. Rising hormone levels 3. Chronic illness 4. Return of vital signs to normal

2. Think about past experience with patients who develop postoperative complications. 3. Decide which activities can be combined for patient B and C.

A nurse who is working on a surgical unit is caring for four different patients. Patient A will be discharged home and is in need of instruction about wound care. Patients B and C have returned from the operating room within an hour of each other, and both require vital signs and monitoring of their intravenous (IV) lines. Patient D is resting following a visit by physical therapy. Which of the following activities by the nurse represent(s) use of clinical decision making for groups of patients? (Select all that apply.) 1. Consider how to involve patient A in deciding whether to involve the family caregiver in wound care instruction. 2. Think about past experience with patients who develop postoperative complications. 3. Decide which activities can be combined for patient B and C. 4. Carefully gather any assessment information and identify patient problems.

3. Call the poison control center.

A parent calls the pediatrician's office frantic about the bottle of cleaner that her 2-year old son drank. Which of the following is the most important instruction the nurse gives to the parent? 1. Give the child milk. 2. Give the child syrup of ipecac. 3. Call the poison control center. 4. Take the child to the emergency department.

1. Denial.

A patient who is having difficulty managing his diabetes mellitus responds to the news that his hemoglobin A1C, a measure of blood sugar control over the past 90 days, has increased by saying, "the hemoglobin A1C is wrong. My blood sugar levels have been excellent for the last 6 months." The patient is using the defense mechanism: 1. Denial. 2. Conversion. 3. Dissociation. 4. Displacement.

2. Demonstration

A patient who is hospitalized has just been diagnosed with diabetes. He is going to need to learn how to give himself injections. Which teaching method does the nurse use? 1. Simulation 2. Demonstration 3. Group instruction 4. One-on-one discussion

3. Obtains an interpreter to facilitate communication of medication information

A patient with limited English proficiency is going to be discharged on new medication. How does the nurse complete the discharge teaching? 1. Uses a dictionary to give directions for medication administration 2. Explains the directions to the patient's 14-year old daughter 3. Obtains an interpreter to facilitate communication of medication information 4. Uses a picture board and visual aids to communicate medication administration information

3, 4, 1, 5, 2

A physician writes an order to apply a wrist restraint to a patient who has been pulling out a surgical wound drain. Place the following steps for applying the restraint in the correct order. 1. Explain what you plan to do. 2. Wrap a limb restraint around wrist or ankle with soft part toward skin and secure. 3. Determine that restraint alternatives fail to ensure patient's safety. 4. Identify the patient using proper identifier. 5. Pad the patient's wrist.

Before admitting a patient to the operating room, the nurse recognizes that which of the following must be attached to the chart of all patients?

C. A physical examination report. It is essential to have a physical examination report attached to the chart of a patient going for surgery. This document explains in detail the overall status of the patient for the surgeon and other members of the surgical team.

4. Nurses frequently experience stress with the rapid changes in health care technology and organizational restructuring.

A staff nurse is talking with the nursing supervisor about the stress that she feels on the job. The supervising nurse recognizes that: 1. Nurses who feel stress usually pass the stress along to their patients. 2. A nurse who feels stress is ineffective as a nurse and should not be working. 3. Nurses who talk about feeling stress are unprofessional and should calm down. 4. Nurses frequently experience stress with the rapid changes in health care technology and organizational restructuring.

79. When assessing tactile fremitus, the nurse recalls that it is normal to feel tactile fremitus most intensely over which location? A) Between the scapulae B) Third intercostal space, MCL C) Fifth intercostal space, MAL D) Over the lower lobes, posterior side

A) Between the scapulae Page: 424. Normally, fremitus is most prominent between the scapulae and around the sternum. These are sites where the major bronchi are closest to the chest wall. Fremitus normally decreases as one progress down the chest because more tissue impedes sound transmission.

52. A patient tells the nurse that he has noticed that one of his moles has started to burn and bleed. When assessing his skin, the nurse would pay special attention to the danger signs for pigmented lesions and would be concerned with which additional finding?

A) Color variation

87. During auscultation of breath sounds, the nurse should use the stethoscope correctly, in which of the following ways? A) Listen to at least one full respiration in each location. B) Listen as the patient inhales and then go to the next site during exhalation. C) Have the patient breathe in and out rapidly while the nurse listens to the breath sounds. D) If the patient is modest, listen to sounds over his or her clothing or hospital gown.

A) Listen to at least one full respiration in each location. Pages: 426-427. During auscultation of breath sounds with a stethoscope, it is important to listen to one full respiration in each location. During the examination, the nurse should monitor the breathing and offer times for the person to breathe normally to prevent possible dizziness.

A patient is scheduled for a hemorrhoidectomy at an ambulatory day-surgery center. An advantage of performing surgery at an ambulatory center is a decreased need for

A. laboratory tests and perioperative medications. Ambulatory surgery is usually less expensive and more convenient, generally involving fewer laboratory tests, fewer preoperative and postoperative medications, less psychological stress, and less susceptibility to hospital-acquired infections. However, the nurse is still responsible for assessing, supporting, and teaching the patient undergoing surgery, regardless of where the surgery is performed.

90. The nurse is assessing voice sounds during a respiratory assessment. Which of these findings indicates a normal assessment? Select all that apply. A) Voice sounds are faint, muffled, and almost inaudible when the patient whispers "one, two, three" in a very soft voice. B) As the patient says "ninety-nine" repeatedly, the examiner hears the words "ninety-nine" clearly. C) When the patient speaks in a normal voice, the examiner can hear a sound but cannot distinguish exactly what is being said. D) As the patient says a long "ee-ee-ee" sound, the examiner also hears a long "ee-ee-ee" sound. E) As the patient says a long "ee-ee-ee" sound, the examiner hears a long "aaaaaa" sound.

A) Voice sounds are faint, muffled, and almost inaudible when the patient whispers "one, two, three" in a very soft voice. C) When the patient speaks in a normal voice, the examiner can hear a sound but cannot distinguish exactly what is being said. D) As the patient says a long "ee-ee-ee" sound, the examiner also hears a long "ee-ee-ee" sound. Page: 446. As a patient says "ninety-nine" repeatedly, normally, the examiner hears sound but cannot distinguish what is being said. If a clear "ninety-nine" is auscultated, then it could indicate increased lung density, which enhances transmission of voice sounds. This is a measure of bronchophony. When a patient says a long "ee-ee-ee" sound, normally the examiner also hears a long "ee-ee-ee" sound through auscultation. This is a measure of egophony. If the examiner hears a long "aaaaaa" sound instead, this could indicate areas of consolidation or compression. With whispered pectoriloquy, as when a patient whispers a phrase such as "one-two-three," the normal response when auscultating voice sounds is to hear sounds that are faint, muffled, and almost inaudible. If the examiners hears the whispered voice clearly, as if the patient is speaking through the stethoscope, then consolidation of the lung fields may exist.

86. An adult patient with a history of allergies comes to the clinic complaining of wheezing and difficulty in breathing when working in his yard. The assessment findings include tachypnea, use of accessory neck muscles, prolonged expiration, intercostal retractions, decreased breath sounds, and expiratory wheezes. The nurse interprets that these assessment findings are consistent with: A) asthma. B) atelectasis. C) lobar pneumonia. D) heart failure.

A) asthma. Page: 451. Asthma is allergic hypersensitivity to certain inhaled particles that produces inflammation and a reaction of bronchospasm, which increases airway resistance, especially during expiration. Increased respiratory rate, use of accessory muscles, retraction of intercostal muscles, prolonged expiration, decreased breath sounds, and expiratory wheezing are all characteristic of asthma. See Table 18-8 for descriptions of the other conditions.

89. The nurse is auscultating the lungs of a patient who had been sleeping and notices short, popping, crackling sounds that stop after a few breaths. The nurse recognizes that these breath sounds are: A) atelectatic crackles, and that they are not pathologic. B) fine crackles, and that they may be a sign of pneumonia. C) vesicular breath sounds. D) fine wheezes.

A) atelectatic crackles, and that they are not pathologic. Pages: 429-430. One type of adventitious sound, atelectatic crackles, is not pathologic. They are short, popping, crackling sounds that sound like fine crackles but do not last beyond a few breaths. When sections of alveoli are not fully aerated (as in people who are asleep or in the elderly), they deflate slightly and accumulate secretions. Crackles are heard when these sections are expanded by a few deep breaths. Atelectatic crackles are heard only in the periphery, usually in dependent portions of the lungs, and disappear after the first few breaths or after a cough.

69. A 68-year-old woman is in the eye clinic for a checkup. She tells the nurse that she has been having trouble with reading the paper, sewing, and even seeing the faces of her grandchildren. On examination, the nurse notes that she has some loss of central vision but her peripheral vision is normal. These findings suggest that: A) she may have macular degeneration. B) her vision is normal for someone her age. C) she has the beginning stages of cataract formation. D) she has increased intraocular pressure or glaucoma.

A) she may have macular degeneration. Page: 285. Macular degeneration is the most common cause of blindness. It is characterized by loss of central vision. Cataracts would show lens opacity. Chronic open-angle glaucoma, the most common type of glaucoma, involves a gradual loss of peripheral vision.

The reason pts are sent to a PACU after surgery is: A. to be monitored while recovering from anesthesia. B. to remain near the surgeon immediately after surgery. C. to allow the medical-surgical unit time to prepare for transfer. D. to provide time for the pt to cope with the effects of surgery.

A. Pts are sent to a PACU to be monitored while they're recovering from anesthesia.

Select all that apply. A nurse is caring for patients on a medical-surgical unit. The nurse plans the patients' care and instructs the nursing assistant to assist in repositioning patients every 2 hours. Which patients are at the greatest risk for complications if not repositioned properly?

A. A 20-year-old unconscious patient B. A 90-year-old frail patient (&) D. A 40-year-old patient who has paraplegia Patients who are at the greatest risk for complications if not properly repositioned are those who are unconscious, frail, or paralyzed.

In the operating room, a patient tells a circulating nurse that he is going to have the cataract in his left eye removed. If the nurse notes that the consent form indicates that surgery is to be performed on the right eye, what should be the nurse's first action?

A. Ask the patient his name. Ensuring proper identification of a patient is a responsibility of all members of the surgical team. In a specialty surgical setting where many patients undergo the same type of surgery each day, such as cataract removal, it is possible that the patient and the record do not match. Nurses do not assume in the care of their patients. The priority is with the nurse identifying the patient and the patient's consent form before the physicians are notified.

Which of the following preoperative assessment findings should be reported to a surgeon for preoperative treatment?

A. Excessive thirst. The classic clinical manifestations of diabetes mellitus are increased frequency of urination (polyuria); increased thirst and fluid intake (polydipsia); and as the disease progresses, weight loss despite increased hunger and food intake (polyphagia). Weakness, fatigue, and recurrent blurred vision are associated with diabetes mellitus but are not considered priority manifestations because of the generalization of these complaints being applied to other disease processes. Weight loss is the cardinal sign related to the depletion of water, glycogen, and triglyceride stores.

Select all that apply. Which of the following best describes a consent form?

A. May be signed by an emancipated minor. (&) C. Signifies that the patient understands all aspects of the procedure. A consent form may be signed by an emancipated minor, and consent may be obtained by fax or phone with appropriate witnesses. Only in the cases of underage children or unconscious or mentally incompetent people must a family member be aware of the procedure. The document protects the surgeon and the health care facility in that it indicates that the patient knows and understands all aspects of the procedure.

Which of the following nursing interventions should receive highest priority when a patient is admitted to the postanesthesia care unit?

A. Positioning the patient. A patient is received in the postanesthesia care unit on a bed or stretcher. Proper positioning is necessary to ensure airway patency in a sedated, unconscious, or semiconscious patient. Observation of the operative site, receiving report from operating room personnel, and checking postoperative orders are interventions made after proper positioning of the patient.

In caring for a person receiving an opioid analgesic through an epidural catheter, the nursing responsibility of prime importance is

A. assessing for respiratory depression. Possible side effects of epidural opioids are pruritus, urinary retention, and delayed respiratory depression, occurring 4 to 12 hours after a dose. Establishing a baseline laboratory profile is outside the scope of practice for a nurse. Hourly inspection of the catheter insertion site is an unnecessary nursing intervention. In general, the site is assessed once a shift unless unexpected complications occur. Strict bed rest is not necessary for the patient with an epidural catheter; however, assistance with getting out of bed could be necessary related to effects of the opioid analgesic.

Addison's Disease: tests

ACTH stimulation test (cortisol testing) urine: decreased 17-ketosteroids, 17-hydroxycorticosteroids

MENINGITIS : disease discription

Acute infection of the meninges. Bacterial meningitis is an infection of the ventricular system and the CSF.

115. In obtaining a history on a 74-year-old patient the nurse notes that he drinks alcohol daily and that he has noticed a tremor in his hands that affects his ability to hold things. With this information, what should the nurse's response be? A) "Does your family know you are drinking every day?" B) "Does the tremor change when you drink the alcohol?" C) "We'll do some tests to see what is causing the tremor." D) "You really shouldn't drink so much alcohol; it may be causing your tremor."

B) "Does the tremor change when you drink the alcohol?" Page: 632. Intention tremor/ senile tremor is relieved by alcohol, although this is not a recommended treatment. The nurse should assess whether the person is abusing alcohol in an effort to relieve the tremor.

108. During an abdominal assessment, the nurse is unable to hear bowel sounds in a patient's abdomen. Before reporting this finding as "silent bowel sounds" the nurse should listen for at least: A) 1 minute. B) 5 minutes. C) 10 minutes. D) 2 minutes in each quadrant.

B) 5 minutes. Pages: 539-540. Absent bowel sounds are rare. The nurse must listen for 5 minutes before deciding bowel sounds are completely absent.

93. The nurse is preparing to auscultate for heart sounds. Which technique is correct? A) Listen to the sounds at the aortic, tricuspid, pulmonic, and mitral areas. B) Listen by inching the stethoscope in a rough Z pattern, from the base of the heart across and down, then over to the apex. C) Listen to the sounds only at the site where the apical pulse is felt to be the strongest. D) Listen for all possible sounds at a time at each specified area.

B) Listen by inching the stethoscope in a rough Z pattern, from the base of the heart across and down, then over to the apex. Pages: 475-476. Do not limit auscultation of breath sounds to only four locations. Sounds produced by the valves may be heard all over the precordium. Inch the stethoscope in a rough Z pattern from the base of the heart across and down, then over to the apex. Or, start at the apex and work your way up. See Figure 19-22. Listen selectively to one sound at a time.

100. The nurse is reviewing an assessment of a patient's peripheral pulses and notices that the documentation states that the radial pulses are "2+." The nurse recognizes that this reading indicates what type of pulse? A) Bounding B) Normal C) Weak D) Absent

B) Normal Pages: 506-507. When documenting the force, or amplitude, of pulses, 3+ indicates an increased, full, or bounding pulse, 2+ indicates a normal pulse, 1+ indicates a weak pulse, and 0 indicates an absent pulse.

98. The nurse is attempting to assess the femoral pulse in an obese patient. Which of these actions would be most appropriate? A) Have the patient assume a prone position. B) Ask the patient to bend his or her knees to the side in a froglike position. C) Press firmly against the bone with the patient in a semi-Fowler position. D) Listen with a stethoscope for pulsations because it is very difficult to palpate the pulse in an obese person.

B) Regular "lub, dub" pattern Pages: 510-511. To help expose the femoral area, particularly in obese people, the nurse should ask the person to bend his or her knees to the side in a froglike position.

111. The nurse suspects that a patient has appendicitis. Which of these procedures are appropriate for use when assessing for appendicitis or a perforated appendix? Select all that apply. A) Test for Murphy's sign. B) Test for Blumberg's sign. C) Test for shifting dullness. D) Perform iliopsoas muscle test. E) Test for fluid wave.

B) Test for Blumberg's sign. D) Perform iliopsoas muscle test. Pages: 543-544 | Page: 551. Testing for Blumberg's sign (rebound tenderness) and performing the iliopsoas muscle test should be used to assess for appendicitis. Murphy's sign is used to assess for an inflamed gallbladder or cholecystitis. Testing for a fluid wave and shifting dullness is done to assess for ascites.

109. A patient is suspected of having inflammation of the gallbladder, or cholecystitis. The nurse should conduct which of these techniques to assess for this condition? A) Obturator test B) Test for Murphy's sign C) Assess for rebound tenderness D) Iliopsoas muscle test

B) Test for Murphy's sign Page: 551. Normally, palpating the liver causes no pain. In a person with inflammation of the gallbladder, or cholecystitis, pain occurs as the descending liver pushes the inflamed gallbladder onto the examining hand during inspiration (Murphy's test). The person feels sharp pain and abruptly stops inspiration midway.

71. During an examination, a patient states that she was diagnosed with open-angle glaucoma 2 years ago. The nurse assesses for characteristics of open-angle glaucoma. Which of these are characteristics of open-angle glaucoma? Select all that apply. A) The patient may experience sensitivity to light, nausea, and halos around lights. B) The patient experiences tunnel vision in late stages. C) Immediate treatment is needed. D) Vision loss begins with peripheral vision. E) It causes sudden attacks of increased pressure that cause blurred vision. F) There are virtually no symptoms.

B) The patient experiences tunnel vision in late stages. D) Vision loss begins with peripheral vision. F) There are virtually no symptoms. Pages: 308-309. Open-angle glaucoma is the most common type of glaucoma; there are virtually no symptoms. Vision loss begins with the peripheral vision, which often goes unnoticed because individuals learn to compensate intuitively by turning their heads. The other characteristics are those of closed-angle glaucoma.

75. The nurse is testing the hearing of a 78-year-old man and keeps in mind the changes in hearing that occur with aging include which of the following? Select all that apply. A) Hearing loss related to aging begins in the mid 40s. B) The progression is slow. C) The aging person has low-frequency tone loss. D) The aging person may find it harder to hear consonants than vowels. E) Sounds may be garbled and difficult to localize. F) Hearing loss reflects nerve degeneration of the middle ear.

B) The progression is slow. D) The aging person may find it harder to hear consonants than vowels. E) Sounds may be garbled and difficult to localize. Page: 326. Presbycusis is a type of hearing loss that occurs with aging and is found in 60% of those older than 65 years. It is a gradual sensorineural loss caused by nerve degeneration in the inner ear or auditory nerve, and it slowly progresses after age 50. The person first notices a high-frequency tone loss; it is harder to hear consonants (high-pitched components of speech) than vowels. This makes words sound garbled. The ability to localize sound is impaired also.

66. The nurse is performing an eye assessment on an 80-year-old patient. Which of these findings is considered abnormal? A) A decrease in tear production B) Unequal pupillary constriction in response to light C) The presence of arcus senilis seen around the cornea D) Loss of the outer hair on the eyebrows due to a decrease in hair follicles

B) Unequal pupillary constriction in response to light Pages: 305-308. Pupils are small in old age, and the pupillary light reflex may be slowed, but pupillary constriction should be symmetric. The assessment findings in the other responses are considered normal in older persons.

91. During an assessment of a 68-year-old man with a recent onset of right-sided weakness, the nurse hears a blowing, swishing sound with the bell of the stethoscope over the left carotid artery. This finding would indicate: A) a valvular disorder. B) blood flow turbulence. C) fluid volume overload. D) ventricular hypertrophy.

B) blood flow turbulence. Page: 471. A bruit is a blowing, swishing sound indicating blood flow turbulence; normally none is present.

51. The nurse keeps in mind that a thorough skin assessment is very important because the skin holds information about a person's:

B) circulatory status.

11. In the majority culture of America, coughing, sweating, and diarrhea are symptoms of an illness. For some individuals of Mexican-American origin, however, these symptoms are a normal part of living. The nurse recognizes that this is true, probably because Mexican-Americans:

B) consider these symptoms a part of normal living, not symptoms of ill health.

What would be the most effective way for a nurse to validate "informed consent"?

C. Ask the patient what he or she understands regarding the procedure. Informed consent in the health care setting is a process whereby a patient is informed of the risks, benefits, and alternatives of a certain procedure, and then gives consent for it to be done. The piece of paper is simply evidence that the informed consent process has been done.

65. A mother asks when her newborn infant's eyesight will be developed. The nurse should reply:

C) "By about 3 months, infants develop more coordinated eye movements and can fixate on an object."

48. When the mid-upper arm circumference and triceps skinfold of an 82-year-old man are evaluated, which is important for the nurse to remember?

C) These measurements may not be accurate because of changes in skin and fat distribution.

104. While examining a patient, the nurse observes abdominal pulsations between the xiphoid and umbilicus. The nurse would suspect that these are: A) pulsations of the renal arteries. B) pulsations of the inferior vena cava. C) normal abdominal aortic pulsations. D) increased peristalsis from a bowel obstruction.

C) normal abdominal aortic pulsations. Pages: 538-539. Normally, one may see the pulsations from the aorta beneath the skin in the epigastric area, particularly in thin persons with good muscle wall relaxation.

114. During an assessment of an 80-year-old patient, the nurse notices the following: inability to identify vibrations at the ankle and to identify position of big toe, slower and more deliberate gait, and slightly impaired tactile sensation. All other neurologic findings are normal. The nurse should interpret that these findings indicate: A) cranial nerve dysfunction. B) lesion in the cerebral cortex. C) normal changes due to aging. D) demyelinization of nerves due to a lesion.

C) normal changes due to aging. Page: 629. Some aging adults show a slower response to requests, especially for those calling for coordination of movements. The findings listed are normal in the absence of other significant abnormal findings. The other responses are incorrect.

81. When auscultating the lungs of an adult patient, the nurse notes that over the posterior lower lobes low-pitched, soft breath sounds are heard, with inspiration being longer than expiration. The nurse interprets that these are: A) sounds normally auscultated over the trachea. B) bronchial breath sounds and are normal in that location. C) vesicular breath sounds and are normal in that location. D) bronchovesicular breath sounds and are normal in that location.

C) vesicular breath sounds and are normal in that location. Pages: 428-429. Vesicular breath sounds are low-pitched, soft sounds with inspiration being longer than expiration. These breath sounds are expected over peripheral lung fields where air flows through smaller bronchioles and alveoli.

The nurse is evaluating the status of the client who had a craniotomy 3 days ago. The nurse would suspect that the client is developing meningitis as a complication of surgery if the client exhibits:

C- A positive Brudzinski's sign

The nurse is evaluation the respiratory outcomes for the client with Guillain-Barre syndrome. The nurse determines that which of the following is the least optimal outcome for the client?

C- Adventitious breath sounds

The nurse is trying to communicate with a client with a stroke and aphasia. Which of the following actions by the nurse would be least helpful to the client?

C- Completing the sentences that the client cannot finish. Rationale: Note that the question asks which is least helpful. These words indicate a negative event query and ask you to select an option that is and incorrect action.

The nurse has given suggestions to the client with trigeminal neuralgia about strategies to minimize episodes of pain. The nurse determines that the client needs reinforcement of information if the client makes which of the following statements?

C- I'll try to eat my food either very warm or very cold

The client has experienced an episode of Myasthenic crisis. The nurse would assess whether the client has precipitating factors such as:

C- Omitting doses of medication

The client with Guillian-Barre syndrome has ascending paralysis and is intubated and receiving mechanical ventilation. Which of the following strategies would the nurse incorporate in the plan of care to help the client cope with this illness?

C- Providing information, giving positive feedback, and encouraging relaxation.

Which statement should be stressed while giving instructions after adrenalectomy? A. Stop taking medication when pts physical appearance improves. B. Pt should take steroids on an empty stomach. C. Pt should take the prescribed medication as directed.

C. The pt should take prescribed medication as directed. Sudden withdrawl of steroids can precipitate adrenal crisis.

Which statement about diabetes mellitus is false? A. Type 2 diabetes commonly occurs in adults <40 yr. old. B. Type 1 diabetes usually occurs before age 30. C. Type 1 diabetes is treatable with exercise, meal planning, and antidiabetic drugs. D. An increasing number of adolescents are being diagnosed with type 2 diabetes.

C. Type 1 diabetes is treated with insulin and dietary management.

Five minutes after receiving a preoperative sedative medication by IV injection, a patient asks the nurse to get up to go to the bathroom to urinate. Which of the following is the most appropriate action for the nurse to take? A.) Assist patient to bathroom and stay next to door to assist patient back to bed when done. B. Allow patient to go to the bathroom since the onset of the medication will be more than 5 minutes. C. Offer the patient to use the urinal/bedpan after explaining the need to maintain safety. D. Ask patient to hold the urine for a short period of time since a urinary catheter will be placed in the operating room.

C. Offer the patient to use the urinal/bedpan after explaining the need to maintain safety. The prime issue after administration of either sedative or opioid analgesic medications is safety. Because the medications affect the central nervous system, the patient is at risk for falls and should not be allowed out of bed, even with assistance.

21. In recording the childhood illnesses of a patient who denies having had any, which note by the nurse would be most accurate?

D. Patient denies measles, mumps, rubella, chickenpox, pertussis, and strep throat.

23. A 5-year-old boy is being admitted to the hospital to have his tonsils removed. Which information should the nurse collect before this procedure?

D. Reactions to previous hospitalizations

2. Since your husband has died, what have you been doing in the evening when you feel lonely?

During the assessment interview of an older woman experiencing a developmental crisis, the nurse asks which of the following questions? 1. How is this flood affecting your life? 2. Since your husband has died, what have you been doing in the evening when you feel lonely? 3. How is having diabetes affecting your life? 4. I know this must be hard for you. Let me tell you what might help.

Diagnosis of HIV/AIDS

ELISA; western blot; viral T-cell count

112. When assessing muscle strength, the nurse observes that a patient has complete range of motion against gravity with full resistance. What Grade should the nurse record using a 0 to 5 point scale? A) 2 B) 3 C) 4 D) 5

D) 5 Pages: 578-579. Complete range of motion against gravity is normal muscle strength and is recorded as Grade 5 muscle strength.

Upon admission for an appendectomy, the patient provides the nurse with a document that specifies instructions his healthcare team should follow in the event he is unable to communicate these wishes postoperatively. This document is best known as:

D) An advance directive, a legal document, allows the patient to specify instructions for his or her healthcare treatment should he or she be unable to communicate these wishes postoperatively. The advance directive allows the patient to discuss his or her wishes with the family members in advance of the surgery. Two common forms of advance directives include living wills and durable powers of attorney for healthcare.

The client is admitted to the hospital with a diagnosis of Guillian-Barre syndrome. The nurse inquires during the nursing admission interview if the client has a history of:

D- Respiratory or gastrointestinal infection during the previous month

The nurse is planning to test the function of the trigeminal nerve (cranial nerve V). The nurse would gather which of the following items to perform the test?

D- Safety pin, hot and cold water in test tubes, cotton wisp

The nurse is teaching the client with myasthenia gravis, about the prevention of myasthenic and cholinergic crises. The nurse tells the client that this is most effectively done by:

D- Taking medications on time to maintain therapeutic blood levels

A nurse is assessing a pt with a closed chest tube drainage system connected to suction. Which finding would require additional evaluation in the post-operative period?

D. Constant bubbling in the water seal chamber is indicative of an air leak. The nurse should assess the entire system to the pt to find the sourse of he leak. The leak may be with in the pts chest or at the insertion site. If it is, notify physician. This could cause the lung to collapse due to a buildup of air pressure within the plural cavity.

An adrenal crisis is characterized by all of the signs and symptoms except: A. weakness and fatigue B. nausea & vomiting C. hypotension D. sodium & fluid retention

D. Sodium and fluid retention are characteristics of Cushing's Syndrome. Adrenal crisis causes decreased sodium levels and hypotension.

Hematocrit (HCT) and Hemaglobin(Hgb) can be ordered to detect what in a GI pt?

GI bleed results would indicate anemia

Lomotil inhibits what?

GI motility - it is a non-analgesic opiod that stops diarrhea in its tracks and is contraindicated w/ infectious diarrhea

1. "I understand your reluctance, but the exercises are necessary for you to regain function in your shoulder. Let's go a bit more slowly and try to relax."

During a home health visit the nurse prepares to instruct a patient in how to perform range-of-motion (ROM) exercises for an injured shoulder. The nurse verifies that the patient took an analgesic 30 minutes before arrival at the patient's home. After discussing the purpose for the exercises and demonstrating each one, the nurse has the patient perform them. After two attempts with only the second of three exercises, the patient stops and says, "This hurts too much. I don't see why I have to do this so many times." The nurse applies the critical thinking attitude of integrity in which of the following actions? 1. "I understand your reluctance, but the exercises are necessary for you to regain function in your shoulder. Let's go a bit more slowly and try to relax." 2. "I see that you're uncomfortable. I'll call your doctor to decide the next step." 3. "Show me exactly where your pain is and rate it for me on a scale of 0 to 10." 4. "Is anything else bothering you? Other than the pain, is there any other reason you might not want to do the exercises?"

NSG Interventions for ICP:

elevate HOB 30-45 degrees to promote venous return, place neck in neutral position, avoid flexion of hip as well as head, restrict fluids, avoid val salves maneuver, insert foley, admin O2 via mask or cannula, and increase body temp.

What is decerebrate posturing?

Extensor response: Hands pushed to sides and body hyper-extended. Arms are stiffly extended, adducted & hyperpronated. Hyperextension of the legs with plantar flexion of the feet. (May indicate more serious damage.) "extensor = All E's"

Gestational trophoblastic disease (hydatidiform mole): assessment

elevated hCg level, uterine size greater than expected for dates, NO FHR, minimial dark red/brown vaginal bleeding with passage of grapelike clusters

Type 2 diabetes: Risk factors

Genetics, Obesity, AA, HTN, older than 45yo, HDL <35mg/dl Triglyceride > 250mg/dl

Retinopathy of prematurity (ROP): Assessment

separates avasuclar retina anteriorly with vascularized retina posteriorly

Abruptio placenta, which is...

separation of the placenta before the third stage of labor, produces painful bleeding.

The adolescent is usually on an...

emotional roller coaster; desire for increased independence and changing roles within the family lead to parent-child conflict; peer relationship are all-important

Parkinsons Disease: Plan

encourage finger exercise (typing piano), ROM

ileostomy:

end of ileum (small intestine) rest of large intestine removed. ostomy return is liquid, soft and semisolid. open end pouch worn at all times. low residue diet no meats, corn or nuts. remove q 4-6 h

Tonometry

Measures intraocular pressure. cornea is anesthetized. Tonometer registers degree of indentation on cornea when pressure is applied. Pressure increased in glaucoma

Labyrinth dysfunction

Meniere's disease is an inner ear disorder that affects balance and hearing.

Intestinal ostomies for fecal diversion: plan

enemas evening before and morning of surgery; antibiotic day before surgery (erthromyocin);

If alkaline-ash diet is ordered to increase pH of urine what will it include:

Milk, veggies, beef, halibut, trout, salmon No prunes or plums

Radiation: Sun, lightening, eclipses

Prevention-Use of eye shields

Bells Palsy (facial paralysis): Nursing Consideration:

Protect head from cold or drafts; teach isometric exercise for facial muscle (blow and suck straw) Recovery 3-5 wks

A client admitted with peritonitis is under a NPO order. The client is complaining of thrist. Which action is the most appropriate for the nurse to take? 1) Increase the I.V. infusion rate 2) Use diversion activities 3) Provide frequent mouth care 4) Give ice chips every 15 minutes

RATIONALE: 3) frequent mouth care helps relieve dry mouth. Increasing the I.V. infusion rate does not alleviate the feeling of thirst. Diversion activities aren't specific. Ice chips are a form of liquid and shouldn't be given as long as the client is under an NPO order.

A client is admitted with acute pancreatitis. Which laboratory result should the nurse expect? 1) Creatinine of 4.3 mg/dl 2) ALT of 124 international units/L 3) Amylase of 306 units/L 4) Troponin level of 3.5 mcg/L

RATIONALE: 3)Pancreatitis involves activation of pancreatic enzymes, such as amylase and lipase. Therefore, serum amylase and lipase levels are elevated in a client with acute pancreatitis. Serum creatinine levels are elevated with kidney disfunction. Injury or disease of the liver elevated ALT levels. Troponin levels are elevated with heart damage such as myocardial infarction.

A client has undergone a colostomy for a ruptured diverticulum. The nurse is assessing the client's colostomy stoma 2 days after surgery. Which assessment finding should the nurse report to the physician? 1) Blanched stoma 2) Edematous stoma 3) Reddish pink stoma 4) Brownish black stoma

RATIONALE: 4) A brownish black stoma color indicates a lack of blood flow to the stoma, and necrosis is likely. A blanched or pale stoma indicates possible decreased blood flow and should be assessed regularly. 2 days postoperatively, the stoma should be edematous and reddish pink.

A client who recently had abdominal surgery tells the nurse he felt a popping sensation in his incision during a coughing spell, following by severe pain. The nurse anticipates an evisceration. Which supplies should she bring to the client's room? 1) A suture kit 2) Sterile water and a suture kit 3) Sterile water and sterile dressings 4) Sterile saline solution and sterile dressings

RATIONALE: 4) Saline solution is isotonic, or close to body fluids in content, and is used along with sterile dressings to cover an eviscerated wound (a wound that opened, allowing the intestines to protrude outside the body) and keep it moist. Sterile water and a suture kit aren't used. The physician will contacted, and the client will most likely return to the operating room for closure.

After an abdominal resection for colon cancer, the client returns to his room with a Jackson-Pratt drain in place. The client's spouse asks the nurse about the purpose of the drain. The best response would be for the nurse to say:

RATIONALE: 4) The accumulation of fluid in a surgical wound interferes with the healing process. A Jackson-Pratt drain promotes wound healing by allowing fluid to escape from the wound. The drain may be placed in the client's incision, or it may be placed in the wound and brought out to the skin surface through a stab wound near the incision. The incision doesn't need to be irrigated. Fluid from the drain is absorbed into the dressings and can't be measured accurately. A Jackson-Pratt drain doesn't prevent infection.

Following abdominal surgery, a client has developed a gaping incision due to delayed wound healing. The nurse is preparing to irrigate the incision using a piston syringe and sterile normal saline solution. Which method should the nurse use as a part of the irrigation process.

RATIONALE: 4) To wash away tissue debris and drainage effectively, irrigate the wound until the solution becomes clear or until all of the solution is used. After the irrigation, dry the area around the wound; moistening it promotes microorganism growth and skin irritation. When the area is dry, apply sterile dressing rather than a wet-to-dry dressing. Always instill the irrigating solution gently; rapid or forceful instillation can damage tissues.

A client is admitted with inflammatory bowel syndrome (Crohn's disease). Which treatment measures should the nurse expect to be part of the care plan? SELECT ALL THAT APPLY!

RATIONALE: 4) and 5) Corticosteroids, such as prednisone, reduce the S/S of diarrhea, pain, and bleeding by decreasing inflammation. Antidiarrheals, such as diphenoxylate (Lomotil), combat diarrhea by decreasing peristalsis. Lactulose is used to treat chronic constipation and would aggravate the symptoms. A high fiber diet, milk, and milk products are contraindicated in clients with Crohn's disease because they may promote diarrhea.

A client is admitted with inflammatory bowel syndrome (Crohn's disease). Which treatment measures should the nurse expect to be part of the care plan? SELECT ALL THAT APPLY! 1) Laculose therapy 2) High fiber diet 3) High protein milkshakes 4) Corticosteroid therapy 5) Antidiarrheal medications

RATIONALE: 4) and 5) Corticosteroids, such as prednisone, reduce the S/S of diarrhea, pain, and bleeding by decreasing inflammation. Antidiarrheals, such as diphenoxylate (Lomotil), combat diarrhea by decreasing peristalsis. Lactulose is used to treat chronic constipation and would aggravate the symptoms. A high fiber diet, milk, and milk products are contraindicated in clients with Crohn's disease because they may promote diarrhea.

Tx for 3rd degree AV block

epinephrine, pacers NO lidocaine or atropine.

As a part of discharge teaching, the nurse instructs a client receiving citalopram (Celexa) 20 mg OD. The nurse determines that further teaching is necessary if the client states which of the following?" 1. "This medication helps me with my depression." 2. "I will notify my physician if I show signs of hyperactivity and mania." 3. "I will see improvement in my symptoms in 1 to 4 weeks." 4. "If I experience a fever I will take Tylenol."

Strategy: "Further teaching is necessary" indicates incorrect information. 1) Celexa is a selective serotonin reuptake inhibitor (SSRI) used to treat depression 2) side effects: mania, hypomania, insomnia, impotence, headache, and dry mouth 3) true statement 4) correct— should notify physician immediately to assess for serotonin syndrome, which is a rare, life threatening event caused by SSRIs; symptoms include abdominal pain, fever, sweating, tachycardia, hypertension, delirium, myoclonus, irritability, and mood changes; may result in death

The nurse in the pediatric clinic performs a well-child assessment on a 20-month-old. The child's mother tells the nurse that she is earning extra money by growing houseplants in her home. Which of the following responses by the nurse is MOST appropriate? 1. "How did you get into that business?" 2. "What a great opportunity." 3. "You should not have plants in your home." 4. "Where do you keep the plants?"

Strategy: "MOST appropriate" indicates discrimination is required to answer the question. 1) encourages the mother to talk about her interests but does not address safety issue of toddler 2) closed response; does not give client opportunity to respond 3) not all plants are toxic; nurse is expressing an opinion without completing the assessment 4) CORRECT— toddlers explore by putting things in their mouth; all potentially toxic agents should be placed out of reach of the toddler; nurse should assess the type of plants in the home and the location of the plants

The nurse prepares an elderly client newly diagnosed with type 1 diabetes for discharge. The client is alert and oriented and lives alone in her home. It is MOST important for the nurse to assess for which of the following? 1. Client's vision and manual dexterity. 2. Client's understanding of diabetes. 3. Client's need for visits from the home care nurse. 4. Client's ability to perform blood glucose monitoring.

Strategy: "MOST important" indicates discrimination is required to answer the question. 1) CORRECT— client must have the visual acuity and manual dexterity to draw up and administer insulin 2) it is important that the client understands diabetes, but priority is assessing client's ability to manage insulin administration 3) may be necessary 4) important, but first assess the client's vision and manual dexterity

A nursing assistant informs the nurse that an elderly client admitted following a hemorrhagic cerebrovascular accident ate half of the food on his tray. The food left on the tray looked as if someone had drawn a straight line down the center of the plate and eaten the food only to one side of the line. Which of the following instructions by the nurse is MOST important? 1. "Rotate the plate so that the food is on the other side." 2. "Offer him a snack later in the day." 3. "Ask the client's family to assist him with the next meal." 4. "Which foods did he omit?"

Strategy: "MOST important" indicates discrimination is required to answer the question. 1) CORRECT— indicates homonymous hemianopsia (loss of half of the visual field); client neglects one side of body; instruct client to turn head in direction of visual loss 2) food pattern on plate indicates loss of visual field 3) passing the buck 4) situation does not require further assessment

The nurse cares for a client receiving a heparin drip via an infusion pump. The physician orders warfarin (Coumadin) 5 mg PO. Which of the following actions should the nurse take NEXT? 1. Administer medication as ordered. 2. Notify the physician. 3. Check the most recent serum partial prothrombin levels. 4. Assess client for signs/symptoms of bleeding.

Strategy: "NEXT" indicates priority 1) CORRECT— warfarin interferes with the hepatic synthesis of vitamin K-dependent clotting factors; oral anticoagulant therapy should be instituted 4 to 5 days before discontinuing the heparin therapy 2) no reason to notify the physician 3) partial thromboplastin time used to monitor effectiveness of heparin; therapeutic level is 1.5 to 2.5 times the control 4) warfarin takes 3 to 5 days to reach peak levels

The nurse instructs a mother of a child diagnosed with a myelomeningocele who developed an allergy to latex. The nurse determines that teaching is effective if the mother selects which of the following menus for her child? 1. Guacamole with pita bread, lettuce, tomato juice. 2. Poached halibut, brown rice, carrots, peach cobbler. 3. Scrambled eggs, whole wheat toast, grapes, skim milk. 4. Baked chicken leg, mashed potatoes, spinach, milkshake.

Strategy: "Teaching is effective" indicates correct information. 1) if a person has a latex allergy, there is cross-reaction to tomatoes and avocados 2) peach is a cross-reactive food with latex 3) grapes are cross-reactive with latex 4) CORRECT— this meal does not have any cross-reactive foods with latex; foods to avoid include apricots, cherries, grapes, kiwis, passion fruit, bananas, avocados, chestnuts, tomatoes, and peaches

The nurse performs discharge teaching for a client diagnosed with gastroesophageal reflux disease (GERD). The nurse determines that teaching is successful if the client selects which of the following menus? 1. Pork loin, lettuce and tomato salad with vinegar and oil dressing, jello, and cola. 2. Cheddar cheese omelet, spinach salad, chocolate brownie, and milk. 3. Broiled chicken, cream of broccoli soup, rice pudding, and apple juice. 4. Baked salmon with lemon butter, baked potato, mint chocolate chip ice cream, and lemonade.

Strategy: "Teaching is successful" indicates correct information. 1) oil dressing high in fat, tomato exacerbates GERD, as do carbonated beverages 2) fatty foods and chocolate exacerbate GERD 3) CORRECT— menu low in fat and contains non-acidic foods 4) lemonade and mint exacerbate GERD

A nurse observes a student nurse administer carvedilol (Coreg) to an elderly patient. The patient refuses medication, saying, "Go away. It makes me dizzy." The nurse should intervene if the student nurse states which of the following? 1. "If you don't take this medication, you will be restrained." 2. "This medication will help control your blood pressure." 3. "Side effects of this medication make some patients feel uncomfortable." 4. "When do you notice the dizziness?"

Strategy: "nurse should intervene" indicates something is wrong. 1) CORRECT— inappropriate action; client has the right to refuse medication; restraining client is an example of battery 2) Coreg is a nonselective beta-blocker used to treat hypertension and heart failure 3) side effects include dizziness, fatigue, weakness, orthostatic hypotension; instruct client to change positions slowly 4) allows nurse to teach about medication

Strangulated hernia:

severe abdominal pain, n/v distention intestinal obstruction. blood supply to the intestine is obstructed.

The nurse cares for children in the outpatient pediatric clinic. It is MOST important for the nurse to perform tuberculosis screening on which of the following children? 1. A child just returned from a 2-week trip to Europe. 2. A child recently moved to an apartment because the family lost their home. 3. A child with a new nanny who just emigrated from Latin America. 4. A child who weighed 4 lb, 10 oz at birth.

Strategy: All answers are assessments. Determine how they relate to risk factors for tuberculosis. 1) tuberculosis is endemic to Asia, Middle East, Africa, Latin America, and Caribbean; consider screening if child has traveled to an endemic region 2) the homeless and impoverished are at risk for developing tuberculosis 3) CORRECT— children traveling to endemic areas or who have prolonged, close contact with indigenous persons should undergo immediate skin testing 4) no reasons to undergo immediate screening

The nurse cares for a 4-year-old on the pediatric unit. The child is unable to go to sleep while in the hospital. It is MOST important for the nurse to take which of the following actions? 1. Turn out the light and close the door. 2. Encourage the child to exercise during the evening. 3. Identify the child's home bedtime ritual. 4. Ask the child's siblings to visit during the evening.

Strategy: Assess before implementing 1) will increase the child's fears; preschoolers fear injury, mutilation, and punishment 2) will not promote sleep 3) CORRECT— preschoolers require bedtime rituals that should be followed in hospital; nurse should assess before implementing 4) will be comforting to child, but to promote sleep it is more important to determine bedtime routine

The nurse plans care for a patient in hemorrhagic shock from injuries sustained in a fall. It is MOST important for the nurse to take which of the following actions? 1. Obtain vital signs. 2. Identify the source of the bleeding. 3. Elevate the head of the bed 30°. 4. Administer 0.9% NaCl IV.

Strategy: Assess before implementing. 1) assessment; more important to determine the source of bleeding 2) CORRECT— assessment first step; initial priority to identify and then apply direct pressure and elevate affected area if possible 3) intervention; elevate the extremities 4) intervention; 1-2 liter bolus of isotonic fluids (lactated Ringer or 0.9% NaCl) will be given

The nurse in a small town is called to a neighbor's house in the middle of a blizzard. The neighbor woman states she is in the 39th week of gestation with her second baby and has been having contractions for several hours. The woman has been unable to obtain assistance because the roads are impassable. The nurse determines that the woman is in the second stage of labor. It is MOST important for the nurse to take which of the following actions? 1. Time the frequency of the contractions. 2. Assess the type of vaginal discharge. 3. Monitor the strength of the contractions. 4. Observe the perineum.

Strategy: Assess before implementing. 1) priority is assessing if baby is crowning 2) priority is assessing if baby is crowning 3) labor is not the priority; nurse should determine if the birth is imminent 4) CORRECT— baby will descend into birth canal and may crown, major responsibility in second state of labor; support infant's head; apply slight pressure to control delivery --

The nurse cares for a client diagnosed with a recurrence of colon cancer. The client tells the nurse that she is dreading taking chemotherapy again. Which of the following responses by the nurse is MOST appropriate? 1. "There are web sites that provide information about chemotherapy." 2. "Have you discussed this with your physician?" 3. "I can give you a handout about how to treat the side effects of chemotherapy." 4. "What are your concerns about taking chemotherapy?"

Strategy: Assessment before implementation 1) assumes that client needs more information about chemotherapy; nurse should respond to client's concerns 2) don't pass the buck; responding to client's concerns is a nursing responsibility 3) assess before implementing 4) CORRECT— think about the nursing process when selecting answers; allows nurse to gather data about what is concerning the client

The nurse evaluates assignments on the unit. The nurse determines that assignments are appropriate if the LPN/LVN is assigned to which of the following patients? 1. A patient with type 1 diabetes scheduled for discharge. 2. A patient newly admitted to the unit with chest pain. 3. A patient receiving chemotherapy. 4. A patient diagnosed with myasthenia gravis.

Strategy: Assign stable patients with expected outcomes. 1) requires teaching; LPN/LVN can reinforce teaching but cannot perform initial teaching 2) is not a stable patient with an expected outcome; requires assessment 3) is not a stable patient with an expected outcome; requires assessment 4) CORRECT— no indication that patient is not stable; myasthenia gravis is deficiency of acetylcholine at myoneural junction; symptoms include muscular weakness produced by repeated movements that soon disappear following rest

The nurse administers promethazine (Phenergan) 25 mg IM to a client complaining of nausea and vomiting. After receiving the medication, the client complains of dizziness when standing up. Which of the following actions should the nurse take FIRST? 1. Notify physician. 2. Monitor severity of symptoms. 3. Instruct client to ask for assistance before ambulating. 4. Assess client's hydration status.

Strategy: Complete assessment before implementing 1) complete assessment before contacting physician 2) is complaining of orthostatic hypotension; determine if fluid volume deficit contributing to dizziness 3) appropriate action, but nurse should first complete assessment 4) CORRECT— side effects include anorexia, dry mouth and eyes, constipation, orthostatic hypotension; client is at risk for fluid volume deficit due to vomiting, which exacerbates the orthostatic hypotension

The nurse in the outpatient clinic receives a call from a client who has been receiving continuous ambulatory peritoneal dialysis (CAPD) for 1 year. The client states that he infused 2 L of dialysate and 1200 cc returned. Which of the following statements by the nurse is BEST? 1. "Record the difference as intake." 2. "When was your last bowel movement?" 3. "Are you having shoulder pain?" 4. "Increase your fluid intake."

Strategy: Determine if it is appropriate to assess or implement. 1) the difference between inflow and outflow is counted as intake; ensure that all fluid has drained from the peritoneal cavity; change positions or ask client to walk around 2) CORRECT— full colon can create outflow problems; ensure that bowel evacuation has occurred 3) referred shoulder pain may be caused by rapid infusion of dialysate; instruct to decrease infusion rate; this client is having an outflow problem 4) will not affect outflow

Oxytocin challenge test (contraction stress test) is considered positive or abnormal if...

fetal bradycardia (FHT less than 110 bpm) or tachycardia (FHT greater than 160 bpm) or BP of mother rises above normal

An older patient is placed in balanced suspension traction for a compound fracture of the femur. The patient complains that her hands, feet, and nose feel cold. Which of the following actions should the nurse take FIRST? 1. Provide the patient with more blankets. 2. Assess for dependent edema. 3. Assess that patient is exhaling when moving in bed. 4. Increase the temperature of the room.

Strategy: Determine if it is time to assess or implement. 1) because of recumbent position, cardiac workload increases; if heart is unable to handle increased workload, peripheral areas of body will be colder; more important to assess cardiovascular status 2) CORRECT— edema caused by heart's inability to handle increased workload; assess sacrum, legs, and feet; also assess peripheral pulses 3) Valsalva maneuver increases workload on heart; to prevent, teach immobilized patients about exhaling when moving about in bed; should first assess patient complaints 4) assess the client; cold extremities may indicate heart is not able to tolerate increased workload

A tornado has just leveled a large housing division near the hospital, and a disaster alarm has been declared at the hospital. The nurse caring for clients on the maternal-child unit considers which of the following clients appropriate for discharge within the next hour? Select all that apply. 1. A multipara client who delivered over an intact perineum 12 hours ago. 2. A postpartum client with an infection who has been on antibiotics for the past 24 hours. 3. A 3-year-old with newly diagnosed type 1 diabetes, diarrhea, and vomiting. 4. A 3-day-old breast-feeding infant with a total serum bilirubin of 14 mg/dL. 5. A client at 34 weeks' gestation diagnosed with generalized edema and complaints of epigastric pain. 6. A 2-day-old infant delivered of a mother receiving intrapartum antibiotic therapy for vaginal group B-streptococcus (GBS).

Strategy: Determine the most stable clients. 1) CORRECT— stable patient 2) do not know if antibiotics are effective or the current WBC count 3) requires frequent assessment of hydration status and blood glucose levels 4) CORRECT— phototherapy considered for the infant with total serum bilirubin of >15 mg/dL at 72 hours of age 5) epigastric pain indicates pending eclampsia 6) CORRECT— group B streptococcal (GBS) disease causes sepsis; because mother received intrapartum prophylaxis, infant has 1-in-4,000 chance of developing sepsis due to GBS

The nurse receives report on the medical/surgical unit. Which of the following clients should the nurse see FIRST? 1. A client newly diagnosed with type 1 diabetes who had a myocardial infarction 2 days ago. 2. A client diagnosed with right-sided heart failure and glaucoma. 3. A client diagnosed with chronic obstructive pulmonary disease and psoriasis. 4. A client diagnosed with rheumatoid arthritis and malnutrition.

Strategy: Determine the most unstable client. 1) CORRECT— both diseases are in the dynamic phase and require close monitoring; most unstable client 2) client should be seen second 3) two chronic illnesses 4) client more stable than #1

The nurse plans care for a 14-year-old hospitalized with a diagnosis of anorexia nervosa. The nurse identifies that which of the following activities is MOST appropriate for this client? 1. Making jewelry with the occupational therapist. 2. Exercising in the physical therapy department. 3. Assisting the dietician to plan the week's menus. 4. Reading teen magazines with other patients her age.

Strategy: Determine the outcome of each answer. 1) CORRECT— one of the goals for a client with anorexia is to achieve a sense of self-worth and self-acceptance that is not based on appearance; this activity will promote socialization and increase self-esteem 2) goal is for client to achieve 85-95% of ideal body weight; may be able to exercise after short term goals are met 3) meal planning is a part of self-care activities, but more important for client to achieve a sense of self-worth 4) can read magazines in the presence of others without interacting

The nurse cares for an infant diagnosed with congenital heart disease. The nurse notes that the infant becomes easily fatigued during feedings and the infant's pulse and respirations increase. The nurse should take which of the following actions? 1. Feed the infant soon after awakening. 2. Change the infant's diaper before feeding. 3. Increase the caloric content of the feeding to 30 kcal/oz. 4. Mix rice cereal in the formula.

Strategy: Determine the outcome of each answer. Is it desired? 1) CORRECT— infant feeds better if well rested; offer small, frequent feeding every 3 hours; enlarge hole in nipple 2) will not affect infant's intake; pin diaper loosely to promote maximum chest expansion 3) allows infant to take in more calories in a smaller quantity; to prevent diarrhea, increase the calories by 2 kcal/oz/day; formulas provide 20 kcal/oz 4) infant would have to expend more energy to eat

A mother reports to the clinic nurse that her daughter developed a large welt, red rash, and shortness of breath after being stung by a bee. The mother asks the nurse, "What should I do if she gets stung again?" Which of the following responses by the nurse is BEST? 1. "Make a paste of baking soda and water and apply it to the sting." 2. "Remove the stinger and immediately apply ice to the site." 3. "Give 12.5 mg of Benadryl by mouth." 4. "Administer 0.3 mg of epinephrine subcutaneously."

Strategy: Determine the outcome of each answer. Is it desired? 1) treatment for sting in persons not allergic to bee stings; treats local reaction 2) not appropriate for this child because she has demonstrated hypersensitivity to bee sting; if no previous hypersensitivity; initial action is to remove stinger as quickly as possible to decrease the amount of venom injected into wound, wash with soap and water, apply cool compress 3) will not work fast enough to prevent anaphylactic reaction 4) CORRECT— child who has demonstrated previous hypersensitivity should have an EpiPen available; instruct child to wear medical identification bracelet

During the change-of-shift report, the charge nurse overhears two nurses exchanging loud, rude remarks about one nurse's excessive use of overtime. Which of the following statements by the charge nurse is MOST appropriate? 1. "I want to see both of you in my office right away." 2. "Would you please lower your voices and finish the report." 3. "I want the two of you to stop yelling and work this problem out." 4. "Both of you are good nurses and are under a lot of stress right now."

Strategy: Determine the outcome of each response. Is it appropriate? 1) confrontation is not the appropriate conflict management approach when emotions are high 2) CORRECT— forcing is the most appropriate conflict management technique; enables nurses to exchange information; client care takes priority over interpersonal conflict 3). need cooling-off period before issues can be discussed; communicating about patient care takes priority 4) "don't worry" response; may make the nurses feel better but does not address the immediate task of completing the report

Positive signs of pregnancy include...

fetal heart tones, Leopold's maneuver, which is manual external palpation of the fetal outline, ultrasound of the fetal outline

The nurse, caring for clients in the outpatient clinic, performs a chart review for clients who are receiving medication. The nurse determines that which of the following clients is at risk to develop problems with hearing? 1. A client receiving spironolactone (Aldactone) and cefaclor (Ceclor). 2. A client receiving metformin (Glucophage) and alendronate (Fosamax). 3. A client receiving paroxetine (Paxil) and cholestyramine (Questran). 4. A client receiving furosemide (Lasix) and indomethacin (Indocin).

Strategy: Think about each answer. 1) Aldactone is a potassium-sparing diuretic and Ceclor is a second-generation cephalosporin; neither drug is ototoxic 2) Glucophage is an oral hypoglycemic and Fosamax is a bone resorption inhibitor; neither is ototoxic 3) Paxil is a selective serotonin reuptake inhibitor (SSRI) and Questran is an antihyperlipidemic agent; neither is ototoxic 4) CORRECT— Lasix is a loop diuretic and is ototoxic, especially when given with other ototoxic drugs; Indocin is a NSAID and is also ototoxic

The home care nurse visits a client diagnosed with type 1 diabetes being managed with insulin in the am and pm. The nurse identifies that which of the following BEST measures the overall therapeutic response to management of the diabetes? 1. Glycosylated hemoglobin (HbA 1 c) 5% of total Hb. 2. Fasting blood sugar 128 mg/dL. 3. Blood pressure 130/82. 4. Serum amylase 100 Somogyi U/dL.

Strategy: Think about each answer. 1) CORRECT— indicates overall glucose control for the previous 120 days; normal is 4.5-7.6% of total hemoglobin 2) normal fasting is 60-110 mg/dL; HbA 1 c more accurate indicator of glucose control 3) evaluates response to antihypertensive medication 4) elevated in acute pancreatitis; normal is 60-160 Somogyi U/dL

The nurse prepares to admit a 6-month-old diagnosed with rotavirus, severe diarrhea, and dehydration. The nurse should place the infant in which of the following rooms? 1. In a semiprivate room with a 2-year-old in traction due to a fracture. 2. In a semiprivate room with a 9-month-old admitted for a shunt revision. 3. In a private room that is close to the nurse's station. 4. In any private room that is available.

Strategy: Think about the outcome of each answer. 1) a diapered or incontinent client diagnosed with rotavirus requires contact precautions for the duration of the illness; is a significant nosocomial pathogen 2) requires a private room; do not place a client with an infection in a room with a client who does not have an infection 3) CORRECT— rotavirus is spread by fecal-oral route and requires contact precautions if client is diapered or incontinent 4) due to severe nature of the symptoms requiring hospitalization, infant requires close observation for changes in condition

The nurse counsels the mother of a child diagnosed with impetigo. The nurse notes that the infection has not improved and learns the mother has not been caring for the child's skin because it "takes too much time." It is MOST important for the nurse to assess for which of the following? 1. White patches on buccal mucosa. 2. Hearing loss. 3. Respiratory wheezing. 4. Periorbital edema.

Strategy: What indicates a complication? 1) describes Candida , a fungal infection 2) not caused by impetigo 3) not caused by impetigo 4) CORRECT— impetigo is caused by Staphylococcus and Streptococcus ; untreated, can cause acute glomerulonephritis; periorbital edema indicates poststreptococcal glomerulonephritis

The nurse is helping to evaluate the effectiveness of bumetanide (Bumex) for a client with heart failure. Which of the following questions would provide the best information from the client? a. "Do your clothes fit differently?" b. "Do you have any ringing in your ears?" c. "How often during the day does your mouth feel dry?" d. "How many pillows do you use under your head while sleeping?"

The correct answer is D. A client with heart failure may report difficulty sleeping unless the head is supported on several pillows at night. To determine the effectiveness of Bumex, a diuretic, the nurse should ask about the number of pillows used during the night to sleep.

The nurse is assessing a client with chronic obstructive pulmonary disease (COPD). The client has lost about 5 kg (11 lb) during the past two months and reports moderate muscle weakness. Which of the following actions would be most appropriate for the nurse to take?

The correct answer is A. COPD is a high-energy disease and the client needs a higher caloric intake to help maintain weight and to prevent muscle weakness. Adding cheese to steamed vegetables adds calories. Steamed vegetables are also soft and easy to chew. Liquid nutritional supplements should be consumed at the end of meals or between meals. Drinking the supplement at the beginning of the meal may cause fatigue or early satiety resulting in decreased food intake by the client.

missed abortion

fetus dies in utero but is not expelled NC: D&C evacuation w/in 4-6 wks after 12 wks dilate cervix with prostglandin or laminaria

Enterocolitis

fever, bloody diarrhea, severe lethargy

The nurse is assessing a 7-day-old, full term infant who is breastfeeding. Which of the following observations should the nurse recognize as normal? a. The infant's weight is equal to the birth weight. b. The infant's conjunctivae are pale. c. The infant does not turn the head when the face is stroked. d. The infant has a bald spot over the occiput.

The correct answer is A. A full term infant who is breastfeeding typically regains weight and by one week is at the birth weight. A bald spot over the occiput may indicate child maltreatment associated with leaving the child supine in the crib for prolonged periods of time.

The nurse is preparing a 3-year-old child for a bone marrow aspiration from the iliac crest. Which of the following statements would be appropriate for the nurse to make to help ensure the child's cooperation during the procedure? a. "You will feel like someone is pressing the lower part of your tummy." b. "A small needle will be used to get important cells from your bone." c. "Take several deep breaths when I tell you to so you won't feel any pain." d. "After the dressing is put on your skin you will be able to go home."

The correct answer is A. A toddler needs simple explanations about what will be seen, heard, and felt. Advising the child that it will feel like someone is pressing on the lower part of the tummy will help the child understand the location of the procedure and the sensation to expect. The toddler is less likely to understand what a dressing means.

The home health nurse has assessed a client with a stage 3 pressure sore on the left foot that has been present for over 3 months without any improvement. The client has a normal serum protein level and has been non-weight bearing on the left foot since the pressure sore was diagnosed. The client is currently applying a topical medication and dry dressings to the site as prescribed. Which of the following individuals would be appropriate to consult initially regarding the client's care?

The correct answer is A. An enterostomal therapist is trained to evaluate and develop treatment plans for complex wounds, such as the wound described in the question.

Diagnostic tool for DIC is presence of...

fibrin split compound.

The nurse is contributing to a staff development conference regarding legal responsibilities of health care providers. Which of the following information, if documented in a client's record, should the nurse identify as increasing the risk for a charge of libel? a. Court date pending for charge of child molestation. b. Stab wound to the chest after incidence of domestic violence. c. Unprotected intercourse with multiple sexual partners. d. Use of crack cocaine in social settings.

The correct answer is A. Defamation of character is making false or malicious (intentionally harmful) statements that may harm another person's character or reputation. Oral statements are considered slander and written statements are considered libel. Documenting that a client has a pending charge for child molestation would increase the risk for a charge of libel. The history of unprotected sex with multiple sex partners is pertinent to the client's care.

The nurse is teaching a client with primary hypertension about methods to prevent hypokalemia since the client will start taking furosemide (Lasix) as prescribed. The nurse should advise the client that which of the following foods is highest in potassium per serving? a. dry milk b. dried beans c. apples d. strawberries

The correct answer is A. Dry milk is highest in potassium per serving of the foods listed.

The nurse is caring for an adult client with acute gastroenteritis. Which of the following observations by the nurse may indicate that the client is becoming dehydrated? a. pulse change from 68 to 80 b. blood pressure change from 110/78 mm Hg to 120/80 mm Hg c. musty urine odor d. hypoactive bowel sounds

The correct answer is A. Indications of dehydration include flushed skin, skin tenting, dry mucous membranes, hypotension or tachycardia. The increasing pulse rate may indicate the client is becoming dehydrated.

The unit clerk informs the nurse about the following client requests. The nurse should first check the client with a. primary hypertension who requests an analgesic for a headache b. peripheral vascular disease who requests lotion for burning in the feet c. diabetes mellitus who requests powder for a red area of skin in the axilla d. chronic obstructive pulmonary disease (COPD) who requests water for the humidifier

The correct answer is A. Maslow's hierarchy of human needs provides a framework to determine priorities. The client with hypertension who requests medication for a headache may be developing a potentially life-threatening change in cerebral perfusion, such as a cerebrovascular accident and should be checked first. The client with COPD should have increased humidity to help minimize drying of secretions but this request, if delayed, is not life-threatening.

The nurse is assessing an adult client who was just diagnosed with primary hypertension. Which of the following questions would be essential for the nurse to ask the client? a. "When was your last eye examination?" b. "How often do you receive pedicures?" c. "What medication do you take when you have a headache?" d. "Have you ever had difficulty urinating?"

The correct answer is A. Primary hypertension causes changes in blood flow to target organs, such as the eyes and kidneys. It would be essential for the nurse to ask when the client had an eye examination. It would be most important for the nurse to identify the frequency and severity of headaches rather than the treatment of headaches since the client is at risk for a CVA.

The nurse is conducting a community education program regarding cancer. Which of the following statements would be correct for the nurse to make? a. "Prostate cancer is more common in African-American males." b. "Breast cancer risks increase for women if oral contraceptives have been used during adolescence." c. "Testicular cancer is common after the age of sixty years." d. "Colon cancer screening begins after the age of fifty years regardless of ethnic origin."

The correct answer is A. Prostate cancer is more common in African-American males and usually has a poor prognosis at the time of diagnosis. Colon cancer screening typically begins at the age of 50 years but is recommended starting around 40 years if the individual is African-American.

The nurse is participating in a community-based health fair. The nurse should recognize that which of the following clients is at increased risk for suicide and should receive a depression screening questionnaire? a. an 80-year-old client with mild Alzheimer's disease (AD) b. a 48-year-old client with psoriasis c. a 16-year-old client who has changed schools d. an 8-year-old client with diabetes mellitus, type 1

The correct answer is A. Risk factors for suicide include being elderly, chronic illness, social isolation and individuals who misuse alcohol or other drugs. The 80-year-old client with mild AD is at increased risk for injury since the client is aware of the cognitive changes that are occurring. Also, this individual typically has multiple losses. This client should receive a depression screening questionnaire. A client who changed schools is at lower risk for depression based on the answer choices given.

The nurse is feeding a full term newborn who is on continuous cardiac monitoring. The infant starts to cough, the alarm on the monitor starts to ring and the newborn resumes sucking. The nurse should next a. silence the alarm b. check the monitor settings c. gently pat the newborn's back d. replace the electrodes on the newborn's chest

The correct answer is A. The cardiac alarm may sound whenever there is a change in infant activity, such as coughing. Coughing is a protective reflex and indicates an effort to clear the airway. Based on the question, the nurse should next silence the alarm and then check the electrodes.

The nurse has reinforced teaching with a client who is scheduled for a cardiac catheterization via the femoral artery. Which of the following statements by the client would indicate a correct understanding of the teaching? a. "I should avoid coughing or sneezing for several days after the procedure is completed." b. "I will be asked to hold my breath when the catheter is removed from my groin." c. "I can sit in a chair when I am in the recovery area after the procedure is completed." d. "I may have temporary residual numbness in my leg due to irritation of the nerves caused by the contrast dye."

The correct answer is A. The femoral artery is the most common site of catheter insertion for a cardiac catheterization. A client is discouraged from coughing or sneezing after the procedure to help minimize bleeding at the catheter insertion site. The client does not need to hold the breath during the procedure.

The nurse is assessing a client who is at 28 weeks gestation. Which of the following tests is common at this stage of pregnancy? a. alpha-fetoprotein (AFP) level b. contraction stress test (CST) c. glycosylated hemoglobin (HbA1C) level d. one-hour glucose tolerance test (GTT)

The correct answer is D. A one-hour GTT is typically performed at this stage of pregnancy. AFP is performed during an amniocentesis if fetal anomalies are suspected.

The nurse is caring for a client who has end-stage chronic obstructive pulmonary disease (COPD). The client receives continuous oxygen via nasal cannula and bronchodilators via nebulizer q.i.d as prescribed. Which of the following actions by the nurse would increase the risk for a charge of negligence? a. using a container of acetone to remove adhesive from the client's skin b. allowing the client to use an electric razor to shave c. making sure that the client's undershirt is made from cotton d. mixing a hydrogen-peroxide solution for the client to use as a mouth rinse

The correct answer is A. The nurse must remain aware of standards of care for clients who are receiving oxygen therapy and take action to minimize the risk for injury to the client. Negligence is defined as failure to provide care that meets the standard and that places the client at risk for injury. Using acetone is a potential fire hazard when oxygen is being used so this action increases the risk for a charge of negligence if the client and/or others are adversely affected by this action.

The home health nurse is checking a client who had an open cholecystectomy and insertion of a biliary drainage tube (T-tube). Which of the following actions would be appropriate for the nurse to take? a. laying a plastic garbage bag across the client's legs before removing the T-tube dressing b. rotating the T-tube 90 degrees before cleaning around the tube c. applying gentle pressure on both sides of the T-tube to check for leaks d. obtaining a specimen for culture and sensitivity testing from the yellow drainage in the T-tube

The correct answer is A. The nurse should maintain infection control procedures when dressings are changed in a client's home. A water-proof bag, such as a trash bag, can be placed over the client's legs to place soiled dressings and to minimize contamination of the immediate area.

The nurse is assessing a client who had a partial thyroidectomy 24 hours ago. The nurse must a. check the client's pupillary reaction to light b. observe the response when the client's cheek is tapped c. measure the client's neck circumference d. auscultate the client's carotid pulses

The correct answer is B. A client who had a partial thyroidectomy is at risk for a calcium imbalance. The nurse should tap the client's cheek and observe for muscle twitching (Chvostek's sign) which may indicate hypocalcemia. The client's neck circumference is not measured.

The nurse is assessing a client who is receiving total parenteral nutrition (TPN). The nurse observes that the client's skin is warm and the client's pulse rate is increased. The client reports feeling weak and confused. The nurse should first check the client's

The correct answer is B. A client who is receiving TPN is at risk for fluid and electrolyte imbalances. Hyperglycemia may be indicated by warm skin, tachycardia, muscle weakness and change in mental status. The nurse should first check the client's blood glucose level so that appropriate care can be initiated. The client's oral temperature can be checked after the blood glucose level is checked.

The nurse is teaching a client who was recently diagnosed with a generalized seizure disorder. Which of the following should the nurse state has been associated with the onset of seizure activity? a. drinking cold beverages b. sunlight flickering through tree branches c. working in a noisy environment d. the smell of burning debris

The correct answer is B. A client with a generalized seizure disorder should be taught methods to help minimize seizure activity. Flickering lights have been identified as a stimulus for seizure activity.

The charge nurse in the long-term care facility is scheduling procedures for several clients as prescribed. Which of the following clients should the nurse make sure is scheduled for the first appointment in the morning?

The correct answer is B. A client with a low WBC is at risk for infection and should have procedures scheduled at the first available appointment time to help minimize the client's risk for infection in a busy or crowded waiting area. The client with an elevated BUN is at lower risk for a potentially life-threatening infection.

The nurse has assessed a client with acute renal failure. The nurse should immediately report that the client has a. voided urine that looks like water b. frothy pink sputum c. not had a bowel movement in 3 days d. 1+ sacral edema

The correct answer is B. A client with acute renal failure is at risk for heart failure and pulmonary edema. The presence of frothy pink sputum requires immediate action by the nurse.

The home health nurse is talking with a client who has hepatic cirrhosis and esophageal varices. The client reports losing about 2.2 kg (5 lbs) during the past 4 weeks without any attempt at weight loss. Which of the following should the nurse recommend to help the client maintain weight?

The correct answer is B. A client with hepatic cirrhosis and esophageal varices is at risk for altered nutrition. The client should avoid foods that are hard, like popcorn and fresh fruit, since this may increase the risk for bleeding. Eating fruit canned in syrup would help to add calories and vitamins with minimal risk for bleeding.

The nurse is assessing an adult client with hypothyroidism who is taking prescribed levothyroxine (Synthroid). Which of the following would indicate that the client's treatment has been effective? a. sleeping 6 hours a day b. daily bowel movements c. neck fullness d. pulse, 100

The correct answer is B. A client with hypothyroidism may experience weight gain, anorexia, cold intolerance, constipation, lethargy or slowed movements. Having a daily bowel movement would indicate that the client's treatment has been effective. Sleeping 6 hours a day requires additional information to help determine if this is a change in the typical sleep pattern since excess thyroid hormone replacement can cause insomnia.

The nurse is reviewing the record of a client who has several new prescriptions. The client has a history of coronary artery disease (CAD) and renal insufficiency. Which of the following new medications would be contraindicated based on this history and should be clarified by the nurse? a. metoprolol (Toprol XL) b. hydrochlorothiazide (HydroDIURIL) c. atorvastatin (Lipitor) d. clopidogrel (Plavix)

The correct answer is B. A client with renal disease should not take HydroDIURIL since this medication further compromises renal function.

The nurse is caring for a client who had an abdominal hysterectomy 48 hours ago. The client reports abdominal pain rated 8 on a scale of 0 (no pain) to 10 (severe pain). Which of the following medications prescribed p.r.n. would be appropriate for the nurse to administer to the client? a. ketorolac (Toradol) b. metoclopramide (Reglan) c. bisacodyl (Dulcolax) d. meperidine hydrochloride (Demerol)

The correct answer is D. Demerol should be administered for moderate to severe pain. A pain scale is used to determine the severity of pain and a rating of 8 indicates moderate to severe pain.

The nurse is reinforcing preoperative teaching with a client who has signed a surgical consent and is scheduled for a radical prostatectomy. Which of the following statements by the client would increase the risk for a charge of battery if the surgery is performed as scheduled? a. "I know that I may have pink-colored urine for several weeks after surgery." b. "I will be able to resume sexual activity within six weeks of surgery." c. "I should avoid becoming constipated after surgery." d. "I can donate my own blood for transfusion since this surgery can cause severe anemia."

The correct answer is B. Informed consent means that the client has received and understands the surgical procedure as well as the risks and benefits of treatment to include specific details regarding complications. Battery is touching a person without consent with or without harm to the client. A radical prostatectomy increases the risk for sexual dysfunction. The client's statement about resuming sexual activity indicates the client does not have a correct understanding of the surgical procedure and increases the risk of a charge of battery if the surgery proceeds without providing the client with additional information regarding sexual function after surgery.

The nurse is preparing to administer enoxaparin (Lovenox) as prescribed to a client. Which of the following actions would be appropriate for the nurse to take? a. Check the client's activated partial thromboplastin time (APTT) before giving the Lovenox. b. Inject the Lovenox at a 90 degree angle into the client's abdomen. c. Pull the plunger on the syringe backward to check for blood before injecting the Lovenox. d. Verify the client's current weight.

The correct answer is B. Lovenox is injected either at a 45 or 90 degree angle into the client's abdomen. The APTT is not routinely monitored when Lovenox is administered.

The nurse is preparing a client for discharge after the client had a colonoscopy with removal of a rectal polyp. Before the client is discharged, the nurse must a. measure the client's abdominal girth b. check the client's blood pressure c. determine if the client has expelled any flatus d. assess the client's gag reflex

The correct answer is B. Prior to discharging a client, the nurse must make sure that a client is not experiencing any complications. After a colonoscopy, the client is at risk for hemorrhage and bowel perforation. The nurse must check the client's blood pressure before the client is discharged. The client is not required to expel flatus prior to discharge.

The nurse is caring for a 72-year-old client who is receiving percutaneous endoscopic gastrostomy (PEG) tube feedings. The client has residual dysphagia and hemiparesis due to a cerebrovascular accident (CVA). The nurse observes that the client is having difficulty remaining alert and has the new onset of confusion. The nurse should immediately check the client's a. abdominal girth b. oxygen saturation level c. peripheral pulses d. gastric residual

The correct answer is B. Risk factors for pneumonia include immobility, advanced age, aspiration of gastric contents and a suppressed immune system. Geriatric-aged clients have a suppressed immune system and may not have typical fever and respiratory changes when an infection develops. Based on the history presented in the question, the nurse should immediately check the client's oxygen saturation level since a change in mental status may indicate hypoxia associated with aspiration pneumonia.

The parent of an 8-day-old infant watches television in the room while the home health nurse completes a weight check on the infant. The infant's weight is the same as the birth weight and the parent reports that the infant does not like to be touched. Which of the following additional observations of the infant by the nurse would indicate that child protective services must be contacted? a. dried formula on the chin b. reddened occiput c. cheesy substance in the axilla d. edematous breasts

The correct answer is B. The home health nurse must remain alert for indications of child maltreatment, such as failure to gain weight, avoidance of contact with the infant and indications of bruising that may indicate inappropriate aggression toward the infant. Child protective services must be contacted by law if the infant's occiput is reddened since this indicates pressure associated with the infant remaining in the supine position. Vernix caseosa is a cheesy substance that is normally present in the axilla and other skin folds of a newborn.

The nurse is conducting a staff development conference regarding internal disasters. The nurse should state that if building evacuation is required, priority should be given to evacuating clients who are a. receiving continuous oxygen b. ambulatory c. comatose d. sitting in wheelchairs

The correct answer is B. The nurse must establish evacuation plans in case of an internal disaster. Priority is given to evacuating the ambulatory clients. This can be accomplished by having a staff member lead the clients to the designated exit.

The nurse has been reassigned to the emergency department (ED) due to a staff shortage. The nurse is assigned to irrigate a superficial stab wound but the nurse has never performed a wound irrigation. To help prevent a charge of negligence, the nurse should initially a. refuse to irrigate the client's wound b. review the procedure for wound irrigation c. ask another staff member to help d. ask the nursing supervisor to come to the ED

The correct answer is B. The nurse must possess the appropriate knowledge, skills and abilities to safely provide care to assigned clients. When a nurse is asked to float to an unfamiliar area and complete an unfamiliar task, the nurse should review the procedure for wound irrigation. The nurse can be charged with negligence if the task is not completed according to the agency policy. The nurse is responsible for the correct performance of an assigned task by the nurse or if delegated to another staff person. Asking for help would be appropriate after the nurse reviews the procedure.

The nurse is reviewing home safety with a 79-year-old client. Which of the following should the nurse identify as a safety hazard for the client that needs to be corrected? a. an unopened can of a liquid nutritional supplement is sitting next to the gas stove b. a bottle of paint thinner is stored next to a can of cooking oil c. a towel soaked with alcohol is sitting next to the washing machine d. a tube of denture adhesive is stored next to a tube of toothpaste

The correct answer is B. The nurse should remain alert to safety hazards in a client's home. Storing potentially toxic chemicals with food items is a safety hazard.

Biliary atresia

fibrosis of intrahepatic and extrahepatic bile ducts and gradually results in liver failure

The nurse is teaching a client who was recently diagnosed with hyperlipidemia and diabetes mellitus. The client has new prescriptions for atorvastatin (Lipitor) and rosiglitazone (Avandia). Which of the following statements would be correct for the nurse to make? a. "You will need to stop taking Avandia if you need to start taking insulin." b. "A common side effect of Lipitor is a metallic taste in your mouth." c. "Avandia may need to be changed to a different medication if your legs swell." d. "Lipitor may cause your blood glucose level to increase."

The correct answer is C. Avandia may cause peripheral edema that may necessitate changing to a different medication. Lipitor has not been associated with altered taste. The correct answer is C. Avandia may cause peripheral edema that may necessitate changing to a different medication. Lipitor has not been associated with altered taste. The correct answer is C. Avandia may cause peripheral edema that may necessitate changing to a different medication. Lipitor has not been associated with altered taste.

The nurse is completing the preoperative check list on a client who is scheduled for a cystectomy and urinary diversion. The nurse observes that the operative consent is signed by the client and was witnessed, the client's laboratory test results are within the normal range and the client's electrocardiography (EKG) is normal. The nurse should immediately report which of the following statements by the client? a. "My biggest fear is not waking up after surgery." b. "I hope I don't need a blood transfusion since I was not able to bank my own blood." c. "I am glad that I will only have a tube in my bladder for a couple of days." d. "My last bowel movement was two days ago."

The correct answer is C. During the preoperative time interval the nurse must make sure that the client's record is complete and that the client has completed any preoperative preparation that is required. The nurse should immediately report the client's statement regarding the tube in the bladder since this indicates the client does not understand that a cystectomy and urinary diversion means the client's bladder is being removed. If the nurse fails to report this, the facility is at risk for a charge of battery. The statement regarding the potential need for a transfusion indicates that the client understands the preoperative teaching and does not require notification of any members of the surgical team.

The nurse in the long-term care facility is caring for the following assigned clients. The nurse would increase the risk of a charge of false imprisonment if the nurse administers a. a sedative prescribed p.r.n. to a client with moderate Alzheimer's disease who is agitated at sundown b. a sedative prescribed p.r.n. to a client with an inoperable brain tumor who is too restless to sleep c. an opioid analgesic prescribed p.r.n. to a client with metastatic bone cancer who refuses to sleep during the night d. a skeletal muscle relaxant prescribed p.r.n. to a client with fibromyalgia who reports skin sensations of crawling bugs

The correct answer is C. False imprisonment is confining or restricting a client's movement by using physical or verbal constraint without the client's consent. Administering medications to control a client's behavior for the convenience of staff is a form of false imprisonment, such as administering an analgesic to a client who refuses to sleep at night.

The nurse is caring for a client with chronic low back pain. The client reports pain rated 8 on a scale of 0 (no pain) to 10 (severe pain). The client received a prescribed oral analgesic 5 minutes ago. Which of the following actions by the nurse may enhance the client's comfort? a. turning the client onto the left side with both knees flexed to the chest b. elevating the client's legs above the level of the heart while lying in bed c. placing a rolled towel behind the client's back while sitting d. assisting the client to lay on the abdomen with the arms raised above the head

The correct answer is C. Nonpharmacologic methods to enhance the comfort of a client with low back pain include sitting with a pillow to support the back, resting supine with pillows under the legs and resting in the lateral position with a pillow between the knees. If there is leg pain, the painful leg is flexed while the client is in the lateral position.

The nurse enters the room of a client who has metastatic cancer and who has a prescription not to resuscitate. The nurse observes that the client has sighing respirations and the client's extremities are cool. There has been no urine in the client's indwelling urinary catheter in over 12 hours. The nurse should a. dim the lights in the client's room before returning to the nurse's station b. elevate the client's legs above the level of the head c. remain at the client's bedside until the family member returns from the cafeteria d. administer an analgesic prescribed p.r.n. for pain

The correct answer is C. Palliative care for a client with terminal cancer includes offering comfort and dignity to the client during the dying process. The information in the question is an indication that the client's body systems are shutting down and that death is imminent. The nurse should remain at the client's bedside until the family member returns. The information in the question does not indicate the client is in pain, such as agitation, therefore the nurse should not administer an analgesic unless requested by a designated family member.

The nurse is teaching a client who is to start total parenteral nutrition (TPN). The client asks the nurse how the decision is made to use TPN compared to nasogastric (NG) tube feedings. Which of the following statements would be correct for the nurse to make? a. "Total parenteral nutrition is more cost effective for a client than NG tube feedings since the nutritional value is higher quality." b. "Nasogastric tube feedings cause a client to have more oral infections than TPN." c. "Total parenteral nutrition is chosen when a client is not able to absorb nutrients from the intestinal tract." d. "Nasogastric tube feedings are beneficial for clients who do not have a gag reflex but who require enteral nutrition for less than a month."

The correct answer is C. TPN is provided when a client is not able to absorb nutrients through the gastrointestinal tract, such as with inflammatory bowel disorders, cancer and malnutrition. TPN requires regular laboratory tests to determine the appropriate components based on evaluation of the client's changing nutritional needs. There is no evidence to support that TPN is more cost effective than NG feedings. NG tube feedings are administered when a client's protective reflexes are intact, such as the gag reflex and is used when the client needs nutritional support for less than 5 days. In addition, the client's gastrointestinal tract must be able to absorb the nutrients.

The charge nurse has reviewed several irregular occurrence reports during the past month associated with unwitnessed falls. To help resolve these occurrences, the nurse should first a. evaluate the layout of each client care area. b. interview the staff members who completed the irregular occurrence reports. c. identify what prescribed medications were given to the clients. d. determine if any equipment repairs have occurred in the areas of falls.

The correct answer is C. The charge nurse should be familiar with the problem-solving process when irregular occurrences are identified. The first step in the problem solving process is assessing client characteristics, such as types of medications that the clients received. Risk for falls include medications, lighting, muscle weakness and delay of staff in answering call lights. Interviewing staff members who discovered the incidents has lower priority than identifying client characteristics.

The nurse is reviewing the laboratory test results of an adult client who had an annual physical examination. Which of the following laboratory test results is within the normal range? a. white blood cell (WBC) count, 23,000/cu mm b. glycosylated hemoglobin (HgbA1C), 8.2% c. serum potassium, 3.7 mEq/L d. urine specific gravity, 1.000

The correct answer is C. The normal serum potassium level is 3.5 to 5 mEq/L; HgbA1C less than 6% is normal or less than 7% indicates glycemic control for a client with diabetes mellitus; a normal WBC is 5,000 to 10,000/cu mm; and the urine specific gravity is typically between 1.010 to 1.025.

The nurse is caring for a client who has an abdominal aortic aneurysm. The client requests medication for back pain rated 8 on a scale of 0 (no pain) to 10 (severe pain). Which of the following actions should the nurse take first ?

The correct answer is C. The nurse must be able to identify a client's risk for complications associated with an abdominal aortic aneurysm. If the client reports back pain, the nurse must first identify the quality of the pain to help identify if a potentially life-threatening emergency is occurring, such as aortic dissection. If the client's pain is knife-like, this is an emergency that requires action to prevent hemorrhage.

The nurse is reviewing the record of a 62-year-old female client who was just admitted. The client had a cerebrovascular accident (CVA) and also has osteoporosis. The client receives continuous percutaneous endoscopic gastrostomy (PEG) tube feedings and the client's prescribed medications are administered via the PEG tube. Which of the following prescribed medications should the nurse clarify before administering the medication to the client? a. oxybutynin (Ditropan) b. paroxetine (Paxil) c. warfarin sodium (Coumadin) d. estrogen (Premarin)

The correct answer is D. A cerebrovascular accident may be caused by hemorrhage or thrombus. The nurse should clarify the prescription for Premarin since a history of thrombus is a contraindication for Premarin. Coumadin can be administered via the PEG tube and is not contraindicated based on the information presented in the question.

The nurse is assessing a client who had a colonoscopy an hour ago. The client reports feeling weak and dizzy and the client's blood pressure has changed from 118/70 mm Hg to 104/56 mm Hg. Which of the following actions should the nurse take first? a. Notify the client's primary health care provider b. Check the client's temperature c. Auscultate the client's bowel sounds d. Elevate the client's legs above the level of the heart

The correct answer is D. A client who had a colonoscopy is at risk for bowel perforation and hemorrhage. The onset of weakness, dizziness and a decreasing blood pressure indicates possible hypovolemia and the nurse should elevate the client's legs above the level of the heart to help maintain cerebral tissue perfusion.

The nurse is assessing a client who had a total hip arthroplasty 2 days ago. The nurse observes that the client's pulse is increased, the client reports feeling anxious and has pain in the operative leg. The nurse's initial response should be based on an understanding that the client may have a. a wound infection b. a dislocated hip prosthesis c. atelectasis d. a pulmonary embolism

The correct answer is D. A client who had a hip prosthesis is at risk for deep vein thrombosis and pulmonary embolism. A pulmonary embolism may be indicated by the development of tachycardia and anxiety. The nurse's initial response should be based on an understanding of pulmonary embolism since this is a potentially life threatening condition. A wound infection is not life-threatening and should not be the basis for the nurse's first response.

The nurse has taught a client who just had drainage of a pilonidal cyst. Which of the following statements by the client would indicate a correct understanding of the teaching? a. "I need to rest with my legs elevated on pillows." b. "I should refrain from sexual activity until the pain is gone." c. "I cannot use any rectal medications until this is healed." d. "I should sit in a warm tub of water several times a day."

The correct answer is D. A client who had a pilonidal cyst drained should sit in a warm tub of water to help enhance healing. The client may be more comfortable by sitting on cushioned surfaces which minimize pain in the region of the coccyx.

The nurse is assessing a client with preterm labor at 32 weeks gestation. The client is receiving terbutaline sulfate (Bricanyl) as prescribed. The client reports feeling palpitations and leaking urine when coughing or sneezing. Which of the following actions should receive priority by the nurse? a. measuring the client's fundal height b. checking the client's weight c. obtaining a electrocardiography (EKG) on the client d. using nitrazine paper to check the moisture in the client's underwear

The correct answer is D. A client who is being treated for preterm labor is monitored for indications of a change in status associated with medications and effectiveness of the treatment regime. The nurse should give priority to using nitrazine paper to check the moisture in the client's underwear since this may be an indication of leaking or rupture of amniotic membranes. A potentially life-threatening infection can develop if the client has prolonged rupture of membranes. Palpitations are a common side effect of Bricanyl. The nurse should check the client's vital signs.

The nurse is assessing a client who reports frequent sore throats and swollen glands beneath the jaw. The nurse observes that the client is 20% above the ideal body weight and that the client's weight has fluctuated by about 4.5 kg (10 lb) during the past few months. Which of the following questions would be essential for the nurse to ask the client? a. "Have you noticed any change in your vision?" b. "How often do you eat?" c. "What type of beverages do you enjoy?" d. "Do you feel any heart palpitations?"

The correct answer is D. A client with a binge-purge eating disorder may have frequent sore throats and a chipmunk facial appearance due to swelling of the parotid glands. The nurse should ask the client about palpitations since a client with a binge-purge eating pattern is at risk for cardiac dysrhythmias.

3. knowledge application

The nurse asks a patient how she feels about her impending surgery for breast cancer. Before the discussion the nurse reviewed the description of loss and grief and therapeutic communication principles in his textbook. The critical thinking component involved in the nurse's review of the literature is: 1. experience 2. problem solving 3. knowledge application 4. clinical decision making

What are some common RN interventions to prevent/ minimize paralytic ileus?

The nurse can prevent/minimize paralytic ileus after surgery with pt positioning and early ambulation. Evidence of bowel function returning to normal includes auscultation of bowel sounds and passing of flatus and stool.

Post EGD procedure pt would need to remain NPO until ___________ :

gag reflex returns (pt is at risk for aspiration until it has returned)

2. Description of the efforts to restore the child's blood pressure, what was used, ad questions about the child's response 3. The meaning the experience had for the nurse with respect to her understanding of dealing with a patient's death 4. A description of what the nurse said to the mother, the mother's response, and how the nurse might approach the situation differently in the future

The nurse cared for a 14-year old with renal failure who dies near the end of the work shift. The health care team tried for 45 minutes to resuscitate the child with no success. The family was devastated by the loss, and, when the nurse tried to talk with them, the mother said, "You can't make me feel better; you don't know what it's like to lose a child." Which of the following examples of journal entries might best help the nurse reflect and think about this critical experience? (Select all that apply.) 1. Data entry of time of day, who was present, and condition of the child 2. Description of the efforts to restore the child's blood pressure, what was used, ad questions about the child's response 3. The meaning the experience had for the nurse with respect to her understanding of dealing with a patient's death 4. A description of what the nurse said to the mother, the mother's response, and how the nurse might approach the situation differently in the future

2. Leave a night light on in the bathroom. 6. Provide scheduled toileting during the night shift. 7. Keep the pathway from the bed to the bedroom clear.

The nurse found a 68-year old female patient wandering in the hall. The patient says she is looking for the bathroom. Which interventions are appropriate to ensure the safety of the patient? (Select all that apply.) 1. Insert a urinary catheter. 2. Leave a night light on in the bathroom. 3. Ask the physician to order a restraint. 4. Keep the bed in low position with upper and lower side rails up. 5. Assign a staff member to stay with the patient. 6. Provide scheduled toileting during the night shift. 7. Keep the pathway from the bed to the bedroom clear.

3. The patient is short of breath.

The nurse has a patient who is short of breath and calls the health care provider using SBAR (situation-background-assessment-recommendation) to help with the communication. What does the nurse first address? 1. The respiratory rate is 28. 2. The patient has a history of lung cancer. 3. The patient is short of breath. 4. He or she requests an order for a breathing treatment.

4. "I'm going to attend a support group to learn more about multiple sclerosis."

The nurse is evaluating the coping success if a patient experiencing stress from being newly diagnosed with multiple sclerosis and psychomotor impairment. The nurse realizes that the patient is coping successfully when the patient says: 1. "I'm going to learn to drive a car so I can be more independent." 2. "My sister says she feels better when she foes shopping, so I'll go shopping." 3. "I've always felt better when I go for a long walk. I'll do that when I get home." 4. "I'm going to attend a support group to learn more about multiple sclerosis."

2. When the patient's pain medications are working 3. Just before lunch, when the patient is most awake and alert

The nurse is planning to teach a patient about the importance of exercise. When is the best time for teaching to occur? (Select all that apply.) 1. When there are visitors in the room 2. When the patient's pain medications are working 3. Just before lunch, when the patient is most awake and alert 4. When the patient is talking about current stressors in his or her life

1. Role play

The nurse is teaching a parenting class to a group of pregnant adolescents. The nurse pretends to be the baby's father, and the adolescent mother is asked to show how she would respond to the father if he gave her a can of beer. Which teaching approach did the nurse use? 1. Role play 2. Discovery 3. An analogy 4. A demonstration

At 3-6 months, the infant...

gains control of the head, rolls from abdomen to back, sits with support, and can move hand to mouth. Lower centeral incisors erupt. Socializes by imitating sounds and laughing loud.

4. Search for identity with peer groups and separating from family

The nurse plans care for a 16-year old male, taking into consideration that stressors experienced most commonly by adolescents include which of the following? 1. Loss of autonomy caused by health problems 2. Physical appearance, family, friends, and school 3. Self-esteem issues, changing family structure 4. Search for identity with peer groups and separating from family

Alpha-fetoprotein

glucoprotein produced by fetal yolk sac, GI tract, and liver.

3. Remove all patients in immediate danger

The nurse's first action after discovering an electrical fire in a patient's room is to: 1. Activate the fire alarm 2. Confine the fire by closing all doors and windows 3. Remove all patients in immediate danger 4. Extinguish the fire by using the nearest fire extinguisher

Blood glucose levels in the mother should be reported to the doctor

over 120 mg/dl

1. Smoking is prohibited around oxygen. 3. Do not use electrical equipment around oxygen. 4. Special precautions may be required when traveling with oxygen.

To ensure the safe use of oxygen in the home by a patient, which of the following teaching points does the nurse include? (Select all that apply.) 1. Smoking is prohibited around oxygen. 2. Demonstrate how to adjust the oxygen flow rate based on patient symptoms. 3. Do not use electrical equipment around oxygen. 4. Special precautions may be required when traveling with oxygen.

SEIZURE : S/S Complication Crisis

Tonic Phase: Loss of consciousness; muscles contract 10-20 sec Clonic Phase: rhythmic contraction <2min Aura: warning sx Crisis Preventions: HAVE SUCTION, AIRWAY, O2 AT BEDSIDE! Protect pt: lower to floor, pad siderails, pillow under head, don't restrain, allow post-ictal rest. Prevent Aspiration: turn side, loosen neck clothing, suction. Ongoing: Monitor VS, LOC, O2 saturation, Glasgow coma scale, reassure & orient pt after seizure

Librium (anti-anxiety) side effects

pancytopenia, thrombocytopenia, drowsiness and blurred vision. administer with after meals or with milk to decrease GI irritaion

This drug is an antiemetic that should never be given IV and when given IM MUST be Z-tracked

Vistaril (hydroxyzine) - is also antihistamine and antidepressant that is often used for post-op pts

Hyperthyroidism Nursing Interventions

Vital signs including weight Antithyroid meds Thyroidectomy pt teaching monitor for s/s of Thyroid Storm

Hyperthyroidism S/S

Weight loss with increased appetite Exophthalmos (bulging eyes) Heat intolerance Menstrual problems- amenorrhea Insomnia hyperexcitable

2. Analogy

When a nurse is teaching a patient about how to administer an epinephrine injection in case of a severe allergic reaction, he or she tells the patient to hold the injection like a dart. Which of the following instructional methods did the nurse use? 1. Telling 2. Analogy 3. Demonstration 4. Simulation

3. A thorough physical assessment

When assessing an older adult who is showing symptoms of anxiety, insomnia, anorexia, and mild confusion, one of the first assessments include which of the following? 1. The amount of family support 2. A 3-day diet recall 3. A thorough physical assessment 4. Threats to safety in her home

posttraumatic stress disorder (PTSD)

When doing an assessment of a young woman who was in an automobile accident 6 months before, the nurse learns that the woman has vivid images of the crash whenever she hears a loud, sudden noise. The nurse recognizes this as ___________________.

2. "When you brush me off like that, it takes me even longer to do my job."

You ask another nurse how to collect a laboratory specimen. The nurse raises her eyebrows and asks, "Why don't you figure it out?" what would be the best response? 1. Say nothing and walk away. Find a different nurse to help you. 2. "When you brush me off like that, it takes me even longer to do my job." 3. "Why do you always put me down like that?" 4. "I guess I just enjoy having you make fun of me."

2. Pulling the curtain to provide privacy 3. Offering to discuss information about her condition 5. Sitting quietly by her bed and hold her hand

Your patient has just been told that she has cancer, and she is crying. Which actions facilitate therapeutic communication? (Select all that apply.) 1. Turning on the television to her favorite show 2. Pulling the curtain to provide privacy 3. Offering to discuss information about her condition 4. Asking her why she is crying 5. Sitting quietly by her bed and hold her hand

A GCS between 9 and 12 indicates...

a moderate coma.

2nd degree AV block: type 2 or mobitz2

no change in PR interval, QRS drops

variable decelerations

no correlation with contractions; decrease of 15 bpm lasting 15 sec and return to baseline less than min; indicative of cord compression(change mothers position)

Pancreatitis: diet:

no etoh and caffeine, low fat bland diet, small frequent meals.

A complete spinal cord injury results in...

no function below the level of injury.

Appendicitis: plan

no heatin pads, enemas or laxative pre-op, Maintain NPO, Ice bag to abdomen to alleviate pain, sudden cessation of pain indicates rupture of appendix

complete abortion

no retained tissue. NC: methergine

Signs of withdrawal related to hallucinogen abuse...

none known


Ensembles d'études connexes

Psychiatric/Mental Health Pharmacology

View Set

LearnningCurve 16b- Evaluating Psychotherapies

View Set